MULTIPLE CHOICE QUESTIONS-1

1. Consider the following hemodynamic data: left atrial pressure, 15 mmHg; LV pressure, 220/15 mmHg; aortic pressure, 100/60 mmHg; and cardiac index, 1.9 L/minute/m2. These are most consistent with which valvular lesion?
A) Tricuspid stenosis
B) Mitral Stenosis
C) Aortic stenosis
D) Aortic Regurgitation 
E) Tricuspid regurgitation
 
Answer: C
 
2. A 68-year-old man presents with central retrosternal chest pain on exercise. The best diagnostic option is
A) Exercise ECG
B) Coronary angiography
C) Exercise echocardiography
D) Exercise thallium imaging
E) None of the above
 
Answer: B
 
3. A 35-year-old woman presented with tender lesions on both legs. She had no respiratory symptoms and was not on any medications. O/E: she was febrile, pulses 90 regular and blood pressure 136/88. Her chest was clear. There were bilateral erythematous raised lesions on her shins. Chest x-ray shows bi-hilar lymphadenopathy. The most helpful test that would give a definitive diagnosis would be: 

A) Kveim test 
B) Serum ACE level 
C) Serum calcium 
D) Skin biopsy 
E) Transbronchial needle biopsy

Answer: E

4. A 78-year-old man complains of increasing fatigue and bone pain, especially around the knees and ankles. He has long-standing anemia with hemoglobin of 9 to 10 g/dL and MCV of 102. He had not responded to therapeutic trials of iron and vitamin B12, but had been symptomatically stable until the past month. Examination reveals pallor and spleen tip just palpable at the left costal margin. CBC reveals a hemoglobin of 8.2 g/dL, but for the first time his platelet count is low (15,000); the white blood cell count is 14,000. What is the likely cause of his worsening anemia?
A) Folic acid deficiency 
B) Acute myeloid leukemia 
C) Myelofibrosis 
D) Tuberculosis 
E) Viral infection

Answer: B

5.  A 58-year-old man with history of chronic myeloid leukemia was admitted with pneumonia and deep venous thrombosis. He was started on antibiotics and I.V. heparin. His condition deteriorated and had acute respiratory distress syndrome and hypotension. Despite large amounts of i.v. fluids and inotropes, he remained hypotensive. You are considering adrenal insufficiency. What should you do next?
A) Order random cortisol then treat with hydrocortisone
B) Start hydrocortisone then do the ACTH stimulation test
C) Start dexamethasone then do the ACTH stimulation test
D) Start hydrocortisone
E) Do the ACTH stimulation test; treat according to the results

Answer: C

6. A 68-year-old diabetic woman with chronic renal insufficiency is admitted to the hospital with urosepsis. She has a history of anaphylactic reaction to penicillin that required intubation for severe bronchospasm. Urinalysis shows > 100 WBC, and Gram's stain shows 3+ plump gram-negative rods.
Which of the following antibiotics would be most appropriate for this patient with a known severe penicillin allergy?
A) Ampicillin
B) Ceftazidime
C) Vancomycin
D) Aztreonam
E) Imipenem

Answer: D

7. 65-year-old man with a history of chronic AF presents to ER with acute memory impairment (recent amnesia), confusion, and impaired vision. Physical examination confirms the confusion state and right homonymous hemianopsia. No paresis was noted. The current status is most probably related to the occlusion of: 
A) Left posterior cerebral artery 
B) Basilar artery 
C) Left middle cerebral artery (posterior branch) 
D) Left common carotid artery 
E) Right vertebral artery

Answer: A

8. A 24-year-old man is in the operating room for a massive liver injury sustained when his motorcycle hit a truck. After one hour of surgery he has received 15 units of packed cells and has developed diffuse oozing from the surface of his liver. Clots are no longer forming. His body temperature is 34oC.
Coagulation abnormalities expected in this patient include all of the following EXCEPT 
A) Prolonged PT
B) Prolonged aPTT
C) Prolonged bleeding time
D) Low fibrinogen level
E) Viscoelastic test (point of care coagulation testing) is usually normal.

Answer: E

9. A 42-year-old female with a recent diagnosis of systemic sclerosis, is referred to hospital with a complaint of headaches and blurred vision. She has a past medical history of asthma. On examination, her blood pressure is 230/120, and there is bilateral papilloedema. Which of the following medications should be prescribed immediately?
A) IV Furosemide
B) IV Labetolol
C) Oral angiotensin II receptor blockers plus IV Sodium Nitroprusside
D) Oral captopril plus IV Sodium Nitroprusside
E) Sublingual Nimodipine

Answer: D

10. A 55 year old patient with acute necrotizing pancreatitis has been in the Intensive Care unit with multiple organ failure for approximately 4.5 weeks. Although his clinical situation improved over the last few days, the patient now deteriorates. There is fever up to 104 °F, a rising CRP of 398 and white blood cell count of 27 x 109/L. A contrast enhanced CT demonstrates a large, heterogeneous, walled off, peripancreatic collection with gas inside. What is the preferred treatment strategy at this point?
A) Conservative treatment
B) Image-guided percutaneous or endoscopic catheter drainage
C) Primary open necrosectomy
D) Primary minimally invasive surgical necrosectomy
E) Video assisted retroperitoneal debridement.

Answer: B

11. A 25-year-old female is admitted with acute dyspnea and chest pain. A diagnosis of pulmonary embolism is confirmed and her investigations reveal urine dipstick protein ++ but no blood, anti-double stranded DNA antibodies of 200 U/mL (0 - 73), with a 24 hour urinary protein concentration of 5g (< 0.2). Which one of the following diagnoses is most likely to be found on renal biopsy?
A) AA amyloid
B) Focal segmental glomerulonephritis
C) IgA nephropathy
D) Membranous nephropathy
E) Minimal change nephropathy.

Answer: D

12. A 46-year-old woman is referred for preoperative evaluation before undergoing resection of newly diagnosed glioblastoma. Her HB is 12 gm/dl, leukocyte count 6.7 x109/L, and PLT 198 x109/L, the surgeon is particularly interested in an opinion concerning her risk of bleeding. Which of the following will provide the best estimate of her surgical risk of bleeding? 
A) Bleeding time 
B) PLT function analyzer-100 analysis 
C) PLT aggregation studies 
D) Medical history, including outcomes of previous surgical procedures 
E) PT and partial thromboplastin time.

Answer: D

13. A 42-year-old woman presents with acute onset of headache and neck pain vertigo, nausea and vomiting. Neurological examination reveals left nystagmus, left Horner syndrome and absent left gag reflex. Left appendicular ataxia and anesthesia to pin prick in the left face and right arm and leg. Which of the following is the most likely diagnosis? 
A) MCA 
B) PCA 
C) ACA 
D) dissection of aorta 
E) Vertebral artery dissection

Answer: E

14. A 30-year-old man is evaluated for a thyroid nodule. The patient reports that his father died from thyroid cancer and that a brother had a history of recurrent renal stones. Blood calcitonin concentration is 2000 pg/mL (normal is less than 100); serum calcium and phosphate levels are normal. Before referring the patient to a surgeon, the physician should do which of the following?
A) Obtain a liver scan 
B) Perform a calcium infusion test 
C) Measure urinary catecholamines 
D) Administer suppressive doses of thyroxine and measure levels of thyroid stimulating hormone 
E) Treat the patient with radioactive iodine

Answer: C

15.  All of the following vaccine can be given to pregnant lady except: 
A) Flu vaccine 
B) H1N1 
C) Hepatitis B vaccine 
D) Rubella vaccine 
E) Tetanus vaccine

Answer: D

16. A 78-year-old man is evaluated in the hospital for poor glycemic control before undergoing femoral popliteal bypass surgery. He has been on the vascular surgery ward for 3 weeks with a non-healing foot ulcer. The patient has an extensive history of arteriosclerotic CV disease, including PVD, and a 20 year history of DM 2, his most recent Hb A1c obtained 2 months before admission was 8.9%, his diabetes regimen consists of glibenclamide 15 mg/day. While in the hospital his plasma glucose levels have generally been in the 200 to 250 mg/dl range he is eating well. In addition to stopping glibenclamide, which of the following is the most appropriate treatment for this patient? 
A) Basal insulin and rapid acting insulin before meals. 
B) Insulin infusion 
C) NPH insulin twice daily 
D) Sliding scale regular insulin 
E) Insulin glargine once daily

Answer: A

17. One of flowing statement regarding COPD is true: 
A) Most smokers develop COPD during their life 
B) Tiotropium is as effective as smoking cessation in reduction the rate of decline in FEV1. 
C) Long term O2 therapy improves survival in all COPD treatment with FEV1 less than 50% of predicted. 
D) Rehabilitation may improve survival in patients with severe COPD 
E) Inhaled corticosteroids decreases exacerbation rate of COPD patients

Answer: E

18. What is the best laboratory goal to use to guide fluid resuscitation in acute pancreatitis?
A) Hematocrit
B) Blood urea nitrogen
C) Serum creatinine
D) Amylase
E) Lipase

Answer: A

19. A 62-year-old female with a history of a recent pulmonary embolus presents to your office for follow-up on anticoagulation treatment. She takes warfarin on a daily basis. She reports that for the last week she has noticed mild rectal bleeding and multiple bruises over the extremities with minimal trauma. She is comfortable appearing with normal vital signs and is not orthostatic. You ordered a stat CBC and PT/INR which revealed a mildly decreased Hgb at 11 g/dL and an elevated INR of 7. Which of the following would be the most appropriate intervention? 
A) Fresh frozen plasma 
B) Withhold warfarin
C) Intravenous vitamin K 
D) Reduce warfarin dose 
E) Oral vitamin K

Answer: E

20. Which of the following produces the greatest increase in bone mineral density (BMD) in patients with osteoporosis? 
A) Estrogen 
B) Calcitonin 
C) Alendronate 
D) Teriparatide 
E) Raloxifene

Answer: D

21. A 34-year-old woman was found to have a 2-cm right thyroid nodule at the time of a well woman examination. The remainder of the thyroid was palpably normal and there were no lymph nodes palpable. There was no history of thyroid disease or radiation therapy to her head or neck. She was clinically euthyroid. Thyroid-stimulating hormone (TSH) was normal. Which of the following tests would be the most useful in establishing a specific diagnosis? 
A) Ultrasound of the thyroid 
B) Nuclear scan of the thyroid 
C) Thyroid antibody studies 
D) Fine needle aspiration of the nodule 
E) CT of the neck

Answer: D

22.  A slim, healthy 30-year-old woman is scheduled for a dental prosthodontic procedure and was sent for medical evaluation of a known history of mitral valve prolapse (MVP). The patient is a highly active individual and denies palpitations, chest pain, or shortness of breath. She admits to having a family history of heart disease, notably her father, who had died of a heart attack in his forties, and her mother, who had mitral valve prolapse. On physical examination, the patient is comfortable and has normal vital signs. Auscultation of the heart reveals a normal S1 and S2 and a prominent midsystolic click, which is accentuated in the standing position. No systolic murmur is appreciated. What is your overall assessment and plan for this patient?
A) Get an echocardiogram to evaluate mitral valve motion and blood flow prior to clearing her for the procedure
B) Prescribe empiric antibiotics for endocarditis prophylaxis and clear her for the procedure
C) Get a cardiology consultation prior to medical clearance because the patient has a significant family history of heart disease
D) Clear her for the procedure without endocarditis prophylaxis
E) Clear her for the procedure with endocarditis prophylaxis

Answer: D 

23. A 35-year-old man complains of chest pain, which began following the use of cocaine 30 minutes prior to arrival. The patient describes severe substernal chest pressure, radiating to the left arm and jaw. It is associated with shortness of breath. Initial vital signs: temperature 100.4°F, heart rate 120/min, respiratory rate 20/min, blood pressure 185/100 mm Hg, pulse oximetry 98% on room air. An ECG is consistent with acute myocardial infarction. A drug which is contraindicated is:
A) Oxygen.
B) Aspirin.
C) Lorazepam.
D) Bisoprolol.
E) Morphine

Answer: D

24. A 55-year-old man with a history of alcoholism complains of more than a month of malaise, low-grade fever, and a productive cough with greenish sputum tinged with blood. Examination shows periodontal disease with bad breath and clubbing of fingers. On chest x-ray, there is a 2 cm cavity with an air–fluid level in the posterior segment of the right upper lobe. Sputum smear shows many neutrophils and a variety of bacteria. Appropriate treatment includes:
A) Isolate the patient and initiate a four-drug antituberculosis treatment.
B) Start intravenous clindamycin.
C) Refer the patient to a dentist for periodontal care.
D) Schedule a bronchoscopy for the next day.
E) Start administration of methicillin and tobramycin

Answer: B

25. A previously healthy 54-year-old man presents to the emergency department complaining of chest pain. His ECG shows an acute inferior wall myocardial infarction. His blood pressure is 90/60 mm Hg. On physical examination, he has jugular vein distention and clear lungs. You should treat him immediately with which of the following:
A) Intravenous fluids.
B) Norepinephrine.
C) Dopamine.
D) Nesiritide.
E) Nitroprusside

Answer: A

26. Inhalant abusers (“huffers” and “baggers”) are at risk for:
A) Ventricular dysrhythmias.
B) Acute lung injury.
C) Hypoglycemia.
D) Acidosis.
E) Kidney failure

Answer: A

27. A 37-year-old woman has a 2-week history of intermittent headache and general malaise. Over the last 24 hours, she has developed back pain, hematuria, vomiting, fever, and confusion. She denies recent travel or insect bites. There is no history of dysuria, urgency, frequency, or kidney stones. Her oral temperature is 38°C and her heart rate is 100/min. Physical findings include pale conjunctivae, borderline tachycardia, bilateral costovertebral tenderness, and several purpuric skin lesions. Her urine dipstick is strongly positive for hemoglobin, but negative for nitrites and leukocyte esterase. The test that will most likely reveal the correct diagnosis is:
A) Intravenous pyelogram.
B) CBC with differential and peripheral smear.
C) Complete urinalysis with microscopic examination.
D) Blood cultures.
E) Liver function tests

Answer: B

28. A 52-year-old man complains of severe right knee pain. He recalls no trauma, but attended a wine tasting party last weekend. He denies past medical history and is afebrile with normal vital signs. Physical examination reveals a swollen, red, painful right knee. Radiograph shows an effusion, but no bony erosions. You perform arthrocentesis and obtain 27 mL of cloudy straw-colored fluid, which you send to the laboratory for studies. The cell count is 50,000 WBCs/mm3 with 85% PMNs, glucose 120 mg/dL (serum 130), and protein 3.5 g/dL. The Gram stain shows numerous WBCs, but is negative for organisms. Evaluation for crystals shows numerous needle-shaped crystals with negative birefringence. A good treatment plan would be:
A) Oral colchicine 0.5 mg every hour until symptoms abate, GI toxicity develops or the maximum dose of 6 mg in 24 hours is reached. 
B) Subcutaneous colchicine 1 mg plus oral probenecid 250 mg bid for 1 week.
C) Sublingual colchicine 2 mg followed by oral steroids tapered over 5 days.
D) Intra-articular triamcinolone injection.
E) Oral allopurinol 500 mg tid for 1 week

Answer: A

29. A 35-year-old woman presents to her gynecologist with complaints of burning on urination for the past 2 days. Dipstick test of her urine demonstrates marked positivity for leukocyte esterase, but no reactivity for nitrite. Urine culture later grows out large numbers of organisms. Which of the following bacteria are most likely to be responsible for this patient's infection?
A) Enterobacter sp.
B) Enterococcus faecalis
C) Escherichia coli
D) Klebsiella pneumoniae
E) Pseudomonas aeruginosa

Answer: B

30. A 23 year old single woman referred with 3-month history of weight loss and heat intolerance. On examination pulse is 120 beat/min regular, blood pressure 120/72 mmHg, weight 58 kg, height 165 cm. she had diffuse goiter with bilateral exophthalmos. Investigations: serum free T4 3.9 ng/dl (N: 0.9-2.0), serum TSH 0.001 mu/l (N: 0.4-4.0), TSH receptor antibody 8 U/L(N:<2). She was started on methimazole 30 mg/day. Six weeks later she developed severe migratory arthralgia associated with the presence of palpable purpura over both legs and dermal infarcts in the finger tips. Laboratory studies shows ESR 54 mm/h, urine showed microscopic hematuria, ANCA titer was 1:640, antimyeloperoxidase antibodies 112U/l (N:0-9), antiproteinase 3 antibodies 6.4 U/l (N:0-3.5).
The best action is:
A) Continue methimazole and add NSAID.
B) Continue methimazole and add prednisolone 40 mg per day.
C) Substitute propylthiouracil for methimazole.
D) Refer the patient for definitive therapy with radioactive iodine.
E) No intervention

Answer: D

31. A 16-year-old girl consults you because of progressive pallor and poor exercise tolerance. She has had dysfunctional uterine bleeding for several months. Her haemoglobin is 6.5 g/dl. Which one of the following would be inconsistent with iron-deficiency anaemia in this girl?
A) Thrombocytosis
B) Low serum iron
C) Low serum ferritin
D) Reticulocyte percentage 10%
E) Transferrin saturation 9%

Answer: D

32. A 48-year-old auto mechanic presents to the clinic with complaints of many years of “pins and needles” in his left hand that initially occurred only while working but have worsened substantially. He claims the pain wakes him almost every night. On examination, marked weakness and wasting of the left hand muscles are evident. What is the patient’s most likely diagnosis?
A) Amyotrophic lateral sclerosis
B) Angina
C) Carpal tunnel syndrome
D) Multiple sclerosis
E) Myasthenia gravis

Answer: D

33. Three months following laparoscopic cholecystectomy, a 50-yearold woman presents with right upper abdominal quadrant pain, excessive flatulence and intolerance to fatty food. Abdominal ultrasound and ERCP fail to detect any residual stone or biliary stricture. What is the cause of this woman’s presentation?
A) Small common bile duct stones
B) Cholangiocarcinoma
C) Peri-ampullary adenocarcinoma
D) Post-cholecystectomy syndrome
E) Choledochal cyst

Answer: D

34. All of the following clinical findings are consistent with severe mitral stenosis except
A) atrial fibrillation
B) opening snap late after S2
C) pulmonary vascular congestion
D) pulsatile liver
E) right-ventricular heave

Answer: B

35. A 50-year-old man has a blood pressure of 160/105 mm Hg, which is repeatedly confirmed to be high. He has migraine without aura for which he takes sumatriptan during pain episodes. Which is the best antihypertensive medication for this patient to be given initially?
A) Amlodipin 
B) Propranolol
C) Hydrochlorothiazide
D) Losartan
E) Alpha methyldopa

Answer: B

36. All of the following represent examples of hypothalamic-pituitary negative feedback EXCEPT:
A) Cortisol on the CRH-ACTH axis
B) Gonadal steroids on the GnRH-LH/FSH axis
C) IGF-1 on the growth hormone–releasing hormone (GHRH)-GH axis
D) Renin-angiotensin-aldosterone axis
E) Thyroid hormones on TRH-TSH axis

Answer: D

37. You are investigating the cause for a patient’s anemia. He is a 50-year-old man who was found to have a hematocrit of 25% on routine evaluation. His hematocrit was 47% 1 year ago. Mean corpuscular volume is 80, mean corpuscular hemoglobin concentration is 25, mean corpuscular hemoglobin is 25. Reticulocyte count is 5%. Review of the peripheral blood smear shows marked numbers of polychromatophilic macrocytes. Ferritin is 340 μg/L. What is the cause of this patient’s anemia?
A) Defective erythroid marrow proliferation
B) Extravascular hemolysis
C) Intravascular hemolysis
D) Iron-deficiency anemia
E) Occult gastrointestinal bleeding

Answer: A

38. A 23-year-old Japanese man attends a party where he drinks three glasses of wine. In a short period of time, he develops facial erythema and experiences severe facial flushing. Which of the following is the most likely diagnosis?
A) Alcohol dehydrogenase deficiency
B) Glucoronyl tranferase deficiency
C) Aldehyde dehydrogenase deficiency
D) Angioedema
E) Photosensitivity reaction

Answer: C

39. A 46-year-old woman with mitral stenosis presents with dense right-sided hemiplegia and global aphasia. Her radial pulse is rapid and irregular. Brain CT scan shows cerebral infarction and she is admitted to the neurology ward to receive treatment for embolic stroke. Which one of the following arteries is likely to be occluded?
A) Main stem of the left middle cerebral artery
B) Lower posterior branch of the right middle cerebral artery
C) Left anterior cerebral artery
D) Top of the basilar artery
E Left vertebral artery

Answer: A

40. Which of the following is consistent with a diagnosis of subacute thyroiditis?
A) A 38-year-old female with a 2-week history of a painful thyroid, elevated T4, elevated T3, low TSH, and an elevated radioactive iodine uptake scan
B) A 42-year-old male with a history of a painful thyroid 4 months ago, fatigue, malaise, low free T4, low T3, and elevated TSH
C) A 31-year-old female with a painless enlarged thyroid, low TSH, elevated T4, elevated free T4, and an elevated radioiodine uptake scan
D) A 50-year-old male with a painful thyroid, slightly elevated T4, normal TSH, and an ultrasound showing a mass

Answer: B

41. A 65-year-old obese man presents with recurrent episodes of cough. His wife said his coughing attacks disturb her sleep almost every night. He noticed that his voice is hoarse in the morning. He has never smoked and has no history of cardiac or respiratory problems. There is no history of allergy and the chest X-ray is normal. The procedure most likely to yield crucial diagnostic information is 
A) Pulmonary function tests 
B) 24-hour pH monitoring of the lower oesophagus 
C) High-resolution CT scan of the chest 
D) Barium swallow 
E Indirect laryngoscopy

Answer: B

42. A 38-year-old man has a six-month history of diarrhoea, abdominal pain and tenesmus. Flexible sigmoidoscopy reveals proctitis. A biopsy specimen shows acute and chronic inflammation. Each of the following statements is true about this disorder, except 
A) There is an increased risk of gastrointestinal malignancy 
B) Other family members are likely to develop the same illness 
C) Steatorrhoea is likely to develop as a result of pancreatic insufficiency 
D) The likelihood of renal stones will increase 
E) The likelihood of gallstones will increase

Answer: C

43. A 56-year-old man is referred with dyspnoea and cough productive of putrid green sputum of four weeks duration. The chest X-ray shows cavity formation in the left lower lobe. He has advanced motor neurone disease. This clinical picture is most likely to be caused by infection with 
A) Legionella pneumophila 
B) Mycoplasma pneumoniae 
C) Streptococcus pneumoniae 
D) Anaerobic bacteria
E) Common cold virus

Answer: D

44. A 50-year-old publican is referred with anuria. The initial renal function test showed the urea at 45 mmol/I and the creatinine at 480 μmoI/l. A urine sample from a catheter was strongly positive for myoglobin. This clinical picture can develop as a result of each of the following clinical situations except 
A) Alcohol ingestion 
B) Hypokalaemia 
C) Barbiturate overdose 
D) Hypophosphataemia 
E) Hypocalcaemia

Answer: E

45. A 75-year-old woman is admitted with left lower lobe pneumonia. She was given a course of cephalosporin antibiotics and she improved. However, at the end of the course she started to have diarrhoea, passing a watery stool. Investigations confirmed the diagnosis of pseudomembranous colitis. Each of the following oral medications is beneficial in treating this condition, except 
A) Vancomycin 
B) Bacitracin 
C) Colestyramine 
D) Metronidazole  
E) Gentamicin

Answer: E

46. A 38-year-old woman has dyspnoea on exertion and orthopnoea. Examination reveals a loud first heart sound, a diastolic rumble, and a large v wave in the jugular pulse. The most likely diagnosis is
A) Mitral stenosis and tricuspid regurgitation 
B) Isolated mitral stenosis 
C) Aortic regurgitation 
D) Mitral regurgitation and tricuspid regurgitation 
E) Tricuspid regurgitation

Answer: A

47. A 40-year-old man has a six-month history of increasing shortness of breath and wheeze. The full blood count shows an increased absolute eosinophil count. Chest X-ray and a CT scan of the chest show right apical bronchiectasis. Which one of the following is the most likely diagnosis?
A) Loeffler's syndrome 
B) Allergic bronchopulmonary aspergillosis 
C) Polyarteritis nodosa 
D) Nitrofurantoin hypersensitivity 
E) Strongyloides stercoralis infection

Answer: B

48. A 22-year-old woman has a three-month history of fatigue of insidious onset, poor appetite and athralgia. She has no previous history of liver disease. She takes thyroxine replacement therapy, Physical examination reveals spider naevi and hepatomegaly, The ALT is five-times higher than the normal. Her disease is most likely be associated with 
A) Antimitochondrial antibodies 
B) Low serum caeruloplasmin levels 
C) Anti-parietal cell antibodies 
D) Anti-smooth muscle antibodies 
E) Hepatitis B surface antigen (1-HbsAg )

Answer: D

49. A 35-year-old woman has slurred speech and a progressive gait disturbance. Magnetic resonance imaging (MRI) shows several focal abnormalities in the periventricular areas, normal sized ventricles and no space occupying lesion. The cerebellum was also normal. The most likely diagnosis is 
A) Motor neurone disease 
B) Friedreich's ataxia 
C) Multiple sclerosis (MS) 
D) Syphilis 
E) Huntington's disease

Answer: C

50. An 85-year-old man is referred with a three-month history of dysphagia. Barium swallow showed a filling defect in the oesophagus. Gastro-oesophagoscopy and biopsy confirmed the presence of squamous cell carcinoma of the oesophagus. Each of the following conditions might be associated with this tumour, except 
A) Achalasia 
B) Smoking 
C) Tylosis 
D) Barrett's oesophagus 
E) Head and neck cancer

Answer: D

51. An 84-year-old woman presented with tiredness. On examination, she was anaemic but had no palpable splenomegaly. Investigations revealed a haemoglobin of 9.7 g/dL (11.5 - 16.5). She was commenced on oral iron therapy for one month and her haemoglobin remained unchanged. Further investigations revealed:
MCV 102 fL (80 - 96)
blood film marked anisopoikilocytosis
serum ferritin 70 ug/L(15 - 300)
Vitamin B12 280 ng/L (160 - 760)
red cell folate 230 ug/L (160 - 640)
serum urea 9.1 mmol/L (2.5-7.5)
serum creatinine 150 umol/L (60 - 110)
What is the most likely diagnosis?
A ) aplastic anaemia
B ) anaemia due to renal disease
C ) hypothyroidism
D ) iron deficiency anaemia
E ) sideroblastic anaemia

Answer: E

52. A 29-year-old male presents with symptoms of severe gastro-oesophageal reflux. Which one of the following is most useful in assessing the role of surgery?
A ) cardiac sphincter manometry
B ) gastric emptying study
C ) intragastric pH monitoring off therapy
D ) oesophageal motility study
E ) oesophageal pH monitoring on therapy

Answer: D

53. An 80 year old man with a 5 year history of diet controlled type 2 diabetes mellitus presents with a one month history of cough and weight loss. He was a non-smoker and had difficulty expectorating. Investigation revealed a HbA1c of 7% but his chest X-ray showed a cavitating left apical shadow. Which of the following investigations would be most useful in establishing the cause of this lesion?
A ) bronchoscopy
B ) CT scan of the chest
C ) Gastric aspirate for acid-fast bacilli
D ) Percutaneous lung biopsy 
E ) Sputum for acid-fast bacilli

Answer: A

54. A 68 year old male diagnosed with nephritic syndrome receives steroid therapy without benefit. His investigations show an albumin of 20 g/L (37 - 49), Total cholesterol of 12 mmol/l, dipstick urinanalysis reveals +++ protein and a renal biopsy shows focal segmental glomerulosclerosis. Which one of the following is most likely to preserve renal function?
A ) dietary salt restriction
B ) low dietary protein intake
C ) ramipril
D ) simvastatin
E ) warfarin

Answer: C

55. Which of the following is characteristically inherited in an autosomal recessive manner?
A ) Achondroplasia
B ) Adult polycystic kidney disease
C ) C1 esterase deficiency
D ) Familial hypercholesterolaemia
E ) Friedreich's ataxia

Answer: E

56. A clinical trial assessing a new lipid lowering therapy for stroke allocates 1000 patients to active treatment and another 1000 patients to placebo. Results demonstrate that number needed to treat (NNT) is 20 for the prevention of the primary end-point. Which of the following best describes the results?
A ) 20 patients in the treatment group were protected from stroke.
B ) 20 extra patients in the placebo group had a stroke
C ) For 1000 patients treated with active therapy, there would be 20 fewer strokes
D ) For 1000 patients treated with active therapy, there would be 50 fewer strokes.
E ) For every 1000 patients treated with active therapy there would be 100 fewer strokes

Answer: D

57. A 70 year old male presents with haemoptysis. Bronchoscopy reveals a tumour in the proximal right main bronchus. Which of the following is a contraindication to radical radiotherapy?
A) Adenocarcinoma
B) FEV1 of 25% predicted
C) Involvement of the pulmonary artery
D) Ischaemic heart disease
E) Superior vena caval obstruction

Answer: B

58. Which of the following statements concerning iron metabolism is correct?
A) Approximately 0.1% of body iron circulates in the plasma
B) Approximately 90% of dietary iron is absorbed in the intestine
C) The main route of excretion is the liver
D) The serum ferritin concentration is reduced characteristically following surgery
E) The transferrin content of intestinal mucosal cells is high when body iron stores are high

Answer: A

59. A 35 year-old woman with type 1 diabetes mellitus presents for annual assessment. Which one of the following features on fundoscopy would require urgent referral to anophthalmologist?
A) asteroid bodies
B) hard exudates in the macular region
C) intraretinal microvascular abnormalities
D) scattered microaneurysms
E) soft exudates

Answer: B

60. A 34-year-old man presented for an insurance medical. He was symptom free, but clinical examination suggested a small ventricular septal defect. Which one of the following findings was most likely to have been present?
1) An early diastolic murmur
B) A short systolic murmur at the left sternal edge
C) A systolic murmur maximal at the apex
D) A systolic murmur with a thrill at the left sternal edge
E) Fixed splitting of the second heart sound

Answer: D

61. A previously well 28-year-old woman presents with a history of mild nosebleeds and easy bruising for 8 days. She denied a history of abnormal bleeding and had been perfectly healthy, participating in a soccer league. There were no recent upper respiratory illnesses, no recent travel, and no significant trauma other than on the soccer field. Her only medication was a multivitamin pill with iron because of menorrhagia. Physical examination revealed a thin, healthy woman with normal vital signs. There were a few petechiae in her conjunctivae and mouth, and a modest-sized, healing bruise on her abdomen. There was no hepatosplenomegaly or palpable lymphadenopathy. Her laboratory findings included a Hgb of 124 g/L, WBC of 6.8 × 109/L, and platelet count of 9 × 109/L, and her blood film was normal except for markedly decreased but large platelets. Which of the following tests are indicated in her evaluation?
A) Marrow examination
B) Helicobacter pylori evaluation
C) ANA, lupus anticoagulant, and APL antibodies
D) HIV and hepatitis C virus (HCV) titers

 Answer: D

62. A 73-year-old woman had a middle cerebral artery stroke and developed recurrent seizures requiring medication. She then developed pneumonia and respiratory failure and required an intensive care unit (ICU) admission for mechanical ventilation. During her ICU stay, the patient received her usual anticonvulsants and aspirin. In addition, she required antibiotics, prophylactic unfractionated heparin for thromboprophylaxis, sedation, and diuretics. Because of worsening seizures, valproic acid was added on ICU day 4. On day 12, the pneumonia was thought to be resolving, but an extubation trial failed, and the platelet count was noted to have gradually fallen to 36 × 109/L over the prior 6 days. What is included in the differential diagnosis?
A) Sepsis
B) ITP
C) Drug-induced thrombocytopenia
D) Heparin-induced thrombocytopenia
E) All of the above

 Answer: E

63. A 14-year-old boy presents with recurrent respiratory and skin infections. Staphylococcus aureus has been cultured from his skin lesions. He has had no other medical issues. His growth and development have been normal. At physical examination, there are several marks and scars of past skin infection on his face, and he has a skin eruption on his trunk compatible with eczematoid dermatitis. There are no enlarged lymph nodes, and the sizes of his liver and spleen are normal. Laboratory examination reveals a hemoglobin of 143 g/L and white blood cells of 4.5 × 109/L with a normal white cell differential count. Serum levels of immunoglobulins are IgA of 2.1 g/L (reference range, 0.8–3.5 g/L), IgG of 12.4 g/L (reference range, 6–16 g/L), and IgM of 1.9 g/L (reference range, 0.4–2.5 g/L).
What would be the best diagnostic test to confirm the most likely diagnosis?
A) Complement factors and mannan-binding lectin (MBL)
B) Serum immunoglobulin E (IgE) concentration
C) A phorbol myristate acetate (PMA) nitroblue tetrazolium (NBT) test
D) Flow cytometry of blood neutrophils using CD11 and CD18

Answer: B

64. You are treating a 58-year-old man with hypopituitarism following radiation therapy for craniopharyngioma. He is taking hydrocortisone sodium succinate, 15 mg daily; levothyroxine, 0.15 mg daily; and testosterone injections, 200 mg every 2 weeks. He feels weak and tired. His examination is remarkable only for a BP of 95/58 mmHg. Sodium is 131 mEq/L; potassium, 4.8 mEq/L; TSH, 0.23 μIU/mL; and FTI, 9.0 μg/dL. Which of the following would you do next?
A) Decrease levothyroxine
B) Increase testosterone
C) Add fludrocortisone
D) Increase hydrocortisone
E) Begin desmopressin acetate
 
Answer: D
 
65. A patient being screened for intermittent diarrhea has a T4 (total) value of 19.6 μg/dL (normal, 5.0 to 10.5). No other features of hyperthyroidism are present; no goiter is present. A T4U test is elevated at 2.01 (normal, 0.8 to 1.20), whereas the T3RU test is subnormal at 15% (normal, 25% to 35%). The free thyroxine index is calculated to be 9.8 (normal, 5.0 to 10.5). What single test would be most helpful in delineating the patient’s thyroid status?
A) Thyroid-stimulating hormone (TSH)
B) Thyroid receptor antibodies
C) Serum T3
D) Serum FT4 equilibrium dialysis
E) None of the above
 
Answer: A
 
66. You are treating a 58-year-old man with hypopituitarism following radiation therapy for craniopharyngioma. He is taking hydrocortisone sodium succinate, 15 mg daily; levothyroxine, 0.15 mg daily; and testosterone injections, 200 mg every 2 weeks. He feels weak and tired. His examination is remarkable only for a BP of 95/58 mmHg. Sodium is 131 mEq/L; potassium, 4.8 mEq/L; TSH, 0.23 μIU/mL; and FTI, 9.0 μg/dL. Which of the following would you do next?
A) Decrease levothyroxine
B) increase testosterone
C) Add fludrocortisone
D) Increase hydrocortisone
E) Begin desmopressin acetate
 
Answer: D
 
67. A 33-year-old anthropologist from New York had been doing research in a desert region of Arizona for about 6 months. After returning home, he visits his physician complaining of an influenza-like illness with cough, mild chest pain, and occasional fever. He says that the illness started during the last few weeks of his stay in Arizona. Red, tender nodules are present on his shins. Chest x-rays fail to reveal evidence of pulmonary infiltrates or pleural effusion. Which of the following is the most appropriate next step in the management of this patient?

A) Delay treatment until culture results are obtained.
B) Begin treatment with fluconazole.
C) Begin treatment with amphotericin B.
D) Aspirate bone marrow and culture.
E) Institute immediate isolation.

Answer: A

68. A 22-year-old professional female basketball player was noted to have an unusual murmur on her sports physical before the season began. The doctor thought he heard a continuous murmur at the left upper sternal border associated with a slightly widened pulse pressure and brisk to abounding pulses. The most likely diagnosis is
A) VSD
B) ASD
C) Coarctation of the aorta
D) Patent ductus arteriosus (PDA)
 
Answer: D
 
69. A 56-year-old woman with advanced arthritis of the right knee and a diagnosis of hypertension and hyperlipidemia has atypical chest pain with a normal ECG. The best diagnostic option is:
A) Stress (exercise or dobutamine) ECG
B) Coronary angiography
C) Exercise echocardiography
D) Dipyridamole-thallium imaging
E) echocardiography (ECHO)
 
Answer: D
 
70. A young man is referred by his GP following an investigation for recurrent mouth ulceration. Subsequent blood count shows HB of 13.2, WBC 3.8 (neutrophil 1.2, lymph 1.5) PLT 332, examination reveals a fit young male with no evidence of organomegaly or lymphadenopathy, further questioning reveals the history of mouth ulceration occurring over the previous 3-4 years. Select the most likely diagnosis 
A) AML 
B) Post-viral neutropenia 
C) HIV infection 
D) Drug-induced neutropenia 
E) Cyclical neutropenia.
 
Answer: E
 

MULTIPLE CHOICE QUESTIONS-2

1. You are evaluating a 58-year-old man who was admitted in the hospital 7 days ago with severe alcoholic hepatitis. He was drinking on average 12 beers daily in the last 6 months before admission. Currently, he is afebrile and normotensive. Significant jaundice and moderate ascites are noted in the physical exam. He is lethargic and moderately confused. He has been started on prednisolone 40 mg daily since admission. You correctly emphasize the effect of optimized nutrition in the patient’s outcome. Which of the following measures would have the best value in the
assessment of his nutritional status?
A) Body mass index
B) Midarm muscle circumference
C) Fist-grip dynamometry
D) Creatinine-height index
E) Serum transferrin level

Answer: B

2. Which of the following chest radiographic findings is inconsistent
with the diagnosis of asbestosis?
A) Presence of pleural plaques
B) Presence of pleural effusion
C) Reticulonodular infiltrates
D) Upper lobe predominance
E) Reduced lung volumes

Answer: D

3. A 55-year-old woman smoker is seen in the emergency room (ER) complaining of several days of increasing facial fullness, orthopnea, and swelling in her neck and hands. Physical examination is notable for obvious facial swelling, with conjunctival edema, jugular venous distention, and symmetric swelling of both upper extremities. Fullness is present in both supraclavicular fossae, but there is no clear lymph node enlargement, and the lungs are clear. The patient is tachycardic, but no gallop, murmur, or rub is present. No hepatomegaly, ascites, or pedal edema is present. Chest radiography reveals a right hilar mass. The patient is admitted to the hospital at midnight, and you order which of the following?
A) An emergency upper extremity venogram
B) An emergency computed tomography (CT) scan of the chest
C) An emergency echocardiogram (ECHO)
D) Diuretics and elevation of the head of the bed until the morning
 
Answer: D
 
4. Adults at increased risk of hepatitis B infection who should receive hepatitis B vaccine include which of the following?
A. Persons with chronic obstructive pulmonary disease
B. Persons with coronary artery disease
C. Persons with nephrotic syndrome not on dialysis
D. Persons 19 to 59 years old with diabetes mellitus
E. Persons 65 years and older
 
Answer: D
 
5. Based on the half-life and onset of action of warfarin, which of the following is the optimal management of this drug in the case of uncomplicated major surgery?
A) Warfarin should be stopped 5 days before surgery and restarted as soon as the patient can take oral medications after surgery.
B) Warfarin should be stopped 2 days before surgery and restarted as soon as the patient can take oral medications after surgery.
C) Warfarin should be stopped 5 days before surgery and restarted at least 5 days after surgery.
D) Warfarin should be stopped 2 days before surgery and restarted at least 5 days after surgery.
 
Answer: A
 

6. Untreated CML, goes into blast crisis in a median time of ?
A) 1 year
B) 2 years
C) 3 years
D) 4 years

Answer: C

7. A 61-year-old man presents with acute onset of diplopia and difficulty using his right hand. On examination, he has left third nerve palsy, right-sided tremors, and ataxia, but no choreoathetotic movements. Which of the following is the most likely diagnosis?
A) Left hemispheric infarct
B) Left pontine infarct
C) Left midbrain infarct
D) Right pontine infarct
E) Right midbrain infarct
 
Answer: C
 
8. Evaluation of a patient with hemolytic anemia reveals the presence of spherocytes on a peripheral blood smear. Which disorder is NOT in the differential diagnosis?
A) Heinz body hemolytic anemia
B) Autoimmune hemolytic anemia
C) Hereditary stomatocytosis
D) Hereditary spherocytosis
E) Liver disease
 
Answer: C
 
9. A 45-year-old woman who has been treated in the past for presently clinically inactive systemic lupus erythematosus (SLE) presents with a recent onset of fatigue, lightheadedness, and yellowness of her eyes. Her hemoglobin is found to be 6.3 g/dL (with a baseline of about 12.5-13.0), reticulocytes are 15%, indirect bilirubin is 2.8 IU/L, and LDH is 840 mg/dL. A peripheral blood smear shows a large number of polychromatic red cells and spherocytes. Her direct antiglobulin test (Coombs test) result is positive for IgG. Her treatment at this time should include
A) plasmapheresis.
B) rituximab plus danazol.
C) intravenous methylprednisolone.
D) urgent splenectomy.
E) restarting her previous SLE treatment with azathioprine.
 
Answer: C
 
10. You are consulted on a 76-year-old man who is referred for right eyelid ptosis and right pupillary constriction. He also mentions that when he exerts himself, he notices that the right side of his face does not seem to sweat like the left side. Using this information, which of the following would not be a probable cause for these symptoms?
A) A tumor affecting C8-T2 spinal levels
B) A lesion between the hypothalamus and ciliospinal center of Budge
C) Tumor in the right lung apex
D) Large hematoma formation under the subclavian artery following attempted central line placement
E) ICA dissection involving the midcervical region above the bifurcation
 
Answer: E
 

11. A 72-year-old man presents with an acutely painful right knee. On examination, he had a temperature of 37°C with a hot, swollen right knee. Of relevance amongst his investigations, was his white cell count which was 12.6 x109/l and a knee X-ray revealed reduced joint space and calcification of the articular cartilage. Culture of aspirated fluid revealed no growth. What is the most likely diagnosis?
A) Gout
B) Psoriatic monoarthropathy
C) Pseudogout
D) Rheumatoid arthritis
E) Septic arthritis

Answer: C

12. A 40-year-old female who has been prescribed thyroid replacement therapy has routine thyroid function tests. On examination she appeared clinically euthyroid with no abnormal findings. Her TFTs revealed: TSH 3.2 mU/L (0. 35 - 5.0) Total T4 20 nmol/L (55 - 144) free T4 2.6 pmol/L (9 - 24) Total T3 2.5 nmol/L (0. 9 - 2.8).
Which one of the following statements is correct?
A) Her thyroid hormone replacement is adequate.
B) Investigation of pituitary function is required.
C) She has tertiary hypothyroidism.
D) She has thyroiditis.
E) She has sick euthyroid syndrome.

Answer: A

13. Which of the following viral causes of acute hepatitis is most likely to cause fulminant hepatitis in a pregnant woman?
A) Hepatitis A
B) Hepatitis B
C) Hepatitis C
D) Hepatitis D
E) Hepatitis E

Answer: E

14. A 35-year-old healthy woman has a faint systolic murmur on physical examination. An echocardiogram is performed, and she is found to have a bicuspid aortic valve. In explaining the meaning of this finding to her, the most appropriate statement is that?
A) An aortic valve replacement is eventually likely to be required
B) Other family members are likely to have the same condition
C) She should be treated with a cholesterol lowering agent
D) The problem resulted from past injection drug usage
E) This is one manifestation of an underlying autoimmune condition.

Answer: A

15. A 54 year old man presents with central crushing chest pain. Examination is normal. 12-lead ECG shows ST segment elevation in leads II, III, aVF and ST depression in V1, V2 and V3. Which coronary artery is occluded?
A) Circumflex.
B) Left anterior descending.
C) Left main stem.
D) Obtuse marginal.
E) Right coronary artery.

Answer: B

16. Primary hyperparathyroidism may occur in association with the following conditions 
A) Chronic renal failure
B) Vitamin D deficiency
C) Gastrinoma
D) Autoimmune polyendocrine syndrome
E) Sjogren's syndrome

Answer: C

17. A 57-year-old man with a history of diabetes mellitus and chronic kidney disease with a baseline creatinine of 1.8 mg/dL undergoes cardiac catheterization for acute myocardial infarction. He is subsequently diagnosed with acute kidney injury related to iodinated contrast. All of the following statements are true regarding his kidney injury EXCEPT:
A) Fractional excretion of sodium will be low.
B) His creatinine is likely to peak within 3–5 days.
C) His diabetes mellitus predisposed him to develop contrast nephropathy.
D) Transient tubule obstruction with precipitated iodinated contrast contributed to the development of his acute kidney injury.
E) White blood cell casts are likely on microscopic examination of urinary sediment.

Answer: E

18. A 25 year-old male presents with an eight week history of difficulty walking. On examination he had increased tone and pyramidal weakness of the right leg. There was impairment of pinprick sensation in the left leg up to the groin. Which one of the following is the cause of these signs? 
A) A central cauda equina lesion.
B) A cervical spinal cord lesion.
C) A lesion at the foramen magnum.
D) A right-sided thoracic spinal cord lesion.
E) Bilateral cerebral hemisphere lesions.

Answer: D

19. An 81 -year-old man admitted with a stroke becomes increasingly drowsy after receiving nasogastric feeding for five days. Which biochemical abnormality is the most likely cause of his drowsiness?
A) Hyperglycemia.
B) Hypermagnesemia.
C) Hypernatremia.
D) Hypocalcemia.
E) Hypophosphatemia

Answer: C

20. One of the nurses working on the Care of the Elderly ward sustains a needlestick injury while taking blood from a patient. What is the most appropriate immediate management?
A) Administer prophylactic hepatitis B immunoglobulin regardless of vaccine status.
B) Exclude the nurse from performing exposure-prone procedures for three months until a negative HIV antibody test has been obtained.
C) Immediately take the nurse’s blood to test for antibodies to hepatitis B, hepatitis C and human immunodeficiency viruses.
D) Promptly administration of antiretroviral therapy.
E) Wash the wound with soap under running water.

Answer: E

21. A 65-year-old male with left ventricular systolic dysfunction was dyspneic on climbing stairs but not at rest. The patient was commenced on Ramipril and Furosemide.
Which one of the following drugs would improve the patient’s prognosis further?
A) Amiodarone
B) Digoxin
C) Diltiazem
D) Metoprolol
E) Isosorbide dinitrate.

Answer: D

22. All of the following are recognized feature of pulmonary embolism except?
A) S1Q3T3
B) An increase in serum troponin levels
C) An arterial pH greater than 7.2
D) Increased PCO2 on room airk,m
E) Positive D-dimer levels

Answer: D

23. Which of the following concerning diabetic retinopathy is correct?
A) Is unusual in Type 2 diabetic patients.
B) Improved glycemic control is more effective than hypertensive control in reducing progression of disease.
C) Normal visual acuity is seen in proliferative retinopathy.
D) Progression may be reduced with statin therapy.
E) Soft exudates are a feature of background retinopathy.

Answer: C

24. A 52 year old female presents with tiredness. There are no specific abnormalities noted on examination, but investigations reveal a T4 of 21.1 (NR 9.8 - 23), a T3 of 5.2 pmol/l (NR 3.3 - 5.5) and a TSH of 0.05 mU/l (NR 0.1 - 5 mU/l). Thyroid autoantibody titers are all undetectable. These results suggest a diagnosis of: 
A) De Quervain's thyroiditis.
B) Sick euthyroid syndrome.
C) Solitary toxic nodule.
D) Grave's disease.
E) Hashimoto's thyroiditis.

Answer: C

25. A 46-year-old woman with a 10-year history of primary progressive multiple sclerosis is admitted to the hospital for surgical debridement and wound management of a sacral pressure ulcer. The patient is bedbound and is cared for at home by her husband. Her medications include sertraline, baclofen, and oxybutynin. The remainder of the medical history is noncontributory. On physical examination, she is thin, in no distress, and has normal vital signs. There are contraction deformities of the lower extremities, and a 6- × 8-cm sacral ulcer that extends to the fascia, with minimal purulent exudates and no evidence of cellulitis. Laboratory studies include a hematocrit of 34%, leukocyte count of 15,000/µL (15 × 109/L) with 80% neutrophils, and a platelet count of 425,000/µL (425 × 109/L). Subcutaneous prophylactic unfractionated heparin is administered. Six days after initiation of heparin, her platelet count decreases to 210,000/µL (210 × 109/L), and on the ninth day after therapy, the platelet count has decreased to 95,000/µL (95 × 109/L). Which of the following is the most appropriate next step in the management of this patient?
A) Discontinue unfractionated heparin.
B) Discontinue sertraline, baclofen, and oxybutynin.
C) Discontinue unfractionated heparin and begin therapy with a direct thrombin inhibitor.
D) Discontinue unfractionated heparin and begin low-molecular-weight heparin.
E) Discontinue unfractionated heparin and begin novel oral anticoagulants.

Answer: C

26. A 75 year old man has a history of Chronic Lymphocytic Leukemia. He has had treatment with several courses of chemotherapy and has now been admitted to hospital with pneumonia. His past medical history revealed that he had suffered several previous upper respiratory tract infections over the previous six months.
Which of the following components of his immune system is likely to be deficient?
A) complement
B) immunoglobulin G
C) macrophages
D) mast cells
E) T lymphocytes

Answer: B

27. A 42-year-old man presented to the hospital with right upper quadrant pain. He was found to have multiple masses in the liver that were found to be malignant on H&E staining of a biopsy sample. Your initial history, physical examination, and laboratory tests, including prostate-specific antigen, are unrevealing. Lung, abdominal, and pelvic CT scans are unremarkable. He is an otherwise healthy individual with no chronic medical problems. Which immunohistochemical markers should be obtained from the biopsy tissue?
A) α-Fetoprotein
B) Cytokeratin
C) Leukocyte common antigen
D) Thyroglobulin
E) Thyroid transcription factor 1

Answer: B

28. A 47-year-old man is evaluated for 1 year of recurrent episodes of bilateral ear swelling. The ear is painful during these events, and the right ear has become floppy. He is otherwise healthy and reports no illicit habits. He works in an office and his only sport is tennis. On examination, the left ear has a beefy red color, and the pinna is tender and swollen; the earlobe appears minimally swollen but is neither red nor tender. Which of the following is the most likely explanation for this finding?
A) Behçet's syndrome.
B) Cogan's syndrome.
C) Hemoglobinopathy.
D) Recurrent trauma.
E) Relapsing polychondritis.

Answer: E

29. One of the following pulmonary function test results is with the respiratory disorder chronic obstructive pulmonary disease.
A) Increased total lung capacity (TLC), decreased vital capacity (VC), decreased FEV1/FVC ratio.
B) Decreased TLC, decreased VC, decreased residual volume (RV), increased FEV1/FVC ratio, normal maximum inspiratory pressure (MIP).
C) Decreased TLC, increased RV, normal FEV1/FVC ratio, decreased MIP.
D) Normal TLC, normal RV, normal FEV1/FVC ratio, normal MIP.
E) Decreased TLC, normal RV, normal FEV1/FVC ratio, increased MIP.

Answer:A

30. A 23 year old single woman referred with 3-month history of weight loss and heat intolerance. On examination pulse is 120 beat/min regular, blood pressure 120/72 mmHg, weight 58 kg, height 165 cm. she had diffuse goiter with bilateral exophthalmos. Investigations: serum free T4 3.9 ng/dl (N: 0.9-2.0), serum TSH 0.001 mu/l (N: 0.4-4.0), TSH receptor antibody 8 U/L(N:<2). She was started on methimazole 30 mg/day. Six weeks later she developed severe migratory arthralgia associated with the presence of palpable purpura over both legs and dermal infarcts in the finger tips. Laboratory studies shows ESR 54 mm/h, urine showed microscopic hematuria, ANCA titer was 1:640, antimyeloperoxidase antibodies 112U/l (N:0-9), antiproteinase 3 antibodies 6.4 U/l (N:0-3.5). The best action is:
A) Continue methimazole and add NSAID.
B) Continue methimazole and add prednisolone 40 mg per day.
C) Substitute propylthiouracil for methimazole.
D) Refer the patient for definitive therapy with radioactive iodine.
E) No intervention.

Answer: E

31. A 34-year-old man comes for neurologic consultation because of paroxysmal episodes of speech difficulty that have occurred recently. The best way to distinguish whether these are seizures or other types of events is:
A) History
B) Neurologic examination
C) Brain MRI
D) Routine EEG
E) Empiric anticonvulsant trial.

Answer: A

32. A 23-year-old woman is diagnosed with a lower extremity deep venous thrombosis. Which of the following medical conditions represents a contraindication to therapy with low-molecular-weight heparin (LMWH)?
A) Pregnancy
B) Obesity
C) Dialysis-dependent renal failure
D) Uncontrolled diabetes mellitus
E) Jaundice.

Answer: C

33. All the following forms of glomerulonephritis (GN) have associated normal serum complement C4 levels EXCEPT:
A) lupus nephritis stage IV
B) poststreptococcal GN
C) hemolytic-uremic syndrome
D) membranoproliferative GN type II
E) endocarditis-associated GN

Answer: A

34. A 60-year-old man attends clinic because of hypertension. His BP in clinic is 70/90 mmHg and his echocardiogram shows mild LVH and mild LA dilatation. He is not diabetic and has no other medical history of note.Which one of the following medications is most effective in preventing AF?
A) ACE inhibitors
B) Beta-blockers
C) Calcium-channel antagonists
D) Diuretics
E) Alpha-blocker

Answer: A

35. A 35-year-old male is seen in the clinic for evaluation of infertility. He has never fathered any children, and after 2 years of unprotected intercourse his wife has not achieved pregnancy. Sperm analysis shows a normal number of sperm, but they are immotile. Past medical history is notable for recurrent sinopulmonary infections, and the patient recently was told that he has bronchiectasis. Chest radiography is likely to show which of the following?
A) Bihilar lymphadenopathy
B) Bilateral upper lobe infiltrates
C) Normal findings
D) Situs inversus
E) Water balloon–shaped heart

Answer: D

36. All of these findings are consistent with a chronic unilateral urinary tract obstruction EXCEPT:
A) anemia
B) dysuria
C) hypertension
D) pain with micturition
E) pyuria

Answer: D

37. Which of the following is more likely to present as a myopathic disorder of gastrointestinal motility rather than as a neuropathic disorder?
A) Parkinson disease
B) Diabetes mellitus
C) Multiple sclerosis
D) Anticholinergic medication use
E) Dermatomyositis

Answer: E

38. Regarding ophthalmic complications in a patient with diabetes, which ONE of the following statements is INCORRECT?
A. There is a higher risk of vitreous haemorrhage than in the general population
B. Ocular haemorrhage is common in patients with diabetic retinopathy, following thrombolysis for acute myocardial infarction
C. Diabetic retinopathy may cause retinal detachment
D. Measurement of intraocular pressure is important because glaucoma is more common in diabetics

Answer: B

39. Carcinoid tumours of the foregut may be associated with which of the following 
A) Cushing's syndrome
B) hypercalcemia
C) pellagra
D) pulmonary hypertension
E) carcinoid syndrome

Answer: A

40. A 69-year-old woman with rheumatoid arthritis presented with swollen ankles. She was diagnosed as having rheumatoid arthritis over 18 years ago and had been relatively well controlled on non-steroidal antiinflammatory drugs until six months ago, when her joint pains and swelling required the addition of penicillamine to control her symptoms. The patient had a past history of hypertension, for which she took bendroflumethiazide. On examination she had symmetrical joint deformities consistent with rheumatoid arthritis. The heart rate was 90 beats/min and irregular. Her blood pressure measured 140/90 mmHg. The JVP was not raised. Both heart sounds were normal and the chest was clear. Abdominal examination was normal. Inspection of the lower limbs revealed pitting oedema.
Investigations are shown.
Hb 11 g/dl
WCC 5 X 109/l
Platelets 190 X109/l
Sodium 134 mmol/l
Potassium 4.5 mmol/l
Urea 6 mmol/l
Creatinine 70 micromol/l
Bilirubin 11micromol/l
Alkaline phosphatase 100 iu/l
Albumin 26 g/l
Urinalysis Protein +++
What is the management?
A) Stop penicillamine.
B) Start prednisolone.
C) Start ACE inhibitor therapy.
D) Arrange renal biopsy.
E) Arrange IVU.

Answer: A

41. You have received a blood test report with the following results: serum calcium 3 mmol/l, serum phosphate 0.6 mmol/l (normal range 0.8-1.4 mmol/l); plasma parathyroid hormone (PTH) 5.8 pmol/l (normal range 0.9-5.4 pmol/l). Which one of the following disorders could be associated with this abnormal blood test? 
A) Addison's disease 
B) DiGeorge syndrome 
C) Mucocutaneous candidiasis 
D) Medullary-cell carcinoma 
E) Magnesium deficiency

Answer: D

42. Vitiligo is associated with which one of the following disorders? 
A) Nelson's syndrome 
B) Hyperparathyroidism 
C) Amiodarone therapy 
D) Addison's disease 
E) Chronic renal failure

Answer: D

43. A 50-year-old woman is admitted with acute diarrhoea and dehydration. Just prior to her admission, she had a bout of lower urinary tract infection and is on antibiotics. Stool culture is negative and sigmoidoscopy reveals pseudomembranous colitis. Which one of the following statements is most characteristic of this patient's condition? 
A) Bloody diarrhoea, abdominal pain and tenderness is the most common presentation 
B) The detection of Clostridium difficile bacilli in the stool is diagnostic 
C) A severe form of the disease is often associated with gentamicin therapy 
D) It is caused by a Gram-positive anaerobic bacterium 
E) Intravenous vancomycin for 2 weeks is an effective treatment

Answer: D

44. A 45-year-old man is being investigated for persistent dyspepsia and heartburn. Acid secretion studies show gastric acid hypersecretion. Which one of the following conditions is the most likely cause of this patient's symptoms? 
A) Pernicious anaemia 
B) Large-bowel resection 
C) Vasoactive intestinal polypeptide- (VIP-) secreting tumour 
D) Systemic mastocytosis 
E) Cushing's syndrome

Answer: D

45. A 19-year-old student presents with acute shortness of breath and haemoptysis. She reported left loin pain and was found to have haematuria 2 weeks ago. She is under regular review by the renal physicians for nephrotic syndrome. Ultrasound of the kidney shows left renal vein thrombosis and a computed tomography pulmonary angiogram confirms acute pulmonary embolism. Which one of the following is the most likely cause of this patient's increased tendency for developing venous thrombosis?
A) Factor V Leiden mutation 
B) High levels of anticardiolipin antibodies 
C) Protein S deficiency 
D) Protein C deficiency 
E) Antithrombin III deficiency

Answer: E

46. Which of the following statements is most accurate with regard to Wilson's disease? 
A) The primary defect is believed to be enhanced intestinal absorption of copper 
B) An alternative diagnosis should be considered if chorea occurs in the absence of Kayser-Fleischer rings
C) Chronic liver disease and autoimmune haemolytic anaemia are recognised features 
D) Raised serum copper levels are evident at birth 
E) A sibling with biochemical evidence of the disease is treated only when he or she becomes symptomatic

Answer: B

47. Which one of the following conditions is most likely to be associated with pyoderma gangrenosum? 
A) Tuberculosis 
B) Leprosy 
C) Chronic myeloid leukaemia 
D) Sulphonamide therapy 
E) Cushing's syndrome

Answer: C

48. A retired 69-year-old man presents to the Emergency Department with a temperature of 39 °C. The white cell count (WCC) is 45 x 109/l with 90% neutrophils. Which one of the following findings would be most useful for differentiating chronic myeloid leukaemia (CML) from a leukaemoid reaction?
A) Philadelphia chromosome 
B) Splenic enlargement 
C) Low leucocyte alkaline phosphatase score 
D) Hypercellular bone marrow 
E) Elevated platelet count

Answer: A

49. The blood gas analysis in a 40-year-old woman who has presented with fatigue shows: pH 7.51, Pao2 11 kPa, Paco2 6 Pa, bicarbonate 32 mmol/l. Which one of the following is the most likely cause of this patient's clinical presentation? 
A) Spironolactone therapy 
B) Acetazolamide therapy 
C) Conn's syndrome 
D) Addison's disease 
E) Chronic diarrhoea

Answer: C

50. A 30-year-old woman presents to the Emergency Department at 9 pm with severe headache. Two hours earlier she had felt as if she was hit on the back of the head and she then experienced severe occipital headache and vomiting. A diagnosis of subarachnoid haemorrhage is suspected. Urgent computed tomography (CT) scan of the brain is normal. Which one of the following is the most appropriate next step in the management? 
A) Reassure and discharge after prescribing strong painkillers 
B) Observe in hospital and repeat the CT brain scan the next morning 
C) Perform a lumbar puncture and cerebrospinal fluid (CSF) analysis immediately 
D) Observe in hospital and delay lumbar puncture and CSF analysis until the next morning 
E) Arrange urgent magnetic resonance imaging (MRI) of the brain

Answer: D

51. A 43-year-old woman with rheumatoid arthritis visits for a scheduled follow-up. Plain X-ray of the hands reveals marginal erosions at the metacarpal heads. Why are these erosions marginal?
A) Random localization
B) Plain X-rays fail to show central erosions
C) Rotation of the film
D) Marginal joint area is devoid of overlying cartilage
E) Presence of sesamoid bones

Answer: D

52. A 33-year-old homosexual man presents with fever, cough and prostration. His sputum is positive for acid-fast bacilli. Which one of the following antituberculous medications is contraindicated in a patient with AIDS?
A) Isoniazid
B) Thiacetazone
C) Rifabutin
D) Ethionamide
E) Cycloserine

Answer: B

53. A 15-year-old male patient with ventricular septal defect presents to A&E in a dyspnoeic state. After careful examination the senior house officer (SHO) suggests the development of Eisenmenger’s syndrome. You reexamine the patient and find something that points away from the SHO’s diagnosis. What have you found?
A) Clubbing
B) Central cyanosis
C) Soft P2
D) Single S2
E) Graham–Steell murmur

Answer: C

54. A 65 year old female who has a history of longstanding psoriasis and heavy alcohol intake, presents with a severe exacerbation of psoriasis. She was admitted and received topical therapy and over the month of her admission, her gamma-GT concentration had fallen from 400 U/L to 150 U/L (4 - 35). Six weeks after discharge she was seen in outpatients where her psoriasis remained under control, but she complained of generalized hair loss. What is the most likely cause for her hair loss?
A ) Alopecia areata
B ) iron deficiency
C ) telogen effluvium
D ) thiamine deficiency
E ) trichotillomania

Answer: C

55. A 17 year-old girl presents with vomiting and her investigations show:
sodium 120 mmol/L (137 - 144)
potassium 3.0 mmol/L (3.5-4.9)
urea 2.2 mmol/L (2.5 - 7.5)
urine sodium 2 mmol/L
urine osmolality 700 mosmol/kg (350 - 1000)
What is the most likely diagnosis?
A ) Addison's disease
B ) bulimia nervosa
C ) diuretic abuse
D ) syndrome of inappropriate antidiuretic hormone secretion
E ) water intoxication

Answer: B

56. A 70-year-old female is diagnosed with anaplastic thyroid cancer. What is the most likely consequence of this cancer?
A ) Brain metastases
B ) Hypercalcaemia from bony metastases
C ) Liver metastases
D ) Lung metastases
E ) Upper airways obstruction

Answer: E

57. Three months following laparoscopic cholecystectomy, a 50-yearold woman presents with right upper abdominal quadrant pain, excessive flatulence and intolerance to fatty food. Abdominal ultrasound and ERCP fail to detect any residual stone or biliary stricture. What is the cause of this woman’s presentation?
A) Small common bile duct stones
B) Cholangiocarcinoma
C) Peri-ampullary adenocarcinoma
D) Post-cholecystectomy syndrome
E) Choledochal cyst

Answer: D

58. A 23-year-old woman has postpartum haemorrhage following
vaginal delivery of a macrosomic baby. Three hours after starting blood transfusion, she develops fever, breathlessness and diffuse pulmonary infiltrates. Pulmonary capillary wedge pressure is 8 mmHg. You start high-flow oxygen. What has developed in this woman?
A) Over-transfusion
B) Major ABO incompatibility
C) Pulmonary aspiration
D) Re-expansion pulmonary oedema
E) Transfusion-related acute lung injury

Answer: E

59. A 49-year-old man is referred to you for further evaluation of right iliac fossa mass. His abdominal ultrasound shows many small rounded liver masses in addition to the right iliac fossa mass. His 24-hour urinary 5-HIAA is prominently raised. Which one of the following could be part of this man’s illness?
A) Unilateral rhonchus
B) Episodic facial pallor
C) Giant JVP n-wave
D) Fingers telangiectasia
E) Episodic constipation

Answer: C

60. A 23-year-old recently married woman presents with dysuria and suprapubic discomfort. She is afebrile with no loin pain. Her past medical history is unremarkable. How would you treat her empirically?
A) Intravenous ciprofloxacin
B) Oral rifampicin
C) Oral co-amoxiclav
D) Intramuscular gentamicin
E) Oral vancomycin

Answer: C

61. A 60-year-old woman presented with a small right pupil, right ptosis and impaired sweating over the ipsilateral forehead. Sweating on the rest of the face was unaffected. Where is the most likely site of this lesion?
A) Cervical spinal cord
B) Common carotid artery
C) Hypothalamus
D) Internal carotid artery 
E) Lateral medulla

Answer: D

62. A 53-year-old man presented with hypertension of 150/110 mmHg. He is generally asymptomatic and has no previous medical history of note. He is a smoker of 5 cigarettes daily and drinks modest quantities of alcohol. He takes no prescribed medications. Examination reveals a BMI of 33.5 kg/cm2 but nil else. Investigations:
Serum sodium 146 mmol/l (NR 133-145); Serum potassium 3.2 mmol/l (NR 3.5 - 5); Urinary potassium excretion 42 mmol/l (NR less than 30)
What is the likely diagnosis?
A) adrenocortical adenoma
B) Bartter's syndrome
C) Liddle's syndrome
D) liquorice ingestion
E) pheochromocytoma

Answer: A

63. A previously healthy 54-year-old man presents to the emergency department complaining of chest pain. His ECG shows an acute inferior wall myocardial infarction. His blood pressure is 90/60 mm Hg. On physical examination, he has jugular vein distention and clear lungs. You should treat him immediately with which of the following:
A) Intravenous fluids.
B) Norepinephrine.
C) Dopamine.
D) Nesiritide.
E) Nitroprusside.

Answer: A

64. While caring for a patient with suspected pericardial tamponade, you would expect to find: 
A) Equalization of right and left ventricular pressures. 
B) Isolated systolic hypertension. 
C) A hyperdynamic precordium. 
D) Poor R wave progression on the 12-lead electrocardiogram. 
E) Mitral regurgitation.

Answer: A

65. A female patient aged 30 has a 5 years history of difficulty getting upstairs and out of a low chair and mild upper limb weakness but no pain. There is no family history. She presented with severe type 2 respiratory failure. EMG showed evidence of myopathy. 
The most likely diagnosis is:
A) Polymyositis
B) Inclusion body myositis
C) Acid maltase deficiency
D) Miller-Fisher Syndrome
E) Lambert-Eaton Myasthenic syndrome

Answer: C

66. A teenage girl presents with Guillain-Barre syndrome. Her weakness continues to worsen after admission to hospital. Which of the following should be used to monitor her? 
A) Arterial blood gases
B) Chest expansion size
C) FEV1/FVC ratio
D) PEFR
E) Vital capacity

Answer: E

67. A 19-year-old woman presents to the clinic having had 5 blackouts over the last year, all while she is standing up. She gets warnings of blurred vision, nausea, feeling hot. She had been witnessed twice to have jerking of all limbs while she is unconscious. The attacks last 30-60 seconds. She recovers quickly after the attacks. She has never bitten her tongue or sustained any injuries. Physical examination and an ECG are normal. Her grandmother and sister suffer from epilepsy. Which of the following investigations is the most appropriate?
A) EEG
B) 24 hour ECG recording
C) CT brain
D) ECHO
E) Tilt table test

Answer: E

68. A 21-year-old female presented with a sudden onset of left sided head and neck pain. 24 hours later she presents with sudden onset of right hemiparesis, facial weakness and homonymous hemianopia and left Horner's syndrome. A CT brain showed a left middle cerebral artery territory infarction. The most likely diagnosis is: 
A) Cardiac embolism
B) Migrane 
C) Left carotid artery dissection
D) Antiphospholipid syndrome
E) Systemic vasculitis

Answer: C

69. A female presents with headache, lethargy and weight loss. Which of the following would make the diagnosis of giant cell arteritis unlikely?
A) A normal ESR
B) Bilateral headache
C) Non-tender temporal arteries
D) Papilloedema on fundoscopy
E) The patient is 50 years old

Answer: D

70. A 60-year-old man presents with a 2 month history of progressive confusion, gait disturbance, and urinary incontinence. Examination reveals mild global cognitive dysfunction and gait ataxia. CT brain shows enlarged ventricles with no evidence of obstruction to CSF outflow. Lumbar puncture reveals normal CSF pressure and constituents. Which one of the following management steps is likely to be most helpful? 
A) CSF drainage via repeated lumbar puncture
B) EEG
C) Intracranial pressure monitoring
D) MRI brainstem
E) Serum B12 and folate levels

Answer: A

MULTIPLE CHOICE QUESTIONS-3

1. A 42-year-old woman has a12-year history of ulcerative colitis that has responded well to mesalamine and occasional corticosteroid enemas. Recent surveillance colonoscopy with biopsies showed low-grade dysplasia. Which of the following would be the most appropriate next step in the management of this lady?
A) No intervention, repeat colonoscopy in 6 months
B) No intervention, repeat colonoscopy in 1 year
C) Administer continuous corticosteroid enemas
D) Refer to the surgeon for colectomy
E) Administer a high-dose corticosteroid intravenously

Answer: D

2. A 28-year-old man undergoes an endoscopic transthoracic sympathectomy for palmar hyperhidrosis. Postoperatively, he has developed ptosis of his eye. This is due to iatrogenic damage to which one of the following nerves?
A) 8th cervical nerve
B) 1st thoracic nerve
C) Lower cord of brachial plexus
D) 2nd thoracic ganglion
E) 7th cervical nerve

Answer: B

3. A 51-year-old woman undergoes a successful bone marrow transplant from a matched unrelated donor for refractory Hodgkin’s disease. She is discharged from the hospital on no medications and is feeling well. At an appointment 6 months posttransplant, she is well with no evidence of malignancy. Three weeks later, she travels to Florida with her family. She is cautious of the sun but develops sunburn on her face, despite wearing sunscreen and a protective hat. When she returns from her trip 5 days later, she presents with persistent erythema of her face. She also states that her wrists and hands have been sore for the past 2–3 weeks. On examination, her face is mildly tender to touch, and a rash is present. Her hands are diffusely swollen. She is afebrile, and the remainder of her physical examination is benign. What is the most likely diagnosis?
A) Graft-versus-host disease
B) Hypersensitivity to sunlight due to the patient’s antirejection regimen
C) New-onset systemic lupus erythematosus
D) Rosacea
E) Staphylococcal skin infection acquired during travel

Answer: A

4. A 35-year-old woman arrives on the floor after an uneventful hysteroscopy to evaluate her long history of uterine fibroids. About 30 minutes after her arrival, she begins to complain of nausea and has two episodes of vomiting. The physician administers 0.625 mg of droperidol and 400 mg of acetaminophen by mouth. On follow-up evaluation, the patient's neck is involuntarily flexed to one side. She is alert, oriented, and conversant and has an otherwise normal neurologic examination. Which of the following is the most likely diagnosis?
A) Cerebral vascular accident 
B)Conversion disorder 
C) Dystonic reaction to droperidol
D) Munchausen syndrome 
E) Seizure

Answer: C

5. A 50-year-old woman with diabetes mellitus presents with backache and inability to walk unaided. This came on suddenly, while she was trying to lift her shopping bag from the car boot. Which of the following is most suggestive of a lesion of the sciatic nerve?
A) Absent knee tendon jerk
B) Foot drop
C) Inability to flex the hip
D) Decreased sensation on the anterior thigh and medial leg
E) Intervertebral disc prolapse at L2/L3 level

Answer: B

6. A 59-year-old man has a body mass index (BMI) of 42. As might be expected, his fasting blood glucose level is high, 210 mg/dL, as was his HbA1c level, 9.8%. In addition, he suffers from hypertension and dyslipidemia. His physician advised him to lose weight both for his general well-being and to help control his diabetes. Consequently, for the past 4 years, he has desperately tried to lose weight. He worked his way through an alphabet of popular and fad diets, from the Atkins diet to the Weight Watchers diet. If he lost a few pounds while on a particular diet, he gained back the pounds lost, plus a few more within a few months of terminating the diet. Finally, he and his physician decided he should try bariatric surgery, the Roux-en-Y gastric bypass procedure. A week after leaving the hospital, his fasting blood glucose level was 100 mg/dL. Three months later, it was 96 mg/dL and his HbA1c was 6.0%. Which of the following choices is most likely to explain this remarkable improvement is his diabetes?
A) The reduction in level of his circulating glucagonlike peptide-1 (GLP-1)
B) The reduction in level of his circulating peptide YY (PYY)
C) The reduction in his mass of adipose tissue
D) The reduction in the level of his circulating ghrelin
E) The reduction in the level of his circulating leptin

Answer: D

7. A 51-year-old man has recurrent bouts of lightheadedness and mild confusion. Episodes of hypoglycemia were recently documented that are improved with ingestion of food. The patient has no headache, blurred vision, or double vision. He has gained approximately 4.5 kg (10 lb) in the past 2 months. Medical and family histories are noncontributory. Physical examination is normal. The following laboratory data are obtained after an overnight fast: plasma glucose 30 mg/dL (1.67 mmol/L), serum insulin 30 mlU/L (215.25 pmol/L), and an elevated serum C-peptide level. Screening for sulfonylurea is negative and CT scan of the abdomen is normal. 
Which of the following diagnostic studies should be done next?
A) Endoscopic retrograde pancreatography.
B) Transabdominal (Endoscopic) ultrasonography.
C) MRI of the abdomen. 
D) Positron emission tomography.
E) Somatostatin receptor scintigraphy.

Answer: B

8. A 23-year-old married woman comes to the office after recent exposure to a person with active hepatitis A. She has a long history of recurrent sinopulmonary infections and bronchial asthma. In addition, after her last pregnancy, she received a blood transfusion for severe postpartum hemorrhage. After receiving an intramuscular dose of immune serum globulin as prophylaxis against hepatitis A, she develops an anaphylactic reaction. Which of the following is the most likely cause of this patient’s reaction?
A) Immunoglobulin A (IgA) deficiency with anti-IgA antibodies
B) A hemolytic transfusion reaction
C) Contaminated immune serum globulin
D) A type IV hypersensitivity reaction against a protein in the immune serum globulin
E) A febrile reaction

Answer: A

9. A 47-year-old man recently consulted a physician about developing weakness, particularly in his right hand. Upon providing a history, the man explained that he does house repair and has been working on a neighborhood rehabilitation project for the past several months. In doing this, he sandblasts and sands and scrapes by hand to remove the old paint. These
homes were first constructed in the 1920s and since have been covered with several layers of paint. He also revealed that he habitually ate his lunch at the work site, which he described as being dusted with old paint particles. In addition to the weakness in his arm, he admitted to sporadic stomachaches, constipation, and said his wife had complained that he is always irritable. He also states that, until recently, he had been in good health. Upon examination, he was found to be 6 feet (19.7 m) tall and to weigh 170 lb (77.1 kg). His heart, lungs, and abdomen were normal, as were most analytical values, but he did show signs of right wristdrop consistent with radial nerve palsy and his complete blood count (CBC) showed a microcytic anemia; his serum iron levels were found to be normal. Which of the following diagnostic tests would provide the most useful information regarding the appropriate treatment?
A) Nerve conduction velocity (NCV) study of the right arm
B) Radiography of the right arm and wrist
C) Magnetic resonance imaging (MRI) scans of the right arm and wrist
D) Urine screen for heavy metals (lead, mercury, arsenic)
E) Screening for diabetes mellitus.

Answer: D

10. Which of the following clinical findings is least likely in a patient with infarction in the distribution of the left anterior choroidal artery? 
A) Right hemiparesis
B) Right hemi-sensory loss
C) Right hemianopia
D) aphasia.

Answer: D

11. A 76-year-old man presenting with dysphagia was found to have inoperable esophageal adenocarcinoma. He has an endoscopically placed esophageal stent for palliation of his symptoms, but unfortunately found it very painful, and it was removed a few days later. There is no perforation. He asks whether there are any other treatment options to help with his symptoms. Which of the following modalities is an appropriate first-line treatment option to discuss?
A) Band ligation
B) Botulinum toxin injection
C) Brachytherapy
D) Local ethanol injection
E) Photodynamic therapy.

Answer: C

12. An AIDS patient develops symptoms suggestive of a severe, persistent pneumonia with cough, fever, chills, chest pain, weakness, and weight loss. The patient does not respond to penicillin therapy, but goes on to develop very severe headaches. The presence of focal neurologic abnormalities leads the clinician to order a CT scan of the head. This demonstrates several metastatic brain abscesses. Biopsy of one of these lesions demonstrates beaded, branching, filamentous gram-positive bacteria that are weakly acid fast. Which of the following is the most likely causative organism? 
A) Actinomyces
B) Aspergillus
C) Burkholderia
D) Francisella
E) Nocardia.

Answer: E

13. You are a hospitalist called to admit a 70-year-old man to the ICU. His wife states that he was sitting at the table eating breakfast with her when he dropped his fork and had difficulty speaking. Within a couple of minutes he was unable to move his right side. She called the paramedics, who brought him to the hospital. Now in the ICU, his vital signs are as follows: BP 200/98 mm Hg, HR 100, RR 10, O2 saturation 94% on room air. He is afebrile. On physical examination he is lethargic and unable to speak. His pupils are equal and round but sluggish. He has flaccid paralysis of the right arm and leg with a Babinski sign present on the right. His heart is irregularly irregular, and an ECG confirms atrial fibrillation. A CT of the head shows a large bleed in the left frontoparietal area with mass effect and midline shift. You decide to intubate the patient to protect his airway. What is the next most appropriate step in the treatment of this patient while you are awaiting your urgent neurosurgical consult?
A) Hyperventilate the patient to a goal pCO2 of 20 mm Hg
B) Give a bolus of IV mannitol
C) Give a bolus of IV dexamethasone
D) Give sublingual nifedipine to decrease the BP
E) Anticoagulate with IV heparin because of the atrial fibrillation.

Answer: B

14. A 73-year-old woman is brought in by paramedics after fainting in the mall and hitting her face. She does not remember any preceding symptoms, and she did not lose control of her bowel or bladder. Witnesses at the scene say that she was down for less than a minute, then woke up and was fairly alert. She was bleeding from a laceration on her chin and paramedics were called. When she arrived at the hospital, her initial laboratory
values were normal and an EEG did not show epileptiform activity. She is placed on a cardiac monitor. The following day, she becomes lightheaded and loses consciousness while lying in bed, and her monitor shows tachycardia with the QRS complexes being uniformly longer than 120 ms.
What is the most common cause of this rhythm disturbance?
A) Uncontrolled hypertension
B) Distention of the pulmonary veins
C) Accessory pathway
D) Ischemic heart disease.

Answer: D

15. A 47-year-old woman presents at night to the Emergency Department with chest pain. She states that the pain started that evening and has progressively been getting worse. She is concerned that she is having a heart attack. The pain is described as a burning sensation associated with a sour taste in her mouth, and it started shortly after she ate dinner; it has occurred on previous occasions, but never as bad as it is now.
Previously, she used calcium carbonate tablets that were effective for the pain. She has no history of heart disease or other medical problems, and she takes no regular medications. She does not smoke cigarettes or use cocaine. Her vitals are normal, and her physical examination is unremarkable. Initial laboratory tests and an ECG are normal. Which of the following is the most likely cause of this patient’s chest pain?
A) Unstable angina
B) Myocarditis
C) Pulmonary embolism
D) Gastroesophageal reflux disease
E) Costochondritis.

Answer: D

16. A 28-year-old patient with end-stage renal disease (ESRD) on continuous ambulatory peritoneal dialysis (CAPD) for two months presents with fever, abdominal pain and cloudy dialysis fluid. There is no diarrhea or vomiting and the pain has been present for about 12 hours. The patient has ESRD secondary to chronic glomerulonephritis, there is no history of diabetes, urinary infections or antibiotic use. Examination reveals a temperature of 38.9 C (102 F), and blood pressure of 110/70 mm Hg. The throat is clear, as are the lungs. Cardiac examination reveals a grade 2/6 systolic murmur. Abdominal examination reveals decreased bowel sounds with diffuse tenderness. There is mild rebound. There is no edema or skin rash. A complete blood count shows a leukocyte count of 14,200/mm3, hemoglobin is 12.5 g/dL. Peritoneal fluid is cloudy with 1,000 white blood cells, 85% of which are polymorphonuclear leukocytes. Gram's stain of fluid is negative. Cultures of blood and peritoneal dialysis fluid are taken. Which of the following is the most appropriate initial step in management? 
A) Fluconazole
B) Immediate removal of dialysis catheter.
C) Intravenous vancomycin
D) Intravenous gentamicin 
E) Oral ciprofloxacin

Answer: C

17.A 59-year-old lady is admitted with a 30 minute history of heavy central chest pain associated with nausea and sweating. Her ECG shows ST elevation in leads V1, V2, V3 and V4.
Which of the following coronary arteries is most likely to be occluded?
A) Circumflex artery
B) Left anterior descending artery
C) Obtuse marginal artery
D) Posterior descending artery
E) Right coronary artery

Answer: B

18. A family physician cares for a family consisting of a 45-year old husband, 43-year-old wife and a 12-year-old daughter. The family reports that recently the 77-year-old maternal grandmother who lived with them died after a prolonged respiratory infection. Autopsy subsequently confirms that she had active pulmonary tuberculosis at the time of death. The organism tested sensitive to all anti-tuberculosis drugs. In responding to the grandmother's illness, which of the following is the most appropriate step in management? 
A) Obtain leukocyte counts on all family members
B)Obtain sputum cultures for acid fast bacilli 
C)Obtain chest computerized tomograms on all members 
D)Place protein purified derivative (PPD) test on all members
E)Schedule bronchoscopy lavage for the adults.

Answer: D

19. Which of the following extraintestinal manifestations is associated with Crohn disease but not ulcerative colitis? 
A) Ankylosing spondylitis 
B) Erythema nodosum 
C) Nephrolithiasis 
D) Thromboembolic disease 
E) Uveitis

Answer: C

20. A 30 year old woman presents with unprovoked left popliteal deep vein thrombosis. Her family history is negative for venous thromboembolism. She has a history of SLE and takes prednisone 10mg daily, hydroxychloroquine and supplements of calcium and vitamin D. She has one child but has had two miscarriages. An aPTT test performed before starting anticoagulation in this patient is prolonged, but her PT is normal.
Which statement is most likely to be correct?
A) Because her APTT is increased already, Heparin should be withheld and warfarin treatment initiated at INR 2.5 to 3.5, perhaps forever 
B) The patient should be encouraged to use oral contraceptive to prevent pregnancy during the time she is anticoagulated
C) She will need Heparin and then anticoagulation with Warfarin at INR 2 to 3, perhaps indefinitely
D) Start anti-coagulation with Heparin and then Warfarin at INR 2.5 to 3.5 for 3 to 6 months
E) She will need lifelong new oral anticoagulants.

Answer: C

21. A 65-year-old man has been stable on the general medical ward following an admission with acute coronary syndrome several days previously. His drug history consists of aspirin, enalapril and glyceryl trinitrate (GTN) spray. He has developed dyspnea over the last few hours. On examination he has a raised jugular venous pressure (JVP) and crackles to his mid zones. His electrocardiogram (ECG) shows a rate of 140 beats per minute in atrial fibrillation. Which of the following is the most appropriate management?
A) Intravenous amiodarone
B) Intravenous digoxin
C) Intravenous flecainide
D) Observe and screen for MI 
E) Synchronized DC cardioversion.

Answer: E

22. A 48-year-old man is admitted through the ER with a possible left lower extremity cellulitis. The patient says that he has been having fever, swelling, and redness of his left lower extremities that comes and goes spontaneously over the past couple of months. He also tells you about an episode in which he lost vision in his left eye for several minutes a couple of weeks ago, but the vision returned without incident. He had a urologic evaluation for penile trauma 3 months ago. On physical examination, his BP is 125/80, HR 70, RR 14, and he is currently afebrile. His examination is significant for a 3/6 systolic murmur heard at the left lower sternal border without radiation while lying supine. Blood cultures return positive results for Enterococcus species, and an echocardiogram reveals large mitral vegetation. You review treatment for enterococcal endocarditis. Which of the following antibiotics always misses enterococcal infections?
A) Ampicillin-sulbactam
B) Nitrofurantoin
C) Cefipime
D) Vancomycin
E) Linezolid.

Answer: C

23. A 70-year-old woman with a history of type2 DM and CAD with a known left bundle branch block on ECG is admitted to the ICU with sepsis from a urinary source. She is fluid resuscitated, and empiric broad-spectrum antibiotics are begun after the appropriate cultures are obtained. Despite what appears to be adequate resuscitative efforts with volume replacement, the patient has had minimal urine output over the past couple of hours. You decide to place a pulmonary artery catheter to help to determine the patient’s hemodynamic situation. Which of the following complications of pulmonary artery catheter placement is the patient at increased risk for because of her past medical history?
A) Pulmonary artery perforation
B) Pulmonary infarction
C) Complete heart block
D) Pneumothorax
E) Ventricular tachycardia.

Answer: C

24. The first four tests that should be ordered in the initial evaluation of patients with a suspected coagulopathy are all of the following except
A) Platelets count
B) Bleeding time
C) Platelet aggregation studies
D) PT
E) aPTT

Answer: C

25. Which of the following is not true about tetanus?
A) The toxin affects inhibitory GABA and glycine receptors, leading to unopposed contraction and spasm of skeletal muscle.
B) It is characterized by acute onset of skeletal muscle rigidity and convulsive spasm. 
C) Initial symptoms involve lockjaw and risus sardonicus.
D) Fractures, dislocations, and rhabdomyolysis may occur due to forceful sustained muscle 
E) Tetanus disease usually leads to long-lasting immunity.

Answer: E

26. A 55-year-old woman, who has never smoked, presents to you on the ward with a history of weight loss, decreased appetite and finger clubbing. You are told that her chest x-ray revealed opacity in the hilar region of the right lung suggesting a bronchogenic carcinoma. She is currently awaiting a CT-chest with bronchoscopy to follow. From the list below, select the most likely diagnosis: 
A) Squamous cell carcinoma of the lung 
B) Adenocarcinoma of the lung 
C) Small cell carcinoma of the lung 
D) Large cell carcinoma of lung 
E) Carcinoid tumour of the lung.

Answer: B

27. A 68-year-old woman recently diagnosed with multiple myeloma presents to her GP with progressively increasing breathlessness, exercise intolerance and ankle swelling. On examination, there is bilateral pitting leg edema to her thighs, ascites and raised JVP. The apical impulse is impalpable. An ECG shows diffusely diminished voltage. Chest X-ray is normal and the echocardiogram shows small thick ventricles and dilated atria with a thickened interatrial septum. The ventricular myocardium has a granular sparkling texture on echo, and minimal fluid in the pericardial space is noted. What is the most likely diagnosis? 
A) Chronic pericardial effusion with tamponade 
B) Chronic pericardial effusion without tamponade 
C) Constrictive pericarditis 
D) Restrictive cardiomyopathy 
E) Congestive heart failure.

Answer: D

28.A 65-year-old male patient is admitted with renal failure and is diagnosed with acute tubular necrosis. Which of the following is least likely to be the cause of acute tubular necrosis?
A) Rhabdomyolysis
B) Paracetamol poisoning
C) Hypovolemia
D) Hypertension
E) Corticosteroid therapy

Answer: E

29. A 65-year-old woman with long-standing diabetes mellitus visits your clinic for follow-up 2 weeks after the initial visit. Her fasting home glucose monitoring shows elevated blood sugar levels ranging between 200-250 mg/dl. Two weeks ago, her HbA1c was 7.2% and the fasting plasma glucose was 212 mg/dl. You recommend the following to evaluate the discrepancy between the fasting values and the HbA1c:
A) Order a CBC 
B) Prescribe a new glucometer
C) Order a fructosamine
D) Repeat the HbA1c
E) Repeat the fasting plasma glucose.

Answer: A

30. A 43-year-old man presents to the clinic with complaints of fever, night sweats, anorexia, cough, and chest pain. The chest x-ray reveals infiltrates in both the lower and upper lobes, with possible cavitations in the apices. A presumptive diagnosis of tuberculosis is made on the basis of finding acid-fast bacilli (AFB) on microscopic examination of sputum. The patient is started initially on isoniazid, rifampin, pyrazinamide, and ethambutol. 
What is the best way to monitor this patient?
A) Sputum acid-fast stains every month for 6 months
B) Sputum cultures every month until cultures become negative
C) Serial chest x-rays
D) Blood testing for drug toxicity
E) Observe for clinical deterioration.

Answer: B

31. A 49-year-old man with untreated hepatitis C infection develops persistent proteinuria. Which of the following diagnoses is not typically associated with hepatitis C?
A) Membranous nephropathy
B) Focal segmental glomerulosclerosis
C) Membranoproliferative glomerulonephritis
D) Cryoglobulinemia
E) Polyarteritis nodosa

Answer: B

32. A 59-year-old woman is admitted with central abdominal pain. Serum amylase is 1800 IU/L. Her initial Glasgow Coma Scale score is 4. You are asked to review her the next day as the nurses have noticed that her urine output has been just 15 mL in the past 3 hours. The rest of her observations are as follows:
• Blood pressure = 105/45 mmHg
• Pulse = 113 beats/min
• Respiratory rate = 28 breaths/min
• Saturation 93% on 8 L of oxygen
On auscultation of her chest you hear widespread crepitations. What is the most appropriate next course of action?
A) Fluid restriction
B) Colloid bolus
C) Furosemide
D) Transthoracic echocardiogram
E) Central line insertion

Answer: E

33. Features of type 1 renal tubular acidosis include all of the following EXCEPT:
A) Normal AG acidosis
B) Hypokalemia
C) Ca phosphate stone formation
D) Urine pH decrease below 4, following oral ammonium chloride loading test
E) Raised serum PTH hormone

Answer: D

35. All of the following statements regarding non-alcoholic steatohepatitis (NASH) are correct EXCEPT:
A) Liver biopsy is the only means of accurately diagnosing NASH.
B) Weight loss has been shown to result in improvement of elevated liver enzymes.
C) Patients with NASH can develop liver cirrhosis.
D) Histologically it is characterized by severe portal or peri-portal inflammation.
E) Laboratory studies cannot identify patients at risk of progression.

Answer: D

36. A 30-year-old male patient complains of fever and sore throat for several days. T he patient presents to you today with additional complaints of hoarseness, difficulty breathing, and drooling. On examination, the patient is febrile and has inspiratory stridor. Which of the following is the best course of action?
A) Begin outpatient treatment with ampicillin.
B) Culture throat for β-hemolytic streptococci.
C) Admit to intensive care unit and obtain otolaryngology consultation.
D) Schedule for chest x-ray.
E) Obtain Epstein-Barr serology.

Answer: C

37. Regarding management of an episode of hypoglycaemia in a 65-year-old who is on sulfonylurea therapy, which ONE of the following is TRUE?
A) Initial oral or intravenous glucose therapy can be omitted because it is likely to fail
B) Octreotide should be considered for recurrent or persistent hypoglycaemia
C) Hypoglycaemia in a stable diabetic on a regular sulfonylurea dose is not usually due to a precipitating event
D) There is a more sustained response to intravenous glucose therapy in sulfonylurea-induced hypoglycaemia than in insulin-induced hypoglycaemia

Answer: B

38. A 59-year-old man presents to the ED with left-sided chest pain and shortness of breath that began 2 hours prior to arrival. He states the pain is pressure-like and radiates down his left arm. He is diaphoretic. His BP is 160/80 mm Hg, HR 86 beats per minute, and RR 15 breaths per minute. ECG reveals 2-mm ST-segment elevation in leads I, aVL, V3 to V6. Which of the following is an absolute contraindication to receiving thrombolytic therapy?
A) Systolic BP greater than 180 mm Hg
B) Patient on Coumadin and aspirin
C) Total hip replacement 3 months ago
D) Peptic ulcer disease
E) Previous hemorrhagic stroke

Answer: E

39. The best pharmacologic treatment for hypotension related to the spinal anesthetic is
A) atropine
B) epinephrine
C) phenylephrine
D) calcium
E) labetalol

Answer: C

40. An ischemic stroke involving the right side of the pons could lead to which of the following patterns of weakness?
A) Left facial weakness and right body weakness
B) Right facial weakness and left body weakness
C) Right facial weakness and right body weakness
D) Left arm weakness and right leg weakness
E) Right arm weakness and left leg weakness

Answer: B

41A 49-year-old man who smokes two packs of cigarettes a day presents with a lung mass on x-ray and recent weight gain. Laboratory examination shows hyponatremia with hyperosmolar urine. The patient probably has
A) Renal failure
B) Pituitary failure
C) Conn’s syndrome
D) Cardiac failure
E) Inappropriate ADH

Answer: E

42. A 75-year-old woman is referred for assessment of her mental state. Her carer states that she noticed general decline in her physical and mental state. She is more forgetful and her concentration is poor. Sometimes her speech is incoherent. She trips frequently and had many falls. On rare occasions she has imagined seeing people sitting in her room. Each of the following features would help to distinguish between organic and functional disorders, except
A) Nystagmus 
B) Visual hallucinations 
C) Poor performance of the serial sevens test 
D) Dysphasia 
E) Hemianopia

Answer: C

43A 40-year-old man with occasional dysphagia and who otherwise feels well undergoes esophageal motility studies that show an LES amplitude of approximately 60 mmHg. The esophagus relaxes completely when he swallows. The most likely diagnosis is
A) GERD (gastroesophageal reflux disease)
B) Achalasia
C) Hypertensive LES
D) Barrett’s esophagus
E) Esophageal spasm

Answer: C

44A 60-year-old accountant complains of recurrent attacks of exquisite pain and swelling in the left big toe. Each of the following conditions is likely to be associated with this disorder, except 
A) Chronic alcoholism
B) Obesity
C) Rheumatoid arthritis 
D) Diabetes mellitus 
E) Diuretic therapy

Answer: C

45A 60-year-old man was referred for further assessment of a cardiac murmur. On examination it was noted that he has a weak and slow-rising carotid pulse. The most likely underlying cardiac abnormality is
A) Aortic regurgitation 
B) Alcoholic cardiomyopathy
C) Dissecting aneurysm
D) Hypertrophic obstructive cardiomyopathy (HOCM)
E) Aortic stenosis

Answer: E

46A 50-year-old woman has a history of gastrinoma and pituitary tumour. She reports increasing lethargy, drowsiness and constipation. The laboratory studies include raised calcium (2.9 mmol/I), low phosphate and raised PTH levels. The most likely diagnosis is? 
A) Carcinoma of the bronchus 
B) Pseudohypoparathyroidism 
C) Acromegaly 
D) Sarcoidosis 
E) Multiple endocrine neoplasia syndrome (MEN I)

Answer: E

47A 60-year-old teacher is being investigated for increasing shortness of breath and diffuse fibrotic changes found on plain chest X-rays. He was taking diuretics and anti-arrhythmic treatment. The 12-lead ECG shows a prolonged QT interval. Each of the following clinical situations could be responsible for this ECG abnormality, except
A) Heart failure 
B) Hypokalaemia 
C) Furosemide therapy 
D) Hypercalcaemia 
E) Amiodarone therapy

Answer: D

48A 30-year-old mechanic presented with central chest pain, worse on lying flat. He claims that he has had a flu-like illness for a week. Which one of the following ECG changes is most characteristic of this disorder? 
A) PR prolongation 
B) ST depression 
C) Peaked, tall T wave 
D) Prominent U wave 
E) PR-segment depression

Answer: E

49A 16-year-old male treated for acute lymphocytic leukemia develops severe lumbar and abdominal pain. His serum amylase is markedly elevated. Which of the following agents most likely caused these findings?
A) 6-MP
B) Asparaginase
C) Doxorubicin
D) Methotrexate
E) Vincristine

Answer: B

50A 58-year-old man was admitted with a three-weeks history of shortness of breath. The chest X-ray demonstrates a right pleural effusion. Pleural fluid analysis shows 6600 mm3 WBC,40% eosinophils. The condition least likely to be responsible this clinical presentation is 
A) Pneumothorax 
B) Haemothorax 
C) Pulmonary infarction 
D) Benign asbestos pleural effusion 
E) Tuberculous pleural effusion

Answer: E

51A 33-year-old woman presents for evaluation of chronic hepatitis C virus (HCV) infection discovered during a life insurance physical examination. Her risk factor for HCV infection was a blood transfusion after a motor vehicle accident at the age of nine years. Her husband of seven years tests negative for anti-HCV antibody. The couple wishes to conceive a child. Which of the following recommendations is most appropriate?
A) Avoid breast-feeding the baby.
B) Plan delivery of the baby by cesarean section.
C) Administer HCV immunoglobulin to the infant at the time of delivery.
D) Test the infant for HCV after the age of 18 months.
E) Use latex condoms.

Answer: D

52A general practitioner is in a dilemma over a case he has referred to you. He says that the referred 31-year-old man has no myocardial infarction or ischaemic heart disease, but his 12-lead-ECG shows many Q waves. Which one of the following is not responsible for this man’s ECG Q wave?
A) Hyperkalaemia
B) Hypertrophic cardiomyopathy
C) Wolff–Parkinson–White syndrome
D) Limb lead reversal
E) True posterior wall myocardial infarction

Answer: E

53A 55-year-old man presents with gross haematuria. He has recurrent urinary bladder stones. Rapid bedside ultrasound examination fails to demonstrate any stone, but there is a fungating bladder mass. What type of bladder cancer is likely?
A) Transitional cell cancer
B) Clear cell cancer
C) Squamous cell cancer
D) Leiomyosarcoma
E) Bladder lymphoma

Answer: C

54A 45-year-old woman is found to have gallstones on abdominal ultrasonography that forms part of a pre-medical insurance examination. She is reasonably well and healthy and denies abdominal symptoms. She neither smokes nor drinks alcohol and there is no family history of note. Which of the following statements is true regarding this woman’s gallbladder disease?
A) Emergency open cholecystectomy is required
B) Prophylactic laparoscopic gallbladder removal should be considered
C) Advise against cholecystectomy
D) Evaluate for surgery fitness
E) Refer to ERCP

Answer: C

55A 22-year-old man develops a severe attack of acute intermittent porphyria, two days after starting a medication prescribed by his GP. Which one of the following medications has not been prescribed by his GP?
A) Atenolol
B) Tolbutamide
C) Theophylline
D) Valproic acid
E) Dapsone

Answer: A

56A 48-year-old man with acute leukaemia develops fever, chills, headache, flushing, tachycardia and chest tightness one hour after starting blood transfusion because of anaemia. He reports ‘similar things’ whenever he receives blood transfusion. What is the best action now?
A) Continue the transfusion
B) Call the resuscitation team
C) Stop the transfusion
D) Give paracetamol
E) Epinephrine (adrenaline) injection

Answer: C

57. A 59-year-old man with long-standing Type 2 diabetes presents for his annual check-up. Fundoscopy reveals scattered dot haemorrhages and a few hard exudates in the macular area. He states that his vision is not that good but he is coping with it and that his eye glasses are not helpful. Which one of the following is true?
A) He has proliferative retinopathy
B) He has pre-proliferative retinopathy
C) He should be referred to the ophthalmologist
D) He should be referred for pars plana vitrectomy
E) Reassure and ask him to return next year

Answer: C

58A 50-year-old man with septicaemia develops severe shortness of breath, cough and irritability. Which one of the following is inconsistent with the diagnosis of adult respiratory distress syndrome?
A) PaO2/FiO2 150 mmHg
B) Reduced lung compliance
C) Bilateral interstitial infiltrate
D) Bilateral alveolar infiltrate
E) Pulmonary artery capillary tracing of 25 mmHg

Answer: E

59A 60 year old male is brought to casualty in the early hours of the morning after being found unconscious in the street. On examination, he was drowsy but localised to painful stimuli. There was no evidence of head injury or meningism. Investigations revealed:
sodium 134 mmol/L (137-144)
potassium 4.0 mmol/L (3.5-4.9)
urea 4.0 mmol/L (2.5-7.5)
creatinine 80 micromol/L(60-110)
glucose 4.5 mmol/L (3.0-6.0)
chloride 100 mmol/L (95 - 107)
bicarbonate 25 mmol/L (20 - 28)
plasma osmolality 385 mosmol/Kg (278 - 305)
What is the most likely explanation for his presentation
A ) diazepam poisoning
B ) ethanol poisoning
C ) methanol poisoning
D ) phenobarbitone poisoning
E ) Phenytoin poisoning

Answer: C

60A 46-year-old male was seen for an insurance medical examination. He was entirely asymptomatic, but his serum urate concentration was noted to be 0.5 mmol/L (0.23 - 0.46). What is the most appropriate management for this patient?
A ) Allopurinol
B ) Colchicine
C ) Ibuprofen
D ) Lifestyle intervention
E ) Sulphinpyrazone

Answer: D

MULTIPLE CHOICE QUESTIONS-4

1. A 92-year-old man is referred from his nursing home for evaluation of lethargy. Examination is unrevealing, but laboratory results are significant for a serum sodium level of 118 meq/L (normal, 135 148). Serum osmolality is 260, urine osmolality is 450, and urine sodium is 80. Which of the following is the most likely cause of this patient's lethargy?
A) hyperglycemia
B) hyperlipidemia
C) hyperproteinemia
D) SIADH
E) diabetes insipidus

Answer: D

2. Which one of the following is a recognized feature of polymyalgia rheumatica?
A) Weakness of distal muscle groups
B) Elevated serum creatine phosphokinase activity
C) An association with bronchial carcinoma
D) Weight loss
E) A peak incidence in the fourth decade of life

Answer: D

3. A 17-year-old girl notes an enlarging lump in her neck. On examination, her thyroid gland is twice the normal size, firm to rubbery, multilobular, nontender, and freely mobile. There is no adenopathy. Family history is positive for both hypo- and hyperthyroidism. Her serum triiodothyronine (T3) and thyroxine (T4) levels are low normal, and serum thyroid-stimulating hormone (TSH) is high normal. Technetium scan shows nonuniform uptake. Serum and antithyroglobulin titer is strongly
positive. What will thyroid biopsy of this patient most likely disclose?
A) giant cell granulomas and necrosis
B) polymorphonuclear cells and bacteria
C) diffuse fibrous replacement
D) lymphocytic infiltration
E) parafollicular cells

Answer: D

4. A 60-year-old man presents with a nonproductive cough for a week and generalized malaise. He also has noted some abdominal pain associated with diarrhea for the past few days. His temperature is 101.5°F and c linical examination is unremarkable. ACXR shows a left lower lobe infiltrate. His urinalysis shows 50 RBCs, and his BUN (30) and creatinine (1.6) are both mildly elevated. In light of the extrapulmonary symptoms and signs, which of the following is the most likely cause of his pneumonia?
A) Pseudomonas aeruginosa
B) S. aureus
C) H. influenzae
D) S. pneumoniae
E) Legionella

Aswer: E

5. A 78-year-old woman comes to your primary care office practice with her son who is concerned about changes in her mood. She is less interested in going out to dinner and does not want to visit family or friends. Her language skills seem to have deteriorated over the last few years and her memory is not as sharp. Her gait and motor strength are normal. Which of the following is the most likely diagnosis?
A) Parkinson's disease
B) anxiety disorder
C) meningioma
D) Alzheimer's disease
E) dysthymia

Answer: D

6. A 30-year-old woman who has been human immunodeficiency virus (HIV) positive for 4 years was recently diagnosed with AIDS. Which of the following immunologic abnormalities would be expected?
A) increased numbers of CD4+ (helper) T cells
B) decreased number of CD8+ (suppressor) T cells
C) cutaneous anergy to usual skin test antigens
D) normal B-cell function
E) increased natural killer cell function

Answer: C

7. A 60-year-old previously healthy man presents with massive rectal bleeding. Which of the following is the most likely diagnosis?
A) diverticulosis of the colon
B) ulcerative colitis
C) external hemorrhoid
D) ischemic colitis
E) carcinoma of the colon

Answer: A

8. A 59-year-old woman who lives independently and had been healthy, presents to the emergency department with cough and fever. She related she was well until 2 days before when she noted onset of fever, chills, and cough productive of yellow sputum. On examination, you note a tired appearing woman with BP of 160/90, pulse of 105, and respiratory rate of 32. You start her on ceftriaxone and azithromycin and admit her to the hospital. Which of the following factors is a poor prognostic sign in community acquired pneumonia?
A) age less than 60
B) systolic BP = 160 mmHg
C) leukocytosis = 15,000
D) respiratory rate = 32
E) mycoplasma pneumonia infected

Answer: D

9. You see a 19-year-old Caucasian man in your clinic who presents with a history of transient jaundice. On direct questioning, you ascertain that the jaundice is noticeable after periods of increased physical activity and subsides after a few days. The patient has no other symptoms and physical examination is unremarkable. Full blood count is normal (with a normal reticulocyte count) and liver function tests reveal a bilirubin of 37 μmol/L. The most appropriate management is:
A) Reassure and discharge
B) Start on a course of oral steroids
C) Request abdominal ultrasound
D) Request MRCP
E) Refer to Haematology

Answer: A

10. You see a 56-year-old man who was admitted for an elective upper GI endoscopy due to longstanding GORD which has failed to improve on antacids and PPIs. Your registrar suspects that this patient may have Barrett’s oesophagus and asks you to define what this is. The most appropriate description of Barrett’s oesophagus is:
A) Metaplasia of the squamous epithelium of the lower third of the oesophagus to columnar epithelium
B) Metaplasia of the columnar epithelium of the upper third of the oesophagus to squamous epithelium
C) Metaplasia of the columnar epithelium of the lower third of the oesophagus to squamous epithelium
D) Metaplasia of the squamous epithelium of the upper third of the oesophagus to columnar epithelium
E) Metaplasia of the squamous epithelium of the middle third of the oesophagus to columnar epithelium

Answer: A

11. You see a 48-year-old lorry driver, who presents to you with a three-month history of heartburn after meals which has not been settling with antacids and PPIs. You suspect that the patient has a hiatus hernia. The most appropriate investigation for diagnosing a hiatus hernia is:
A) Computer tomography (CT) scan
B) Chest x-ray
C) Upper GI endoscopy
D) Barium meal
E) Ultrasound

Answer: D

12. You see a 47-year-old man in clinic with a three-month history of epigastric dull abdominal pain. He states that the pain is worse in the mornings and is relieved after meals. On direct questioning, there is no history of weight loss and the patient’s bowel habits are normal. On examination, his abdomen is soft and
experiences moderate discomfort on palpation of the epigastric region. The most likely diagnosis is:
A) Gastric ulcer
B) Gastro-oesophageal reflux disease (GORD)
C) Duodenal ulcer
D) Gastric carcinoma
E) Gastritis

Answer: C

13. A 60-year-old farmer presented with non specific symptoms of chronic cough and progressive dyspnea for the last six years. The patient had interstitial pneumonitis. Chest x-ray revealed reticulonodular infiltrate associated with honey-combing of the lung. Pulmonary function studies showed a restrictive pattern with loss of lung volumes, impaired diffusion capacity, decreased compliance and exercise induced hypoxia. This farmer’s symptoms are most likely secondary to the following environmental hazard: 
A) The farmers cheese processing plant
B) The presence of spores of bacillus anthracis in the farmer’s farm
C) The harvesting of crops in rainy weather
D) The presence of asbestos in the water tanks.
E) The presence of antigenic detergents

Answer: C

14. Which of the following antiarrhythmic agents may promote AF?
A) Adenosine
B) Quinidine
C) Propafenone
D) Amiodarone
E) Atenolol

Answer: A

15. Torsades de pointes is characterized by all of the following except:
A) Results from triggered activity (early afterdepolarizations) that occurs during
phase 2 or 3 of the cardiac action potential
B) Prolonged QT interval
C) Exacerbation by bradycardia with short-long coupling intervals
D) Polymorphic VT
E) Often provoked during amiodarone administration

Answer: E

16. A 68-year-old patient is seen for a general examination. Current
recommendations for immunizations include
A) Tetanus booster every 5 years
B) Influenza vaccination yearly
C) Pneumococcal vaccination yearly
D) Hepatitis booster every 5 years
E) Meningococcal vaccination

Answer: B

17. Which of the following statements about familial periodic paralysis is true?
A) It is an autosomal-recessive transmitted disorder.
B) It involves disturbances of potassium regulation.
C) It is associated with permanent muscle weakness.
D) It is aggravated by administration of acetazolamide.
E) It most commonly affects the elderly

Answer: B

18. A 52-year-old man complains of severe right knee pain. He recalls no trauma, but attended a wine tasting party last weekend. He denies past medical history and is afebrile with normal vital signs. Physical examination reveals a swollen, red, painful right knee. Radiograph shows an effusion, but no bony erosions. You perform arthrocentesis and obtain 27 mL of cloudy straw-colored fluid, which you send to the laboratory for studies. The cell count is 50,000 WBCs/mm3 with 85% PMNs, glucose 120 mg/dL (serum 130), and protein 3.5 g/dL. The Gram stain shows numerous WBCs, but is negative for organisms. Evaluation for crystals shows numerous needle-shaped crystals with negative birefringence. A good treatment plan would be:
A) Oral colchicine 0.5 mg every hour until symptoms abate, GI toxicity develops or the maximum dose of 6 mg in 24 hours is reached. 
B) Subcutaneous colchicine 1 mg plus oral probenecid 250 mg bid for 1 week.
C) Sublingual colchicine 2 mg followed by oral steroids tapered over 5 days.
D) Intra-articular triamcinolone injection.
E) Oral allopurinol 500 mg tid for 1 week

Answer: A

19. A 38-year-old woman is brought to the emergency department by her spouse because of decreased mental status. She had knee surgery 2 days ago and was prescribed oral oxycodone for pain. Her spouse notes that she finished the entire 7-day supply during that day. He denies any seizure activity.They have no other drugs or medications in the house. She is afebrile with blood pressure of 130/75 mmHg, heart rate of 70 bpm, respiratory rate of 4 breaths/min, and SaO2 of 85% on room air. She barely responds to painful stimuli but moves all four extremities equally. Which of the following medications is most likely to improve her mental status?
A) Albuterol
B) Alvimopan
C) Flumazenil
D) N-Acetylcysteine
E) Naloxone

Answer: E

20. A 45-year-man with longstanding type 1 diabetes mellitus complains of pain in his feet and ankles that has been present for over a year. All of the following are consistent with neuropathic pain due to diabetes EXCEPT:
A) Burning pain
B) Electric shock quality
C) Exacerbated by light touch
D) Pain referred to scrotum
E) Tingling

Answer: D

21. Which of the following sets of drug–drug interaction and mechanism is accurately described?
A) Ibuprofen and warfarin: increased risk of GI bleeding; ibuprofen inhibition of CYP2C9
B) Sotalol and furosemide: increased risk of QT prolongation and torsades de pointes; furosemideinduced inhibition of CYP3A4
C) Sildenafil and sublingual nitroglycerin: increased risk of hypotension; sildenafil inhibition of the phosphodiesterase type 5 isoform that inactivates cyclic guanosine monophosphate
D) Ritonavir and lovastatin: increased risk of myotoxicity; ritonavir inhibition of CYP2C19
E) Allopurinol and azathioprine: increased risk of blood dyscrasias; allopurinol inhibition of Pglycoprotein

Answer: C

22. Mr. Jonas is a 47-year-old truck driver with a history of HIV, hypertension, coronary artery disease, atrial fibrillation, and ischemic cardiomyopathy. He is on antiretroviral therapy. He presents today complaining of a new rash on his chest and axilla, which you astutely diagnose as tinea corporis. You
would like to prescribe a course of oral ketoconazole for therapy. You should consider dose adjustment for all of the following medicines that he is already taking EXCEPT:
A) Carvedilol
B) Lovastatin
C) Mexiletine
D) Ritonavir
E) Saquinavir

Answer: A

23. A 24-year-old woman who has tested positive for human immunodefi ciency virus (HIV) presents to your officewith mild fever and symptoms of an upper respiratory
tract infection. She is currently in her 26th week of pregnancy and has a recent positive tuberculin skin test (TST) conversion with a 22-mm induration. Additionally, she has genital ulcerative lesions that test positive for herpes simplex virus. Which antimicrobial agent should you avoid prescribing for this patient?
A) Acyclovir
B) Amoxicillin
C) Tetracycline
D) Azithromycin
E) Isoniazid

Answer: C

24. Which of the following characteristics of Mycobacterium tuberculosis can be used to distinguish it from Mycobacterium bovis ?
A) Nonchromogenic pigmentation
B) Positive pyrazinamidase test
C) Negative 68°C catalase test
D) Positive nucleic acid probe for M tuberculosis complex
E) Negative arylsulfatase test

Answer: B

25. A 65-year-old man presents with a left footdrop. On examination,
the left anterior tibialis, extensor hallucis longus, and posterior tibialis muscles are weak. The strength of the quadriceps, gastrocnemius, and gluteus maximus muscles is normal. Stretch reflexes of the quadriceps and ankle muscle are normal. To which structure does the lesion best localize?
A) Peroneal nerve
B) L4 spinal root
C) L5 spinal root
D) Sciatic nerve
E) Thoracic spinal cord

Answer: C

26. A 55-year-old male who has a long history of marginally-controlled hypertension presents with gradually increasing shortness of breath and reduced exercise tolerance. His physical examination is normal except for a blood pressure of 140/90 mm Hg, bilateral basilar rales, and trace pitting edema. Which one of the following ancillary studies would be the preferred diagnostic tool for evaluating this patient? (check one)
A) 12-lead electrocardiography
B) Posteroanterior and lateral chest radiographs
C) 2-dimensional echocardiography with Doppler
D) Radionuclide ventriculography
E) Cardiac MRI

Answer: C

27. Which one of the following extraintestinal manifestations of inflammatory bowel disease do not parallel the course of intestinal inflammation and do not improve in parallel with improvement in intestinal symptoms?
A) peripheral arthritis
B) apthous ulcers
C) spondylitis and sacroiliitis
D) erythema nodosum
E) uveitis and iritis

Answer: C

28. An 18-year-old Asian woman is being treated for hepatitis B. Prior to therapy she was found to have ALT 198 U/L, AST 91 U/L, normal bilirubin, albumin, and prothrombin time. Liver biopsy results showed chronic hepatitis B, grade 3, stage 3. After 12 weeks of therapy, serum ALT is found to have increased to 1,082 U/L, bilirubin 2.1 mg/dL but albumin and prothrombin time remain normal. Apart from some fatigue, the patient is tolerating interferon well. The best course of action is:
A) check for antinuclear antibodies and total immunoglobulin level in serum and consider instituting corticosteroid therapy
B) stop interferon
C) recheck lab work again in 2 weeks time
D) add lamivudine to the regimen
E) ask the patient to skip three scheduled doses of interferon

Answer: C

29. A 53-year-old man presented with hypertension of 150/110 mmHg. He is generally asymptomatic and has no previous medical history of note. He is a smoker of 5 cigarettes daily and drinks modest quantities of alcohol. He takes no prescribed medications. Examination reveals a BMI of 33.5 kg/cm2 but nil else. Investigations: Serum sodium 146 mmol/l (NR 133-145); Serum potassium 3.2 mmol/l (NR 3.5 - 5); Urinary potassium excretion 42 mmol/l (NR less than 30). What is the likely diagnosis?
A) adrenocortical adenoma
B) Bartter's syndrome
C) Liddle's syndrome
D) liquorice ingestion
E) pheochromocytoma

Answer: A

30. A female patient aged 30 has a 5 years history of difficulty getting upstairs and out of a low chair and mild upper limb weakness but no pain. There is no family history. She presented with severe type 2 respiratory failure. EMG showed evidence of myopathy. 
The most likely diagnosis is:
A) Polymyositis
B) Inclusion body myositis
C) Acid maltase deficiency
D) Miller-Fisher Syndrome
E) Lambert-Eaton Myasthenic syndrome

Answer: C

31. A 28-year-man presents with a 1-week history of fever, headache, malaise, limb aching and constipation. Examination reveals a pulse rate of 80 beats/minute and high fever. His fever is gradually rising in a stepladder fashion. The anti-O and anti-H titres are high in the Widal test. Which one of the following is uncommon during the second week of this man’s illness?
A) Cough
B) Diarrhoea
C) Erythematous spots on the upper abdomen
D) Melaena
E) Splenomegaly

Answer: D

32. Which one of the following diseases shares many clinical characteristics with sarcoidosis?
A) Lymphangioleiomyomatosis
B) Berylliosis
C) Asbestosis
D) Alpha-1 antiprotease deficiency
E) Pulmonary alveolar proteinosis

Answer: B

33. A 19-year-old man presents to the emergency department with 2 days of ascending weakness. He had diarrhea 3 weeks earlier. He looks comfortable. On examination, you find moderate weakness in all limbs, with normal strength in facial muscles. You suspect Guillain-Barré syndrome and recommend admission to Neurology. What should be done first?
A) Spinal tap to look for albuminocytologic dissociation
B) Call for an emergency EMG to verify diagnosis
C) Send the patient for a whole spine MRI to rule out cord compression
D) Obtain pulmonary function tests, including FVC
E) Start high-dose steroids and then move to the floor

Answer: D

34. A 54-year-old woman is started on medical treatment following a diagnosis of acromegaly. Which one of the following is indicative of active disease?
A) Excessive sweating
B) Polyuria
C) Abdominal pain
D) Arthropathy
E) Goitre

Answer: A

35. Which of the following statements regarding hepatitis B infection in pregnancy is true?
A) Most women of childbearing age with chronic hepatitis B have a high risk of the development of complications of their disease during gestation.
B) Maternal-fetal transmission is responsible for most cases of hepatitis B worldwide.
C) Mothers who test negative for the hepatitis B e-antigen cannot transmit the virus to their fetuses.
D) Women with hepatitis B can be treated with interferon during pregnancy.
E) Women with hepatitis B should not be treated with lamivudine during pregnancy.

Answer: B

36. All of the following conditions or drugs decrease theophylline clearance, resulting in increased serum level of this drug EXCEPT:
A) Cimetidine
B) Cirrhosis
C) Allopurinol
D) Rifampin
E) Erythromycin

Answer: D

37. A 39-year-old man presents after tripping over his child’s toy and subsequently falling down the stairs. The patient reports landing on his chest and now complains of pain over certain regions of his right chest wall. He endorses some mild pain on deep inspiration. The patient is otherwise healthy and denies alcohol or smoking. Physical examination demonstrates pain on palpation over the sternum. A chest x-ray is ordered which confirms two right rib fractures. The radiologist notes an incidental 2.25-cm coin-shaped lesion on the left upper lung. There is no associated adenopathy or atelectasis noted. Which of the following is the most appropriate next step with respect to the solitary lung lesion?
A) CT scan of the chest
B) Lung biopsy
C) Review of an old chest x-ray
D) Reassurance
E) C-guided biopsy

Answer: C

38. Which of the following renal pathologies is most likely to recur in a renal transplant patient?
A) Diabetic renal disease
B) Focal segmental glomerulosclerosis 
C) Minimal change glomerulonephritis 
D) Membranous glomerulonephritis 
E) Mesangioproliferative glomerulonephritis

Answer: E

39. In which of the following patients is it appropriate to administer the vaccination against herpes zoster?
A) A 35-year-old woman who has never had varicella-zoster infection who is 12 weeks pregnant with her first child
B) A 54-year-old man who has never had varicella-zoster infection and is otherwise healthy
C) A 62-year-old man who had a car accident resulting in splenectomy
D) A 64-year-old woman with dermatomyositis-associated interstitial lung disease treated with prednisone 20 mg daily and azathioprine 150 mg daily
E) A 66-year-old woman who was recently diagnosed with non-Hodgkin lymphoma

Answer: C

40. A 23-year-old pregnant woman in her first trimester develops hyperthyroidism. Which of the following treatments is contraindicated?
A) thyroid surgery
B) propylthiouracil (PTU)
C) drugs that cross the placenta
D) radioactive iodine
E) glucocorticoids

Answer: D

41An elderly patient who becomes acutely short of breath presents with the combination of hypotension, elevated jugular venous pressure, and muffled heart sounds. This triad of symptoms is most suggestive of
A) Chronic pericarditis
B) Chronic pericardial effusion
C) Cardiac tamponade
D) Dissecting aortic aneurysm
E) Right heart failure

Answer: C

42A patient becomes markedly tetanic following a recent thyroidectomy. This symptom can be rapidly reversed by the administration of
A) Vitamin D
B) Calcitonin
C) PTH
D) Plicamycin (mithramycin)
E) Calcium gluconate (CaG)

Answer: E

43A 52-year-old female presents with nausea, fatigue, muscle weakness, and intermittent pain in her left flank. Laboratory examination reveals an increased serum calcium and a decreased serum phosphorus. The patient’s plasma parathyroid hormone levels are increased, but parathyroid hormone–related peptide levels are within normal limits. Urinary calcium is increased, and microhematuria is present. The patient’s abnormality is most likely caused by
A) Primary hyperparathyroidism
B) Primary hypoparathyroidism
C) Pseudohypoparathyroidism
D) Secondary hyperparathyroidism
E) Secondary hypoparathyroidism

Answer: A

44A 55-year-old female given a general anesthetic for a surgical procedure develops hyperthermia, hypertension, hyperkalemia, tachycardia, muscle rigidity, and metabolic acidosis. Which of the following general anesthetics did she receive?
A) Ketamine
B) Midazolam
C) Thiopental
D) Propofol
E) Halothane

Answer: E

45A patient with reduced VC, FRC, and RV is found to have a normal pH. A tentative diagnosis of diffuse interstitial fibrosis is made. Which of the following characteristics are consistent with this disease?
A) An increase in lung compliance
B) A decrease in respiratory rate
C) An increase in the V/Q ratio
D) A decrease in PaCO2
E) An increase in the FEV1/FVC ratio

Answer: E

46An HIV-positive patient asks if you can tell him the chances of him progressing to symptomatic AIDS. Which one of the following tests would be most useful?
A) CD4 lymphocyte count
B) HIV antibody test
C) HIV RT PCR
D) Neopterin
E) HIV p24 antigen

Answer: C

47. A 33-year-old female patient treated with haloperidol for a history of schizophrenia is seen in the ED because of complaints of fever, stiffness, and tremor. Her temperature is 104°F, and her serum creatine kinase (CK) level is elevated. What has occurred?
A) Overdose
B) Allergy
C) Neuroleptic malignant syndrome (NMS)
D) Tardive Dyskinesia
E) Parkinsonism

Answer: C

48A 43-year-old male is recovering from an infectious disease and experiences a marked instability in his blood pressure with episodes of spiking of blood pressure. After a series of extensive examinations, it was concluded that this disorder was due to the effects of the infectious agent upon a component of the peripheral nervous system. Logical sites where an infectious agent could produce such an effect include the
A) Superior ganglia of cranial nerves IX and X
B) Geniculate and trigeminal ganglia
C) Otic and superior salivatory ganglia
D) Carotid sinus and aortic arch
E) Carotid and aortic bodies

Answer: D

49A 25-year-old female suspected of having vitamin D–resistant rickets has decreased blood phosphate levels. Aside from high-dose vitamin D and oral phosphate, an alternative therapeutic approach might be the use of which of the following?
A) Estrogen
B) Pamidronate
C) Hydrochlorothiazide
D) Prednisone
E) Calcitrol

Answer: E

50A 30-year-old female stored her contact lenses in tap water. She noticed deterioration of vision and visited an ophthalmologist who diagnosed her with severe retinitis. Culture of the water as well as vitreous fluid would most likely reveal
A) Naegleria
B) Pneumocystis
C) Acanthamoeba
D) Babesia
E) Entamoeba coli

Answer: C

MULTIPLE CHOICE QUESTIONS-5

1. A 38-year-old woman is referred to you by a neurologist who was seeing her for severe headaches. His final diagnosis was stress headaches, but because a head MRI showed an empty sella, the neurologist wanted you to see her. On questioning, you learn that she has 7 children and all are healthy. When the first child was born, she experienced a difficult delivery and considerable bleeding that required a transfusion. All subsequent deliveries were uneventful. The review of systems was otherwise negative. Physical examination shows normally pigmented abdominal striae. The blood pressure is 100/60. You order some blood tests and find that the electrolytes, glucose, and TSH are normal. Which of the following is the most correct thing for you to do next?
A) Diagnose Sheehan syndrome and begin replacement therapy.
B) Evaluate for pituitary hypofunctioning.
C) Evaluate for pituitary hyperfunctioning.
D) Refer her to a neurosurgeon for exploration of her pituitary gland.
E) Tell her that everything is normal and discharge her from your clinic

Answer: E

2. The condition that characteristically results in recurrent axillary and groin abscesses is:
A) Regional enteritis.
B) Diabetes mellitus.
C) Ulcerative colitis.
D) Hidradenitis suppurativa.
E) Acne conglobata

Answer: D

3. The most important initial therapy for a patient with toxic epidermal necrolysis (TEN) is:
A) Corticosteroids.
B) Antibiotics.
C) Anticoagulants.
D) Analgesics.
E) Crystalloids

Answer: E

4. In evaluating the skin lesions of patients with AIDS:
A) Kaposi sarcoma is common but seldom more than a cosmetic problem.
B) Candidiasis may occur but is easily treated with standard medications.
C) Tinea corporis is no more likely in these patients than in the general population.
D) Lichen planus of the oral mucosa is associated with the disease.
E) Seborrheic dermatitis-like eruptions are frequently found

Answer: A 

5. A 57-year-old man comes to the emergency department because of excruciating pain in his right big toe. He describes the pain as so severe that it woke him from a deep sleep. He has no chronic medical conditions, does not take any medications, and denies any similar episodes in the past. He admits to a few "drinking binges" over the past 2 weeks. His temperature is 38.1 C (100.5 F), blood pressure is 130/90 mm Hg, and pulse is 80/min. Examination shows an erythematous, warm, swollen, and exquisitely tender right great toe. The skin overlying the first metatarsophalangeal joint is dark red, tense, and shiny. Synovial fluid analysis reveals negatively birefringent, needle-shaped crystals within polymorphonuclear leukocytes (PMNs). Laboratory studies show: 
Serum Leukocytes........16,000/mm3; Uric acid...........15 mg/dL; Calcium.............9 mg/dL 
Which of the following is the most appropriate pharmacotherapy? 
A. Allopurinol 
B. Ceftriaxone 
C. Cyclooxygenase-2 inhibitors but in higher dosages than are typically used. 
D. Probenecid 
E. Sulfinpyrazone

Answer: C

6. A 28-year-old woman is diagnosed with lupus nephritis, World Health Organization (WHO) type IV. She has a malar rash, diffuse arthritis, and edema. Her blood pressure is 190/110 mm Hg. Her creatinine is 2.1 mg/dL with a blood urea nitrogen of 28 mg/dL. Her urine reveals 25 red blood cells per hpf, and 3+ protein. One red blood cell cast is seen. A 24-hour urine collection reveals a protein of 11 grams with a creatinine of 1 gram. Which of the following would be the most appropriate management? 
A) Oral azathioprine 
B) Oral cyclophosphamide 
C) Oral gold 
D) Oral prednisone 
E) Pulse IV cyclophosphamide

Answer: E

7. A 50-year-old man is evaluated for a routine physical examination. He was diagnosed with rheumatoid arthritis 5 years ago, and his condition is well controlled with methotrexate and hydroxychloroquine. He does not have fatigue, morning stiffness, or systemic symptoms. His weight is stable. On physical examination, there are no joint deformities. There is minimal soft-tissue swelling around the second and third metacarpophalangeal joints. Grip strength is normal, but squeezing the forefeet elicits mild discomfort. On laboratory studies, hemoglobin is 14.2 g/dL (142 g/L), alanine and aspartate aminotransferase and alkaline phosphatase are normal, and erythrocyte sedimentation rate is 65 mm/h. Which of the following is the most appropriate management for this patient?
A. Prednisone, 10 mg daily
B. Prednisone, 40 mg daily
C. Anti–tumor necrosis factor therapy
D. CT scan of the chest, abdomen, and pelvis
E. No additional therapy

Answer: D

8. A family physician cares for a family consisting of a 45-year old husband, 43-year-old wife and a 12-year-old daughter. The family reports that recently the 77-year-old maternal grandmother who lived with them died after a prolonged respiratory infection. Autopsy subsequently confirms that she had active pulmonary tuberculosis at the time of death. The organism tested sensitive to all anti-tuberculosis drugs. In responding to the grandmother's illness, which of the following is the most appropriate step in management?
A) Obtain leukocyte counts on all family members 
B) Obtain sputum cultures for acid fast bacilli 
C) Obtain chest computerized tomograms on all members 
D) Place protein purified derivative (PPD) test on all members 
E) Schedule bronchoscopy lavage for the adults

Answer: D

9. A patient with a history of papillary thyroid cancer is noted to
have an enlarged lateral neck lymph node on routine
examination. Which one of the following is the most sensitive and specific indicator of metastatic disease within the enlarged lymph
node?
A. Neck ultrasound
B. Fine needle aspiration cytology
C. Stimulated serum thyroglobulin
D. PET‐CT scanning
E. Thyroglobulin wash‐out testing

Answer: D

10. A 60-year-old woman is establishing care in your clinic. She has occasional numbness and tingling in her fingers and toes and has noted some numbness around her mouth, especially when she is stressed or anxious. She had thyroid surgery for Graves’ disease about 2 years ago and takes 100 µg of levothyroxine and 1 tablet of calcium daily. On exam, her blood pressure is 130/80, pulse 80, and she has cramping in her right forearm and fingers when the blood pressure cuff is attached. Based on this history and exam, which of the following is most likely?
A) Calcium 8.5, PTH 65, PO4 4.5
B) Calcium 9.5, PTH 35, PO4 4.0
C) Calcium 6.0 (8.5-10.5), PTH 2 (10-65), PO4 6.0 (2.7-4.5)
D) Calcium 10.8, PTH 108, PO4 2.3
E) Calcium 8.0, PTH 98, PO4 2.1

Answer: C

11. Which of the following sets of drug–drug interaction and mechanism is accurately described?
A. Ibuprofen and warfarin: increased risk of GI bleeding; ibuprofen inhibition of CYP2C9
B. Sotalol and furosemide: increased risk of QT prolongation and torsades de pointes; furosemideinduced inhibition of CYP3A4
C. Sildenafil and sublingual nitroglycerin: increased risk of hypotension; sildenafil inhibition of the phosphodiesterase type 5 isoform that inactivates cyclic guanosine monophosphate
D. Ritonavir and lovastatin: increased risk of myotoxicity; ritonavir inhibition of CYP2C19
E. Allopurinol and azathioprine: increased risk of blood dyscrasias; allopurinol inhibition of Pglycoprotein

Answer: C

12. A 47-year-old truck driver with a history of HIV, hypertension, coronary artery disease, atrial fibrillation, and ischemic cardiomyopathy. He is on antiretroviral therapy. He presents today complaining of a new rash on his chest and axilla, which you astutely diagnose as tinea corporis. You
would like to prescribe a course of oral ketoconazole for therapy. You should consider dose adjustment for all of the following medicines that he is already taking EXCEPT:
A. Carvedilol
B. Lovastatin
C. Mexiletine
D. Ritonavir
E. Saquinavir

Answer: A

13. A 17-year-old patient who is known to have Wolff-Parkinson-White syndrome presents with a regular narrow complex tachycardia with a cycle length of 375 milliseconds (160 bpm) that occurred with a sudden onset. You note that there is a 1:1 atrial-to-ventricular relationship and that the RP interval is 100 milliseconds. The best initial treatment is
A) IV procainamide.
B) atropine.
C) vagal maneuvers.
D) catheter ablation

Answer: C

14. A 65-year-old man presents after an arrest while eating at a local restaurant. On arrival, paramedics documented ventricular fibrillation (VF), and he was successfully resuscitated. He has a history of myocardial infarction (MI) and congestive heart failure (CHF). Serum electrolytes are remarkable only for mild hypokalemia. MI is ruled out by ECG and serial blood tests of myocardial enzymes. Subsequent evaluation includes cardiac catheterization, which shows severe three-vessel coronary artery disease (CAD) and severe left ventricular (LV) systolic dysfunction. A nuclear myocardial perfusion scan shows a large area of myocardial scar without significant viability in the territory of the left anterior descending coronary artery. The decision is made to treat the CAD medically. Which of the following is the best management strategy for his arrhythmia?
A) PO amiodarone
B) Implantable cardioverter defibrillator (ICD) implantation if an electrophysiologic (EP) study shows inducible VT or VF
C) ICD implantation
D) β-Blocker medication

Answer: C

15. A 56-year-old woman is undergoing a cadaveric renal transplant. After revascularization of the transplanted kidney the transplanted renal parenchyma becomes swollen and blue. Which of the following statements is most accurate regarding her transplanted kidney?
A) The donor had pre-formed antibodies against the recipient’s HLA antigens.
B) It is characterized pathologically by fibrin and platelet thrombosis of renal arterioles and small arteries and necrosis of the glomerular tufts.
C) Biopsies should not be obtained intraoperatively.
D) This form of rejection is associated with disseminated intravascular coagulation (DIC).
E) The rejection process can be treated with a steroid bolus and OKT3

Answer: C

16. A 55-year-old man comes to the physician with the chief complaint of weight loss and a depressed mood. He feels tired all the time and is no longer interested in the normal activities he previously enjoyed. He feels quite apathetic overall. He has also noticed that he has frequent, nonspecific abdominal pain. Which of the following diagnoses needs to be ruled out for this man?
A) Pheochromocytoma
B) Pancreatic carcinoma
C) Adrenocortical insufficiency
D) Cushing syndrome
E) Huntington disease

Answer: B

17. You are seeing a 78-year-old man who was brought to the office by his daughter. The daughter says her father is becoming increasingly forgetful. His medical history is significant for a 20-year history of type 2 diabetes and well-controlled hypertension. On examination, he is mildly hypertensive with otherwise normal vital signs. He is oriented to time, place, and person, but is unable to complete “serial sevens” on a mini-mental status examination. Which of the historical features make this diagnosis more consistent with dementia as opposed to delirium?
A) His history of hypertension
B) His history of diabetes
C) His current level of orientation
D) His inability to complete serial sevens
E) The recent onset of his symptoms

Answer: C

18. A 45-year-old woman with Crohn’s disease and a small intestinal fistula develops tetany during the second week of parenteral nutrition. The laboratory findings include Ca 8.2 meq/L; Na 135 meq/L; K 3.2 meq/L; Cl 103 meq/L; PO4 2.4 meq/L; albumin 2.4; pH 7.48; 38 kPa; P 84 kPa; bicarbonate 25 meq/L. The most likely cause of the tetany is
A) Hyperventilation
B) Hypocalcemia
C) Hypomagnesemia
D) Essential fatty acid deficiency
E) Focal seizure

Answer: C

19. A 60 year-old woman has had a mitral valve replacement for chronic mitral stenosis and is on cardiac ITU post-operatively. She is awake and self ventilating. SPO2 91%, FiO2= 0.6. Monitoring shows atrial fibrillation at 90/min, blood pressure 88/60 and pulmonary artery pressures of 45/15. Which single drug is most appropriate first line treatment to reduce the pulmonary artery pressure? 
A) Inhaled nitric oxide 
B) Inhaled prostacyclin 
C) Oral sildenafil 
D) Intravenous isoprenaline 
E) Intravenous milrinone

Answer: E

20. A 62-year-old man is admitted to the emergency department with abdominal pain. The patient has a past history of ischaemic heart disease and atrial fibrillation. Computed tomography scan features are highly suggestive of ischaemic bowel. The patient's blood gases are as follows: pH = 7.25; paO2 = 10; paCO2 = 2.8; HCO3 = 18; Base excess = –8 
Which of the following best describes the patient's acid–base status? 
A) Metabolic acidosis 
B) Metabolic acidosis with respiratory compensation 
C) Respiratory acidosis with metabolic compensation 
D) Metabolic acidosis with inadequate respiratory compensation 
E) Cannot be sure without a serum lactate level

Answer: D

21. Two weeks ago, a 54-year-old man developed a cold sore, which resolved uneventfully. He now complains of a nonpruritic skin rash. Examination reveals raised red lesions resembling hives, some with clear fluid bullae. They are located on his hands, including the palms, and his forearms and anterior tibia. The best way to confirm your suspected diagnosis is by:
A) Viral culture of blister fluid.
B) Smear of blister fluid for Gram stain.
C) Tzanck preparation of blister fluid to look for multinucleate giant cells.
D) Full thickness skin biopsy of involved area.
E) Wood’s light examination of involved areas

Answer: D

22. A 54-year-old man presents for a periodic health examination. His family history is significant for his mother who died of a cerebrovascular accident at age 72, his father who died of a myocardial infarction at age 68, and a brother who developed sigmoid cancer at age 60. The patient is on no medications except for aspirin, 81 mg daily. His physical examination is unremarkable. The patient asks for a recommendation regarding current cancer screening. Which of the following is the most appropriate screening test for this patient? 
A) Annual digital rectal examination and fecal occult blood testing 
B) Flexible sigmoidoscopy 
C) Flexible sigmoidoscopy and barium enema 
D) Colonoscopy 
E) Genetic testing for the p53 gene

Answer: D

23. Which of the following treatment options has been most consistently shown to be effective for the primary prevention of sudden cardiac death in patients with CAD and recent MI?
a. D-Sotalol
b. β-Blocker medications
c. Amiodarone
d. Dofetilide

Answer: B

24A 72-year-old woman is about to undergo an elective total hip replacement for osteoarthritis. She has a history of hypertension and type 2 diabetes mellitus but no ischaemic heart disease or peripheral vascular disease. Which of the following is the most appropriate thromboembolic prophylaxis? 
A) Intermittent pneumatic calf compression 
B) Calf-length thromboembolic deterrent elastic stockings and early ambulation 
C) Full-dose unfractionated heparin to increase the activated partial thromboplastin time to two time control 
D) Insertion of an inferior vena cava filter 
E) Subcutaneous low-molecular-weight heparin

Answer: E

25An AIDS patient under treatment with a nucleoside analog and a protease inhibitor comes to medical attention with complaints of leg weakness and incontinence. His vital signs are within normal limits. Physical examination reveals reduced strength in the lower extremities with accompanying mild spasticity. There is also diminished sensation in the feet and legs bilaterally. Lumbar puncture shows:
Opening pressure.....100 mm H20
Cell count................5 lymphocytes/mm3
Glucose...................48 mg/dL
Proteins, total..........33 mg/dL
Gamma globulin.......8% total protein
Additional laboratory investigations show normal hematologic parameters, vitamin B12 within normal values, and negative serology for syphilis. MRI of the head fails to reveal any focal abnormality. Which of the following is the most likely diagnosis?
A) AIDS dementia complex 
B) CMV polyradiculopathy 
C) Cryptococcal meningoencephalitis 
D) Vacuolar (HIV) myelopathy 
E) Zidovudine-related toxicity

Answer: D

26. A 59-year-old man presents to the ED with left-sided chest pain and shortness of breath that began 2 hours prior to arrival. He states the pain is pressure-like and radiates down his left arm. He is diaphoretic. His BP is 160/80 mm Hg, HR 86 beats per minute, and RR 15 breaths per minute. ECG reveals 2-mm ST-segment elevation in leads I, aVL, V3 to V6. Which of the following is an absolute contraindication to receiving thrombolytic therapy?
A) Systolic BP greater than 180 mm Hg
B) Patient on Coumadin and aspirin
C) Total hip replacement 3 months ago
D) Peptic ulcer disease
E) Previous hemorrhagic stroke

Answer: E

27. Which of the following statements is true about the patient with a bleeding disorder?
A) Factor deficiencies generally present with mucosal bleeding/petechiae.
B) Hemarthrosis generally reflects a platelet deficiency.
C) Hematomas are usually the result of a factor deficiency.
D) All of the above are true

Answer: C

28. Flu season is right around the corner and you are preparing your clinic for the onslaught. First things first … you need to know how much vaccine to order and who will be receiving it. The Centers for Disease Control and Prevention (CDC) annually publishes recommendations for administering influenza vaccine to the American public. The CDC recommends vaccination for all of the following groups, EXCEPT:
A) Health-care workers.
B) Nursing home residents.
C) Egg-allergic, febrile neonates.
D) Diabetics.
E) The elderly

Answer: C

29. During an outbreak, what intervention(s) is/are most appropriate for all your unvaccinated, frail nursing home patients who have no symptoms of febrile respiratory illness?
A) Antiviral prophylaxis with oseltamivir.
B) Antiviral prophylaxis with amantadine.
C) Influenza immunization.
D) A and C given together.
E) B and C given together

Answer: D

30. Which of the following is indicated when evaluating for a suspected inherited coagulopathy?
A) CBC.
B) PT and PTT.
C) Platelet count.
D) PFA-100.
E) All of the above

Answer: E

31. A 43-year-old woman with rheumatoid arthritis visits for a scheduled follow-up. Plain X-ray of the hands reveals marginal erosions at the metacarpal heads. Why are these erosions marginal?
A) Random localization
B) Plain X-rays fail to show central erosions
C) Rotation of the film
D) Marginal joint area is devoid of overlying cartilage
E) Presence of sesamoid bones

Answer: D

32. A 23-year-oldman attends his GP complaining of reduced exercise tolerance, nocturia and frequent hiccups over the course of the past 4 months. He also has generalised bony pain. His BP is 165/90 mmHg and his BMI is stable at 25.
The results of routine blood testsshow:
Creatinine 250 μmol/l
Haemoglobin 14g/dl
Phosphate 2.0 mmol/l
Calcium 2.1 mmol/l
What is the most likely cause of these results?
A) Adult polycystic kidney disease 
B) Diabetes
C) Contrast nephropathy 
D) Hypertensive nephropathy
E) Diffuse proliferative glomerulonephritis

Answer: A

33. A 41-year-old man with a history of nasal congestion, breathlessness, cough and wheeze presents with a left foot drop. Which one of the following is the most likely diagnosis?
A) Diabetes mellitus
B) Wegener’s granulomatosis
C) Churg–Strauss syndrome
D) Pulmonary eosinophilia
E) Polyarteritis nodosa

Answer: C

34. The preferred initial therapy for elderly patients with arthralgia due to osteoarthritis is which of the following?
A) NSAIDs.
B) COX-2 inhibitors.
C) Acetaminophen.
D) Combination narcotic analgesics.
E) Early joint replacement.

Answer: C

35. According to the Eighth Joint National Committee (JNC-8) guideline on managing hypertension in 60 years old patient or more (with no DM or chronic kidney disease) the BP lowering goal is:
A) <150/90 mm Hg
B) <140/90 mm Hg
C) <160/90 mm Hg
D) <120/90 mm Hg
E) <130/90 mm Hg

Answer: A

36. In general, all of the following are risk factors for gout EXCEPT:
A) Tobacco use.
B) Alcohol use.
C) Obesity.
D) Diuretic use.
E) Family history.

Answer: A

37. A 59-year-old woman had three episodes of right renal colic over the last year. Review of system was remarkable for joint pain and fatigability. Past and family history were unremarkable. She is 7 years postmenopause and takes no medication. On physical examination BMI 28.5 kg/m2, pulse 72 beat per minute, BP 150/95 mmHg. Laboratory data revealed. Fasting plasma glucose 89 mg/dl: Serum creatinine 1.1 mg/dl: Serum total calcium 11.9 mg/dl: Serum ionized calcium 6.8 mg/dl: Ultrasound of the abdomen revealed 2 stones at the right renal pelvis: 24 hour urinary calcium 365 mg (N < 250 mg) The most appropriate next diagnostic test is:
A) Measurement of serum 25 hydroxyvitamin D.
B) Measurement of serum PTH.
C) Meseurement of serum parathyroid related protein.
D) Genetic testing for mutation in parathyroid calcium sensing receptor.
E) 24 hour urinary uric acid.

Answer: B

38. A woman presents with headache, lethargy and weight loss. Which one of the following would make the diagnosis of giant cell arteritis unlikely?
A) A raised ESR
B) Bilateral headache
C) Non-tender temporal arteries
D) Papilloedema on fundoscopy
E) Patient is 60 years old

Answer: D

39. Which of the following statements regarding polycythemia vera is correct?
A) An elevated plasma erythropoietin level excludes the diagnosis.
B) Transformation to acute leukemia is common.
C) Thrombocytosis correlates strongly with thrombotic risk.
D) Aspirin should be prescribed to all these patients to reduce thrombotic risk.
E) Phlebotomy is used only after hydroxyurea and interferon have been tried.

Answer: A

40. A 49-year-old female with a 5-year history of diabetes mellitus type 2 presents for an initial visit. She has no known complications of diabetes. She takes metformin, glyburide, andaspirin. On examination, you find a pleasant, obese female in no distress. Her blood pressure is 136/86 mm Hg, pulse 86, respirations 14, and temperature 37°C. As you discuss monitoring her diabetes, you recommend screening for early kidney disease. Which of the following approaches is the recommended way to screen for diabetic kidney disease?
A) Obtain a 24-hour urine collection for albumin now and again in 3 years.
B) Obtain a spot urine microalbumin every year.
C) Obtain a spot urine microalbumin/creatinine ratio every year.
D) Obtain a urinalysis every year.
E) Obtain a serum creatinine every year.

Answer: C

41.  A 34-year-old female presents with recurrent episodes of severe headaches, palpitations, tachycardia, and sweating. A physical examination reveals her blood pressure to be within normal limits; however, during one of these episodes of headaches, palpitations, and tachycardia, her blood pressure is found to be markedly elevated. Workup finds a small tumor of the right adrenal gland. Which one of the following is most likely to be increased in the urine of this individual?
A) Acetone
B) Aminolevulinic acid (ALA)
C) Hydroxy-indoleacetic acid (HIAA)
D) N-formiminoglutamate (FIGlu)
E) Vanillylmandelic acid (VMA)

Answer: E

42One of your 50-year-old male patients with CKD 5 is considering PD as one of his choices for renal replacement therapy. He asks you about indications and contraindications about PD. Which one of the following choices is a contraindication for PD?
A) Abdominal hernia not amenable to surgery
B) Adhesions from previous surgery
C) Severe inflammatory bowl disease
D) Diaphragmatic fluid leak
E) All of the above

Answer: E

43Foreign bodies and/or food boluses can lodge in the esophagus in any of the following four areas of narrowing except:
A) Hiatal hernia
B) Upper esophageal sphincter
C) Level of the aortic arch
D) Level of the mainstem bronchus
E) Gastroesophageal junction

Answer: A

44A 62-year-old man with CKD 5 (eGFR 11 mL/min) due to HTN comes to the clinic for routine follow-up with no complaints of nausea, vomiting, fatigue, or poor appetite. An arterio-venous fistula (AVF) was placed 1 year ago when his eGFR was 12 mL/min and the fistula is ready for use. His BP is 134/80 mmHg. He walks 2 miles everyday without shortness of breath, or chest pain, or fatigue. Pertinent labs include: Na+ 139 mEq/L, K+ 4.4 mEq/L, HCO3 22 mEq/L, BUN 68 mg/dL, Ca2+ 8.8 mg/dL, phosphate 4.2 mg/dL, and albumin 4.1 g/dL. He expresses hemodialysis (HD) as his choice of renal replacement therapy (RRT). According to the KDIGO guideline, which one of the following is the MOST appropriate management in this patient?
A) Start HD in 2 weeks at the outpatient dialysis unit
B) Start peritoneal dialysis (PD) in 4 weeks
C) Convince for preemptive kidney transplantation
D) Start HD when signs and symptoms of kidney failure are present
E) Suggest no RRT at any time, as he may do well with conservative management

Answer: D

45A 75-year-old man underwent surgery to correct a large abdominal aortic aneurysm. The procedure appeared to go well, but you are called a few hours later to evaluate the patient who states that he cannot move or feel his legs. On the way to the ICU, you consider the possible causes of his symptoms and plan your physical examination. What is the most important test to help localize the lesion?
A) MRI of the spine
B) Sensory level
C) Reflexes in lower extremities
D) Plantar flexion reflex
E) Toe position sense

Answer: E

46Which one of the following statements in CKD 5 patients regarding early (eGFR 10–14 mL/min) versus late (eGFR 5–7 mL/min) initiation of HD is CORRECT?
A) Early initiation of HD improves mortality and morbidity of patients
B) Late initiation of HD improves mortality and morbidity of patients
C) No difference between early and late initiation of HD either in survival or other outcomes such as hospitalizations or quality of life
D) Compared to early initiation, late initiation is better in controlling mortality only
E) None of the above

Answer: D

47A 69-year-old man was successfully defibrillated after an episode of ventricular fibrillation secondary to an ST elevation myocardial infarction (STEMI) and transferred to the cardiac catheter laboratory for primary coronary intervention. After the procedure began he had a further episode of ventricular fibrillation. Regarding defibrillation, which is true?
A) A single direct current shock of 360 joules with a biphasic waveform is the most likely to restore spontaneous circulation
B) Defibrillation is no more likely to be successful than a properly delivered praecordial thump
C) It is safe to continue with the coronary angiogram while the shock is delivered to the patient
D) Three shocks delivered with minimal interruptions should be given before any other intervention
E) Two minutes of chest compressions before defibrillation is recommended to optimize coronary perfusion

Answer: D

48Which of the following statements regarding pillinduced esophagitis is true?
A) A bronchoesophageal fi stula is a common complication of NSAID-induced esophageal ulceration.
B) Tetracycline or its derivatives cause pillinduced esophagitis by production of a caustic alkaline solution.
C) Antibiotics as a class are uncommon causes of medication-induced esophagitis.
D) Esophageal damage from a bisphosphonate medication, such as alendronate, can be minimized by ingestion of a full 8-oz glass of water taken in the upright position.
E) Chemotherapeutic agents are unlikely causes of pill-induced esophagitis.

Answer: D

49A 32-year-old nurse presents with symptoms of dizziness, jittery behavior, and headaches before meals. Which of the following supports the diagnosis of factitious hypoglycemia?
A) Elevated sulfonylurea levels
B) Normal proinsulin levels
C) Normal C-peptide levels
D) Plasma insulin-to-glucose ratio <0.3
E) All of the above

Answer: E

50. All of the following paraneoplastic syndromes are associated with gastric cancer except:
A) Acanthosis nigrans
B) Disseminated intravascular coagulation
C) Thrombophlebitis (Trosseau’s sign)
D) Pyoderma gangrenosum
E) Nephrotic syndrome

Answer: D

MULTIPLE CHOICE QUESTIONS-6

1A broncho-pleural fistula is an abnormal communication or a passage between the bronchial tree and the pleural space, causing a persistent leak. If these patients are mechanically ventilated, the management strategy should be:
A) Low tidal volumes and high respiratory rate
B) Reduced inspiratory pressures
C) High tidal volumes and low respiratory rate
D) Low inspiratory times and high PEEP
E) High inspiratory times and low PEEP

Answer: B

2Which of the following is a recognized feature of pulmonary embolism except?
A) S1,Q3,T3
B) an increase in serum troponin levels
C) an arterial pH greater than 7.2
D) increased PCO2 on air
E) positive D-dimer levels

Answer: D

3A 45-year-old surgeon sustained a needle stick injury from a known HIV-positive individual during a routine gall bladder operation and subsequently commenced a course of post-exposure prophylaxis (PEP). Which of the following statements regarding his management is CORRECT?
A) He can discontinue PEP if it is established that the source patient has an undetectable HIV viral load
B) He must refrain from operating until he has proven to be HIV-negative after completion of PEP
C) If he has successfully completed a course of PEP, there is no requirement for him to have a subsequent HIV antibody test
D) He cannot be considered HIV-negative until he tests HIV antibody-negative at three months post-PEP
E) He cannot be considered HIV-negative until he tests HIV antibody-negative at six months post-PEP

Answer: E

4A patient presents with gradually worsening weakness of the proximal arm and leg muscles symmetrically over several months. On examination, neck flexors and extensors are found to be weak also. There is no muscle pain or tenderness. What is the most likely site of dysfunction in the nervous system?
A) Peripheral nerve
B) Brachial plexus
C) Spinal nerve root
D) Internal capsule
E) Muscle

Answer: E

5A 67 year old lady is referred to the bone clinic following a fractured wrist. A DEXA scan is performed which shows a T-score of −2.0. She has no other osteoporotic risk factors. Which one of the following treatments would you offer?
A) Lifestyle advice and vitamin D supplementation
B) Alendronic acid
C) HRT
D) Risedronate
E) No treatment

Answer: A

6A 56-year-old man is admitted to the intensive care unit for acute respiratory distress syndrome (ARDS). The patient was transferred from an outside hospital today after a 2-week hospitalization for pneumonia. During that time, the patient's pulmonary status continued to deteriorate. One week ago he was intubated and placed on mechanical ventilation and over the past week, his oxygenation has worsened with a PaO2 of 66 on an inspired concentration of 100% oxygen. The patient has no other medical history except for rheumatoid arthritis. On transfer to the ICU, the patient is intubated and sedated on a mechanical ventilator. His chest radiograph shows patchy, bilateral, diffuse interstitial infiltrates. The most important intervention that will most benefit this patient is to
A) keep the patient in a prone position during mechanical ventilation
B) keep tidal volumes greater than 15 cc/kg
C) limit peak inspiratory pressure to 45 cm H2O or less
D) limit PEEP levels to less than 10 cm H2O
E) limit tidal volumes to 6cc/kg

Answer: E

7A previously fit and well 50-year-old male underwent a decompressive craniectomy for an acute subdural haematoma following a traumatic head injury 24 hours ago. He is now on the neurosurgical intensive care unit and remains intubated and ventilated. You have been asked to review his urine output, which is recorded as 800 ml over the last 2 hours, despite appropriate intravenous fluid administration. You suspect neurogenic diabetes insipidus.
Which of the following features would support your diagnosis?
A) Urine specific gravity >1.005
B) Serum Na+ <135 mmol/l
C) Urine osmolality <350 mmol/kg
D) Serum osmolality <295mmol/kg
E) Serum creatinine 200 mmol/l

Answer: B

8A 35 year old man presents to his GP with a 2-week history of painful, swollen knees and a painful left heel. He also reveals that he has been experiencing a burning pain when he urinates and has a red, itchy eye. Which one of the following is the most likely diagnosis?
A) Gout
B) Osteoarthritis
C) Enteropathic arthritis
D) Ankylosing spondylitis
E) Reactive arthritis

Answer: E

9A 40-year-old female who is intubated and ventilated following a subarachnoid hemorrhage (SAH) 7 days previously has a serum sodium concentration of 128mmol/l and serum osmolality of 270 mOsm/kg. Which of the following statements is true?
A) Cerebral salt-wasting syndrome (CSWS) is rarely associated with SAH
B) Cerebral salt-wasting syndrome is associated with a reduced serum osmolality
C) To diagnose SIADH, the patient must be clinically dehydrated
D) SIADH almost always requires pharmacological treatment
E) To diagnose SIADH urine osmolality must be greater than serum osmolality

Answer: E

10A 47-year-old woman who is 2 weeks post triple bypass surgery presents to the emergency department with a chief complaint of sudden onset, sharp chest pain for several hours. She is fatigued and short of breath. On physical examination she has distended neck veins that grow more distended on inspiration. Muffled heart sounds are heard. Her temperature is 37.0°C (98.6°F), pulse is 133/min, blood pressure is 70/50 mm Hg, respiratory rate is 30/min, and oxygen saturation is 100% on room air. An echocardiogram shows a large pericardial effusion and chamber collapse; therefore, pericardiocentesis is performed. Although a large amount of blood is aspirated, the patient’s clinical picture acutely worsens. Her pain level increases substantially; pulse is 150/min, blood pressure is 60/41 mm Hg, respiratory rate is 30/min, and oxygen saturation is 100%. Repeat echocardiography shows an even larger pericardial effusion with chamber collapse. Which complication of pericardiocentesis is most likely in this patient?
(A) Acute left ventricular failure with pulmonary edema
(B) Aspiration of 10 mL air into the pericardium
(C) Laceration of a coronary vessel
(D) Pneumothorax
(E) Puncture of the left ventricle

Answer: C

11During this outbreak, what intervention(s) is/are most appropriate for all your unvaccinated, frail nursing home patients who have no symptoms of febrile respiratory illness?
A) Antiviral prophylaxis with oseltamivir.
B) Antiviral prophylaxis with amantadine.
C) Influenza immunization.
D) A and C given together.
E) B and C given together

Answer: D

12A 38-year-old male presents with episodic wheeze and non-productive cough which occurs particularly at night. He has been employed in the plastics industry. Which of the following suggests a diagnosis of occupational lung disease?
A) Absent family history of asthma
B) Commencement of symptoms on his first day in this employment
C) Elevated serum IgE concentration
D) Improved symptomatology when on holiday
E) Increased bronchial reactivity

Answer: D

13A 68-year-old male is admitted with a two months history of difficulty raising his arms, ascending stairs, and is also aware of a dry mouth. He smokes 15 cigarettes daily and admits to heavy alcohol consumption. On examination he has proximal weakness affecting all four limbs with absent tendon reflexes. His chest X-ray shows a right pleural effusion. What is the most likely diagnosis?
A) Alcohol induced myopathy
B) Eaton- Lambert syndrome
C) Myasthenia gravis
D) Polymyalgia rheumatica
E) Polymyositis

Answer: B

14A 65-year-old woman, a heavy smoker for many years, has had worsening dyspnoea for the past five years, without a significant cough. A chest x ray shows increased lung size along with flattening of the diaphragm, consistent with emphysema. Over the next several years she develops worsening peripheral oedema. Her BP 115/70 mmHg. Which of the following cardiac findings is most likely to be present?
A) Constrictive pericarditis 
B) Left ventricular aneurysm 
C) Mitral valve stenosis 
D) Non-bacterial thrombotic endocarditis 
E) Right ventricular hypertrophy

Answer: E

15A young male developed spontaneous DVT of right lower limb and he is having a sister who had DVT 5 years back and having protein C deficiency. He was started on warfarin and the INR came to 2-3 after which he had started following at local hospital where his warfarin dose was steadily increased as his INR never went higher than 1.5 and is currently on warfarin dose of 20 mg/day. He was referred back to hospital and was evaluated and found to be having warfarin level of 2.385 mg/l (therapeutic range 0.7 – 2.3 mg/l), PIVKA is > 10 
(Ref. range < 0.2). What is the most likely explanation for the subtherapeutic INR?
A) Cytochrome p450 mutation
B) VKORC 1 mutation
C) Not taking warfarin
D) Local hospital INR testing quality control is not good

Answer: D

16Which of the following anatomical considerations is correct?
A) Optic chiasm lesions characteristically produce a bitemporal hemianopia.
B) Central scotoma occurs early in papilloedema.
C) In cortical blindness pupillary reactions are abnormal.
D) Optic tract lesions produce an ipsilateral homonymous hemianopia.
E) Opticokinetic nystagmus is found with bilateral infarction of the parieto-occipital lobes

Answer: A

17The most common non-opportunistic protozoon parasite is AIDS patients is:
A) Cryptosporidium 
B) Gardia lamblia 
C) Blastomyces hominis 
D) Entamoeba histolytica 
E) Toxoplasma gondii

Answer: D

18To assist in the decision whether to hospitalize a patient with community acquired pneumonia (CAP), each of the following may be a factor in favor of hospitalization except for which one?
A) The patient is confused
B) Serum creatinine >2.0 mg/dL
C) Respiratory rate >30
D) Blood pressure < 90 mm Hg
E) Age >64 years

Answer: B

19A 35-year-old African-American woman complains of red and irritated eyes with photophobia for about 2 months. Visual acuity is 20/25 for each eye separately, and she says this is her “normal.” On questioning she admits to shortness of breath with exertion, which she attributes to neglecting physical training and advancing age. A chest x-ray shows perihilar adenopathy. A lung biopsy finds noncaseous granuloma. Angiotensinconverting enzyme is elevated. On spirometry, FEV 1 is 80% of predicted normal for her (percent of vital capacity expired in 1 second). Which of the following would be the best therapeutic approach.
A) Non-steroidal anti-inflammatory drugs (NSAIDs)
B) Bronchodilators
C) Inhaled glucosteroids
D) Observation for 4–6 months while treating the eyes symptomatically
E) Systemic glucosteroids

Answer: D

20A 30 year old lady underwent prosthetic valve replacement surgery done for rheumatic heart disease and she is started on UFH by continuous infusion plus warfarin. Six days after she developed DVT of right lower limb. Her CBC was normal preoperatively and now she has dropped her platelet count to 50,000/mm3. Her INR is 2.3 and  APTT is 90 (control 35). The best therapeutic option would be:
A) IVIg followed by steroids
B) Fondaparinux
C) Stop heparin and add aspirin to warfarin
D) Change to low molecular weight heparin

Answer: B

21A 59-year-old male presents with a 1 hour history of central crushing chest pain. He is known to be diabetic, hypertensive and is a non-smoker. On examination his pulse rate is 90 beats/min, blood pressure 130/85 mmHg, S1 S2 are audible with no murmurs. There is no evidence of cardiac failure. An ECG is performed. Which of the following would be an indication for thrombolysis?
A ) Right bundle branch block
B ) Supraventricular tachycardia
C ) ST elevation of 2mm in V4-V6
D ) ST depression of 2mm in leads II,III, avF
E ) Atrial fibirillation >150min-1

Answer: C

22A 46-year-old Caucasian woman complains of increasingly severe fatigue that she believe emanates from poor sleep quality. Her husband notes that she moves frequently during her sleep, and the patient notes that recently as she begins to ready herself for sleep she has discomfort in her legs that is momentarily relieved by moving them. Each of the following may be helpful in the treatment of this condition except?
A) Two month trial of ferrous sulfate
B) Stretching exercises before bedtime
C) A glass of red wine before bedtime
D) Gabapentin
E) Oxycodone 5–10 before bedtime

Answer: C

23. A 35-year-old woman complains of tingling and numbness of the little finger on the left hand. On examination, you corroborate hypesthesia of the 5th finger and also all of the ring finger, and you also find left-side weakness of flexion of fingers and wrist. The patient is able to spread the fingers adequately. Which of the following diagnoses is likely to account for these symptoms?
A) Ulnar nerve injury
B) Carpal tunnel syndrome
C) C6 nerve root compression
D) C7 nerve root compression
E) C8 nerve root compression
 

Answer: E

24Characteristic features of Mitral valve prolapse include
A) Early systolic murmur at the apex
B) Mid-systolic click
C) Valve Replacement gives better results than Valve repair
D) A poor prognosis 
E) Higher incidence in males

Answer: B

25A known case of chronic obstructive pulmonary disease presents to A & E, distressed and cyanosed. Arterial blood gases reveal a pH 7.2, PaO2 55 mmHg and PaCO2 60 mmHg. He is given high concentration oxygen together with a salbutamol nebulizer. Intravenous hydrocortisone is also given. The patient becomes even worse with poorer breathing effort although pulse oximetry showed SaO2 of 93%. What is the cause of patient deterioration?
A) Constriction of bronchioles in response to salbutamol nebulizer
B) High concentration oxygen administration
C) Pulmonary artery relaxation causing mismatch between perfusion and ventilation
D) Pulmonary vein relaxation causing mismatch between perfusion and ventilation
E) Reaction to IV hydrocortisone

Answer: B

26Which of the following is a recognised treatment for complications of cystic fibrosis?
A) DNAase to assist in reinflating collapsed lung segments
B) Hypotonic saline drinks for hypernatraemic dehydration
C) Nebulised tobramycin for pseudomonas colonisation of the lower respiratory tract
D) Pancreatic transplant for diabetes mellitus
E) Rectal pull-through and anastamosis for rectal prolapse

Answer: C

27A 45-year-old busy male attorney complains of headaches and generalized pruritus. He flew into an eastern American city from Denver, Colorado, to attend a high-level legal conference. He smokes 1 pack of cigarettes per day. He denies recent upper respiratory tract infection, foci of specific pain, and urinary and GI symptoms. On examination, he manifests a blood pressure of 160/105, a temperature of 98.6  F, a ruddy complexion, and splenomegaly and hepatomegaly. Blood gases are not immediately available. His CBC showed the hemoglobin level to be 18 g/dL, the white blood cell count to be 14,000, and the platelet count to be 7,00,000. Red blood cells are normochromic and normocytic. The BUN level is 18 mg/dL and the creatinine level is 1.1 mg/dL. Urinalysis is within normal limits with a specific gravity of 1.015. Which of the following is the diagnosis?
A) Polycythemia vera
B) Secondary polycythemia from altitude accommodation
C) Secondary polycythemia caused by chronic obstructive pulmonary disease
D) Spurious polycythemia from dehydration caused by air travel
E) Secondary polycythemia compensating for carboxyhemoglobin in a heavy smoker

Answer: A

28A 26-year-old man has noted painless swelling in the left side of his neck. Furthermore, he has noted afternoon chills and night sweats for several weeks. He has stopped smoking because inhaling tobacco smoke makes him ill. Although he seldom drinks alcohol, during the past week, he took a glass of wine and only then noted pain in the swelling about the neck. Examination is unremarkable except for the neck,
which exhibits a firm irregular mass measuring approximately 3 cm  5 cm located in the left anterior cervical region. The CBC and laboratory chemical battery are within normal limits. Which of the following is the most likely significant cause of these symptoms?
A) Hodgkin disease
B) Sialoadenitis
C) Viral respiratory tract infection
D) Streptococcal pharyngitis
E) Carotid artery aneurysm

Answer: A

29. A 36-year-old male assembly line worker complains that 2 days ago, while hefting a bucket of metal parts that slipped from his right hand while still grasping the other handle, experiences sudden pain in the left side of his neck that radiates into the left lateral upper arm into the thumb, associated with paresthesias in the thumb. Strength testing shows that his left wrist extension is 3/5. Which of the following is the nerve root involved? 
A) C5 
B) C6 
C) C7 
D) C8 
E) T1

Answer: B

30A 55-year-old man is scheduled for abdomen CT with contrast. He has type 2 diabetes; well controlled with pioglitazone, metformin, and glimepride. On the day of the procedure he is to receive nothing by mouth all morning except for medications until after the procedure. Which of the following would be the most appropriate oral regimen on the day of the procedure?
A) Continue all medications
B) Hold metformin, and continue glimepride and pioglitazone
C) Hold glimepride and metformin, and continue pioglitazone
D) Hold glimepride and pioglitazone and decrease metformin dose by half
E) Hold metformin and pioglitazone, and continue glimepride

Answer: C

31. A 65-year-old woman, has smoked 50 cigarettes a day for 40 years. She has had increasing dyspnoea for the several years, but no cough. A Chest X-ray shows increased lung size along with flattening of the diaphragms, consistent with emphysema. Over the next several years she develops worsening peripheral oedema. Her vital signs show T° 36.7 C, P 80, RR 15, and BP 120/80 mm Hg. Which of the following cardiac findings is most likely to be present?
A) Mitral valve stenosis
B) Constrictive pericarditis
C) Right ventricular hypertrophy
D) Left ventricular aneurysmm
E) Non-bacterial thrombotic endocarditis

Answer: C

32Which of the following is a typical feature of Farmer's lung?
A) basal crackles
B) Eosinophilia
C) Haemoptysis
D) Increased pCO2
E) Positive serum paraproteins

Answer: A

33. A 19-year-old female developed pleural effusions, ascites and ankle swelling. Her blood pressure was 112/76 mmHg.
Investigations revealed:
• serum alanine transferase 17 U/L (5 - 15)
• serum total bilirubin 17 umol/L (1 - 22)
• serum albumin 21 g/L (34 - 94)
• serum total cholesterol 9.8 mmol/L (<5.2)
What is the next most appropriate investigation?
A) Antinuclear antibody
B) Pregnancy test
C) Prothrombin time
D) Serum protein electrophoresis
E) Urinary protein estimation

Answer: E

34. The following are recognized features of Pancoast's tumour except:
A) ipsilateral Horner's syndrome
B) wasting of the dorsal interossei
C) pain in the arm radiating to the fourth and fifth fingers
D) erosion of the first rib
E) weakness of abduction at the shoulder

Answer: E

35.  Which of the following is a recognised cause of a phrenic nerve palsy?
A) Aortic aneurysm
B) Dermoid
C) Ganglioneuroma
D) Pericardial cyst
E) Sarcoidosis

Answer: A

36. A 58 year old woman falls and suffers a vertebral compression fracture. BMD confirms osteoporosis. Prior history of hysterectomy/oopherectomy at age 40 and prior DVT. Pain controlled with Tylenol. Best treatment:
A) Estrogen
B) Estrogen/progesterone
C) SERM
D) Bisphosphonate
E) Calcitonin

Answer: D

37. A 56-year-old man is admitted to the intensive care unit for acute respiratory distress syndrome (ARDS). The patient was transferred from an outside hospital today after a 2-week hospitalization for pneumonia. During that time, the patient's pulmonary status continued to deteriorate. One week ago he was intubated and placed on mechanical ventilation and over the past week, his oxygenation has worsened with a PaO2 of 66 on an inspired concentration of 100% oxygen. The patient has no other medical history except for rheumatoid arthritis. On transfer to the ICU, the patient is intubated and sedated on a mechanical ventilator. His chest radiograph shows patchy, bilateral, diffuse interstitial infiltrates. The most important intervention that will most benefit this patient is to
A) keep the patient in a prone position during mechanical ventilation
B) keep tidal volumes greater than 15 cc/kg
C) limit peak inspiratory pressure to 45 cm H2O or less
D) limit PEEP levels to less than 10 cm H2O
E) limit tidal volumes to 6cc/kg

Answer: E

38A 56-year-old woman presents with a 3-day history of fever, headache, fatigue, and myalgia, along with nausea and vomiting. She reports cough with minimal hemoptysis but denies abdominal pain and dysuria. About a week ago, she came back from a month-long missionary trip to a small village in Liberia. She had received appropriate pre-travel vaccines including the yellow fever vaccine. She was also compliant with her malaria prophylaxis as prescribed. On examination, she is tachypneic, tachycardic, and in mild respiratory distress. She has generalized petechiae but no other rashes or lymphadenopathy. What is the most likely infectious pathogen?
A) Plasmodium malariae
B) Salmonella typhi
C) Influenza virus
D) Ebola virus
E) Mycobacterium tuberculosis

Answer: D

39. In a 21 year old man with symptoms of chronic back pain, pain in his feet, particularly the great toe and metatarsophalangeal joints, and bilateral sacroiliitis on plain films, the most likely diagnosis is:
A) Ankylosing spondylitis
B) Gout
C) Inflammatory bowel disease-related arthropathy
D) Reiter’s syndrome
E) Psoriatic arthritis

Answer: D

40. A 49-year-old woman comes to the office because of difficulty breathing, fevers reaching 40 C (104 F), and a productive cough with blood tinged sputum. She was recently diagnosed with ductal carcinoma of the breast and underwent a radical mastectomy with four rounds of adjuvant chemotherapy with vinblastine and doxorubicin. Her last infusion of chemotherapy was 5 days ago. A chest radiograph shows focal infiltrates in both lungs. Laboratory studies show: You admit her to the hospital and start her on vancomycin and ceftazidime. Over the next 3 days, her clinical status continues to worsen. Blood cultures are negative. A bronchoscopy is performed and biopsy samples are obtained. The biopsy specimen shows septated, branching hyphae that are locally invading tissue. The most appropriate pharmacotherapy at this time is
A) amphotericin B, intravenously
B) fluconazole, intravenously
C) fluconazole, orally
D) itraconazole, intravenously
E) rifampin plus isoniazid, orally

Answer: A

41An 82-year-old woman with a history of Hashimoto's thyroiditis is evaluated for a rapidly expanding thyroid mass and progressive dysphagia and dyspnea. She has lost 2.2 kg over the preceding 6 weeks. O/E: BP118/78 mm Hg, PR 86/min. Thyroid examination reveals a firm 5-cm left thyroid mass that moves poorly with swallowing. Respiratory stridor is evident. Fine needle aspiration of the mass shows numerous uniform lymphocytes. CT scan of the neck shows the thyroid to completely encircle the trachea. 
Which of the following is the most likely diagnosis?
A) Medullary thyroid cancer 
B) Anaplastic thyroid cancer 
C) Thyroid hemorrhage 
D) Thyroid lymphoma 
E) Laryngeal cancer

Answer: D

42Thiazide diuretics can contribute to all of the following metabolic effects EXCEPT:
A) Hypomagnesemia
B) Hypourecemia
C) Hypercalcemia
D)Hypercholesrolemia
E) Hyponatremia

Answer: B

43A 55 year old women is referred for evaluation of hypercalcemia. Three years ago, she had a left mastectomy for breast carcinoma, after which she underwent treatment with radiation and adjuvant chemotherapy because 2 of 12 lymph nodes were found to be positive for carcinoma. She has been clinically well since then, but on a routine postoperative check-up 3 months ago, she was found to have a serum calcium level of 2.75 mmol/L (N:2.2-2.6 mmol/L). On repeat testing, the serum calcium level is 2.9 mmol/L
The best first diagnostic step to identify the cause of this patient’s hypercalcemia is:
A) serum alkaline phosphatase level
B) serum parathyroid hormone level
C) serum 1,25- dihydroxyvitamin D level
D) Serum 25 hydroxyvitamin D level
E) Bone Scan.

Answer: B

44All of the following bedside maneuvers are useful to distinguish the murmur of mitral valve prolapse from other valvular lesions EXCEPT ?
A) Isometric hangrip
B) Standing from a supine position
C) Carotid sinus massage
D) Valsalva maneuver
E) Squating from standing position

Answer: C

45. One of your 52-year-old female patients with an eGFR of 12 mL/min comes to your office with complaints of fatigue and poor appetite due to metallic taste in the mouth for the last 2 years. She has no chest pain or hyperkalemia. Her BP is 130/80 mmHg. Her serum [HCO3] is 20 mEq/L. During her previous visit 3 months ago, you discussed about the choice of her future renal replacement therapies, including transplantation. After prolonged discussion, she feels that PD is a reasonable choice in view of her daily work in the school. Based on her clinical history, which one of the following choices is MOST appropriate regarding the management of her symptoms and selection of dialysis modality is
CORRECT?
A) Admit to hospital and start HD with a central vein catheter (CVC), and then start PD
B) Tell her that she would receive in-center HD once acute HD treatment is over
C) Admit to hospital and place a PD catheter, and then train her for continuous ambulatory PD with small volume exchanges initially
D) Change her diet and increase her NaHCO3 to 1350 mg every 8 h
E) Tell her that she may not need any renal replacement therapy (either HD or PD) until her eGFR falls to 7 mL/min

Answer: C

46. Each of the following combinations has the potential for significant pharmacological interaction and drug toxicity except
A) Simvastatin and Erythromycin
B) Sildenafil and Nitroglycerin
C) Pravastatin and Ketconazole
D) Cyclosporine and St. johns wort
E) Digoxin and Verapamil

Answer: C

47A 40 year old man with recurrent severe headaches. They appeared to occur episodically every 3-6 months, with headaches occuring daily for up to 8 weeks at a time. They often woke him from sleep at 2 am, with sever right peri-oricular pain and would typically last for 30-60 mins. He also described right nasal congestion and lacrimation but no vomiting or photophobia. Neurological exam., MRI brain were normal. What is the most appropriate prophylactic treatment:
A) Carbamazepine
B) Erogotamine
C) Propranolol
D) Sumatriptan
E) Verapamil

Answer: E

48A 57-year-old female school cleaner is undergoing investigation for breathlessness. All the following would be in keeping with a diagnosis of constrictive pericarditis except:
A) Ascites 
B) Elevated JVP with absent y descent 
C) Orthopnoea 
D) Peripheral oedema 
E) Previous cardiac surgery

Answer: B

49A 45-year-old woman undergoes upper endoscopy for symptoms of dyspepsia. The endoscopic examination is normal. Physical examination, routine laboratory studies, serologic tests forHelicobacter pylori, and abdominal ultrasonography are also normal. The patient has not received any medications for her symptoms. Which of the following is the most appropriate initial empiric therapy for this patient? 
A) A proton pump inhibitor 
B) Alosetron 
C) Ondansetron 
D) Tegaserod 
E) Sumatriptan

Answer: A

50By definition, an episode of pancreatitis is considered chronic pancreatitis if
A) It is a recurrent episode.
B) It occurs in the face of alcohol use.
C) There are radiographic findings of ductal irregularity and parenchymal fibrosis.
D) It is associated with steatorrhea.
E) A pseudocyst is present.

Answer: C

MULTIPLE CHOICE QUESTIONS-7

1A 37-year-old woman with a history of intravenous drug use, hepatitis B, asthma, and acquired immunodeficiency syndrome (AIDS) is admitted to the hospital because of fever, night sweats, and malaise. Her last CD4 count was 1 month ago and measured 180/mm3. Vital signs are: temperature 38.5 C (101.3 F), blood pressure 145/76 mm Hg, and pulse 90/min. Physical examination is significant for a soft diastolic murmur heard best at the lower left sternal border. Auscultation of the lungs reveals diffuse rhonchi. The abdominal and neurologic exams are unremarkable. The next step in managing this patient is 
A) analysis and culture of spinal fluid
B) a blood culture
C) a CT of the head
D) a urinalysis
E) an x-ray of the chest

Answer: E

2. An 18-year-old man presents with pain in the right side of the neck, veering to the right, and numbness on the right side of the face and in the left arm and leg that came on acutely with lifting heavy weights. His examination shows right hemiataxia, right miosis and ptosis, poor palatal elevation on the right, and decreased sensation to pinprick of the right side of the face and the left arm and leg. To which of the following structures does the lesion best localize?
A) Right lateral medulla
B) Left lateral medulla
C) Right cerebellum
D) Left midbrain
E) Central pons

Answer: A

3. A 78-year-old woman is admitted to the hospital because of a fever, productive cough, and a chest x-ray demonstrating right lower lobe consolidation. Her past medical history is significant for seasonal allergies. She has been taking estrogen/progesterone replacement since menopause 19 years ago and occasional acetaminophen for headaches. The patient lives alone at her home and she does not drink alcohol or smoke. Review of systems is significant for weakness attributed to "old age". On the day prior to discharge, a repeat chest x-ray shows the pneumonia to be resolving. An incidental note is made of severe osteoporosis involving all of the bones visualized on the film. Vital signs are temperature 38.8 C (101.8 F), blood pressure 100/50 mm Hg, pulse 90/min, and respirations 10/min. Physical examination is significant only for decreased breath sounds at the right lung base. The patient is neurologically intact and wants to return home. Laboratory studies show a leukocyte count 15,000/mm3, hematocrit 28%, and platelets 150,000 mm3. The next step in the management of this patient is to
A) discharge her and do a bone marrow biopsy as an outpatient
B) discharge her and send her for a bone scan as an outpatient
C) discharge her and order serum protein electrophoresis as an outpatient
D) do a bone marrow biopsy before discharge
E) order a bone scan and serum protein electrophoresis before discharge

Answer: C

4Eosinophilia is frequently seen with fever of unknown origin from all but 1 of the following illnesses. Which illness is not frequently associated with eosinophilia?
A) Systemic lupus erythematosus (SLE)
B) Drug fever
C) Tuberculosis
D) Myeloproliferative disease
E) Polyarteritis nodosa (PAN)

Answer: C

5. In a patient with a known Chiari I malformation, burning pain develops in both shoulders. On examination, strength is normal in the limbs, but the sensation to pinprick is reduced in a capelike distribution around the shoulders extending to the upper part of the arms. To what structure does the lesion best localize?
A) Bilateral dorsal columns
B) Right lateral funiculus
C) Central spinal cord
D) Bilateral ventral funiculus
E) Bilateral ventral horns

Answer: C

6A 33-year-old asymptomatic woman was referred because of abnormal ALT. History of migraine headache for which she takes Sumatriptan. No risk factors or history of liver disease. Examination is notable for obesity.AST 60, ALT 75, ALP , bilirubin , INR, iron studies and albumin are normal. What would be the most appropriate next step ?
A) Observe and repeat ALT in 3 months 
B) Ultrasound abdomen
C) Check viral markers 
D) Liver biopsy
E) Stop Sumatriptan

Answer: A

7. A 63-year-old woman with 4 hour history of abdominal pain , fever and nausea. On examination , patient has fever, jaundice and mild epigastric tenderness . TC 18000, with shift to left, bilirubin elevated, ALP 150,(normal upto 125) AST 745 IU, ALT 650 IU, USG multiple small stones in gall bladder, no bile duct dilatation , normal pancreas. Started on antibiotics, but following day still has fever. Repeat labs bilirubin elevated, AST 284 IU, ALT 200 IU, WBC 25000, blood culture positive for E.Coli. which of the following would you advice next ?
A) Doppler of hepatic vessels 
B) Lab cholecystectomy
C) MRCP
D) Endoscopic ultrasound
E) ERCP

Answer: E

8. A 18-year-old woman presents with acute onset of jaundice and somnolence. On examination , jaundiced , sleepy but arousable. Labs INR 1.6, AST 240, ALT 210, total bilirubin increased , mostly in direct, hemoglobin 9.4, ceruloplasmin 8ng/L(normal more than 22), 24 hr urine copper 563mcg/L (normal less than 60).which of the following would you advise now ?
A) Trientine 
B) Penicillamine
C) Urgent liver transplantation 
D) Intracranial pressure monitoring 
E) Liver biopsy

Answer: C

9. Warfarin is administered to a 56-year-old man following placement of a prosthetic cardiac valve. The warfarin dosage is adjusted to maintain an INR of 2.5. Subsequently, trimethoprim-sulfamethoxazole therapy is begun for a recurring urinary tract infection. In addition to monitoring prothrombin time, which of the following actions should the physician take to maintain adequate anticoagulation? 
A) Begin therapy with vitamin K 
B) Increase the dosage of warfarin 
C) Make no alterations in the dosage of warfarin 
D) Decrease the dosage of warfarin 
E) Stop the warfarin and change to low dose aspirin

Answer: D

10You are called emergently to the medical floor where a 66-year-old man was found to be minimally responsive. His past medical history is unclear but his arm band lists allergies to penicillin and sulfa medications. On arrival, chest compressions are being performed and 2 operators are mask ventilating the patient. Evaluation with an electrocardiogram reveals sinus tachycardia and the diagnosis of pulseless electrical activity is made. Volume is infused and compressions are continued. The patient remains apneic, so mask ventilation continues. During masking, the patient appears to regurgitate large volumes of gastric contents. The most appropriate immediate next step in the management of this patient is to
A) cease mask ventilation and suction the mouth
B) continue masking the patient
C) insert a nasogastric tube
D) intubate the trachea and suction the airway
E) intubate the trachea and ventilate

Answer: D

11Which statement about evaluating a patient with fever of unknown origin (FUO) is true?
A) Bone marrow cultures have a high yield when the fever is greater than 39 ° C and other testing has been unrevealing.
B) A temporal artery biopsy is a reasonable test to perform next for a 75-year-old man who has FUO and no localizing complaints, an erythrocyte sedimentation rate greater than 100 mm/h, mild anemia, normal blood chemistry results, and negative blood cultures at 3 days and who has not had any other evaluation.
C) Lumbar puncture should be done on every patient with FUO.
D) When there are no clues to the underlying diagnosis, extensive serologic testing for unusual infections is often helpful.
E) Liver biopsy is unlikely to be helpful in a patient with FUO and miliary tuberculosis

Answer: B

12A 31-year-old man describes a burning sensation in the substernal area after eating chocolates, caffeine, or alcohol. The symptoms are exacerbated at night, and he has woken on several occasions from sleep because of coughing. He has tried over-the-counter antacids and H2 receptor antagonists with little relief. He often takes antacids before dinner. Which of the following is the most likely explanation for his symptoms of coughing?
A) Acid-induced bronchoconstriction 
B) Aspiration of antacids taken before bedtime 
C) Aspiration of solid food ingested with dinner 
D) H2 receptor antagonist-induced bronchoconstriction 
E) Tracheoesophageal fistula

Answer: A

13A 24-year-old lady at 10 weeks of gestation during her first pregnancy. She has had 2 days of nausea and mild fever. She vomited once this morning. On examination jaundiced, with tender , mild hepatomegaly. Labs AST 56) IU, ALT 1150 IU, bilirubin increased, platelets 140. most likely diagnosis 
A) Hyperemesis gravidarum
B) Acute viral hepatitis 
C) Acute cholecystitis
D) Intra hepatic cholestasis of pregnancy
E) Fatty liver of pregnancy

Answer: B

14. A 36-year-old male with alcoholic hepatitis. Which clinical or lab parameter is least predictive of patient survival ?
A) Bilirubin 
B) Prothrombin time 
C) AST/ALT ratio
D) Encephalopathy
E) Creatinine

Answer: C

15A 64-year-old male executive with remote history of heavy alcohol use is noted to have cirrhosis on liver biopsy. There are no varices detected on upper GI Endoscopy. When would you recommend that the patient return for a repeat upper Endoscopy to screen for esophageal varices ?
A) 6 months 
B) 12 months 
C) 18 months 
D) 24 months 
E) Never

Answer: D

16Which of the following statements about pulmonary function during pregnancy is true?
A) Total lung capacity is reduced.
B) Functional residual capacity is reduced.
C) FEV1 /FVC ratio is reduced.
D) Lung compliance is reduced.
E) The single breath diffusing capacity for carbon monoxide is reduced

Answer: B

17. Which one of the following is the commonest cause of death in SLE?
A) Infection
B) Cardiovascular disease
C) Lupus nephritis
D) Cerebral lupus
E) Thromboembolism

Answer: A

18Each of the following is a solid indication for neuroimaging in a patient with headache EXCEPT: 
A) Onset of headaches over the age of 50 years 
B) Seizures associated 
C) Prolonged aura 
D) Nausea and vomiting 
E) Headache worsening with movement

Answer: D

19A lady is newly diagnosed with osteoporosis. She has never been on treatment. Which one of the following is the first-line treatment?
A) Alendronic acid
B) Etidronate
C) Risedronate
D)  Strontium ranelate
E) Zoledronate

Answer: A

20A 48 year-old lady is referred to the rheumatology outpatient clinic with a 4-month history of fatigue, aches and weakness of her thighs.She has a past medical history of hypercholesterolaemia and is on simvastatin. On examination she has normal muscle bulk. Active hip flexion was reduced. Neurological examination was otherwise normal.
Investigations:
Hb 98 g/L (115–165)
WCC 12.0 × 109 /L (4.0–11.0)
Platelets 400 × 109/L (150–400)
Plasma viscosity 2.0 (1.5–1.72)
Creatine kinase 15000 IU/L
The diagnosis is likely to be which one of the following?
A) Polymyalgia rheumatica
B) Statin-induced myopathy
C) Polymyositis
D) Dermatomyositis
E) Hip osteoarthritis

Answer: C

21A 78-year-old man has collapsed several hours ago and been found on the floor by the paramedics. He was found to be hypothermic at 32°C. His creatinine kinase is 10,000 IU. His serum potassium is 5mmol/L, urea is 20mmol/L and creatinine is 300μmol/L. The most important form of initial treatment should be:
A) Intravenous infusion of warmed crystalloid.
B) Active warming.
C) Intravenous administration of mannitol 0.25-0.5g/kg.
D) Intravenous furosemide.
E) Urgent haemodialysis

Answer: A

22A 25-year-old woman with a history of epilepsy presents to the emergency room with impaired attention and unsteadiness of gait. Her phenytoin level is 37. She has white blood cells in her urine and has a mildly elevated TSH. Examination of the eyes would be most likely to show which of the following?
A) Weakness of abduction of the left eye
B) Lateral beating movements of the eyes
C) Impaired convergence
D) Papilledema
E) Impaired upward gaze

Answer: B

23A 35-year-old female is referred for evaluation of positive antinuclear antibodies (ANA). She is asymptomatic. Her lab work reveals ANA 1:320 homogeneous pattern; extractable nuclear antigens are negative. Hematologic and renal function values are normal and urinalysis is without sediment. Her examination is unremarkable apart from a smooth nontender goiter. The past medical history is significant for Hashimoto’s thyroiditis. No rash, synovitis, or serositis is observed. What further treatment or investigations are warranted?
A) Start prednisolone 40 mg daily
B) Start prednisolone 20 mg daily
C) Repeat ANA test
D) Start hydroxychloroquine
E) No further intervention is warranted

Answer: E

24A 41-year-old man is diagnosed with iron deficiency anemia and is found to have heme-positive stools. Colonoscopy reveals a large ulcerated tumor in his transverse colon. He also has two smaller polyps in his ascending colon. Pathologic examination of the tumor biopsy reveals adenocarcinoma, while biopsies of the polyps confirm that these are adenomas. His sister has been diagnosed with uterine cancer, and two cousins have died of colon cancer. All of the following are true statements about this case except
A) Referral for genetic counseling is indicated
B) He is at increased risk for other epithelial-derived tumors
C) He likely has familial adenomatous polyposis (FAP), with a germ line mutation in the APC gene
D) His condition is often associated with a defect in DNA mismatch repair

Answer: C

25A 60-year-old man presents with abdominal pain and a cupful of haematemesis. On examination he is noted to have ascites, hepatomegaly and a very enlarged spleen extending to the right iliac fossa. His initial blood tests reveal a leukoerythroblastic picture with a haemoglobin of 8, white cell count (WCC) of 3 and platelets of 120. A diagnosis of myelofibrosis is made. What is most likely to be seen on the peripheral blood smear?
A) Schistocytosis
B) Sickle cells
C) Spherocytes
D) Dacrocytes
E) Target cells

Answer: D

26All the following conditions are associated with high output heart failure EXCEPT
A) Iron overload
B) Hyperthyroidism
C) Systemic arteriovenous fistula
D) Thiamine deficiency
E) Paget disease

Answer: A

27Primary prophylaxis to prevent acute infection in susceptible patients is recommended for all the following HIV Infected patients EXCEPT
A) Pneumocystis jiroveci (formerly carinii) pneumonia.
B) Disseminated Mycobacterium avium complex.
C) Mucocutaneous candidiasis.
D) Malaria for persons traveling to areas where malaria is endemic.
E) Toxoplasma gondii encephalitis

Answer: C

28. A 30-year-old male with Alports and gradually worsening renal dysfunction has reached ESRF and is on hemodialysis. His family is being evaluated for donation for renal transplant. His 57 year old mother and one aunt have microscopic hematuria but normal renal functions. Two of his uncles also have ESRD and are on dialysis. He has a 40 brother with normal urine analysis, a 38 year old sister with normal urine analysis and a 25 year old sister with microscopic hematuria, all with normal renal function.
Which of the following can most safely donate kidney for this patient?
A) Brother as he has extremely low probability of having Alports.
B) Sister with normal urine analysis, as she has no probability of having Alports.
C) Sister with microscopic hematuria as she is younger.
D) Mother 
E) None of family members should be taken as donor

Answer: B

29A 48-year old female with rheumatoid arthritis presents to the emergency department with 2-week pain and tightness behind the left knee. Examination reveals cystic swelling over the left popliteal fossa. Which of the following is the most appropriate next action?
A) Arthrogram of the left knee
B) Synovial biopsy of left knee
C) Ultrasound study of left knee and popliteal fossa
D) Venogram of left lower limb
E) None of the above

Answer: C

30A 37-year-old woman is admitted to accident and emergency with severe facial burns. Despite prompt management, she develops acute respiratory distress syndrome (ARDS). Which of the following is not associated with the diagnostic criteria for ARDS?
A) Bilateral infiltrates on chest x-ray
B) Acute onset
C) Pulmonary capillary wedge pressure >19
D) Refractory hypoxemia (PaO2:FiO2 <200)
E) Lack of clinical congestive heart failure

Answer: C

31.  A 55-year-old man with chronic hepatitis C is being considered for liver transplantation. The patient has cirrhosis that was documented by liver biopsy 10 years ago. For the past 3 months, he has had ascites and edema, which are poorly controlled with diuretics. Lactulose was recently begun because of confusion. Which of the following combinations of laboratory studies will be most helpful in estimating his survival over the next 6 months? 
A) Serum total bilirubin and INR 
B) Serum aspartate aminotransferase and gamma globulin 
C) Serum alanine aminotransferase and hepatitis C RNA (HCV RNA) 
D) Serum alkaline phosphatase and ammonia 
E) Serum albumin and ?-glutamyltransferase

Answer: A

32.  A 67-year-old African American man with bronchogenic carcinoma returns to the office for follow-up of confusion and lethargy that have been gradually increasing during the past 3 weeks. CT scan of the head 4 weeks ago showed no metastases. Current medications include inhaled bronchodilator medication and oxycodone for pain. The patient is 185 cm (6 ft 1 in) tall and weighs 61 kg (135 lb); BMI is 18 kg/m2. Vital signs are normal. Physical examination shows generalized muscle wasting. Auscultation of the lungs discloses scattered rhonchi in all fields and expiratory wheezes. Mini- Mental State Examination score is 24/30. Results of laboratory studies are shown:
Serum Na+ 125 mEq/L
K+ 3.2 mEq/L
Cl− 100 mEq/L
HCO3 − 25 mEq/L
Blood
Hematocrit 32.2%
Hemoglobin 11.2 g/dL
Which of the following is the most appropriate study to order at this time?
A) 24-Hour urine collection for creatinine clearance
B) Determination of AM serum cortisol concentration
C) Determination of serum iron concentration
D) Determination of serum magnesium concentration
E) Determination of urine sodium concentration

Answer: B

33Which of the following β-adrenergic antagonists is a nonselective β1 and β2 blocker?
A) Atenolol
B) Betaxolol
C) Esmolol
D) Metoprolol
E) Nadolol

Answer: E

34. Signs and symptoms of opioid withdrawal include all of the following EXCEPT
A) Increased blood pressure (BP) and heart rate
B) Seizures
C) Abdominal cramps
D) Jerking of the legs
E) Hyperthermia

Answer: B

35A 45-year-old man is evaluated for heartburn with water brash, usually occurring at bedtime. He notes worsening of symptoms when lifting weights, but he is asymptomatic during his routine 5-km run. He has not had dysphagia, bleeding, vomiting, or weight loss. He has been taking nonprescription omeprazole once daily for the past two weeks. BMI is 32. Vital signs and physical examination are normal. Complete blood count, serum electrolytes, and electrocardiogram are normal.
Which of the following should you recommend?
A) An increase of omeprazole dosage to twice daily
B) Exercise stress test
C) Computed tomography of the abdomen
D) Ultrasonography of the abdomen
E) Esophagogastroduodenoscopy

Answer: A

36A pregnant woman is being followed for gestational diabetes. She walks 30 minutes per day and is following a diabetic diet. She is not gaining weight. Fasting sugars are 5 - 6.5; postprandial, 7 - 8.5. She now has ketonuria in the mornings. What should you advise?
A) Increase caloric intake, plus begin insulin NPH 10u QHS
B) Increase caloric intake, plus begin insulin lispro ac meals
C) Increase caloric intake and exercise before breakfast 
D) Increase calorie intake and postprandial exercise
E) Increase calorie intake, plus an oral agent

Answer: B

37A 60-year-old man is admitted to the hospital for management of acute pancreatitis. Results of laboratory studies are shown: Serum Blood Amylase 1000 U/L Hematocrit 42%; Calcium 8.4 mg/dL WBC 14,000/mm3; Urea nitrogen 5 mg/dL. Results of serum liver chemistry profile are within the reference ranges. After 48 hours of fluid therapy and observation, a poor prognosis would be indicated by which of the following laboratory study results?
A) Serum alanine aminotransferase (ALT) concentration of 106 U/L
B) Serum amylase concentration of 2000 U/L
C) Serum bilirubin concentration of 4.2 mg/dL
D) Serum calcium concentration of 6.6 mg/dL
E) Serum glucose concentration of 200 mg/dL

Answer: D

38Linezolid has the following characteristics as compared to vancomycin:
A) Decreased nephrotoxicity at higher doses
B) Increased intrapulmonary penetration
C) Increased incidence of thrombocytopenia
D) All of the above
E) None of the above

Answer: D

39A 19-year-old female developed pleural effusions, ascites and ankle swelling. Her blood pressure was 112/76 mmHg.
Investigations revealed:
• serum alanine transferase 17 U/L (5 - 15)
• serum total bilirubin 17 umol/L (1 - 22)
• serum albumin 21 g/L (34 - 94)
• serum total cholesterol 9.8 mmol/L (<5.2)
What is the next most appropriate investigation?
A) Antinuclear antibody
B) Pregnancy test
C) Prothrombin time
D) Serum protein electrophoresis
E) Urinary protein estimation

Answer: E

40The following are recognized features of Pancoast's tumour except:
A) ipsilateral Horner's syndrome
B) wasting of the dorsal interossei
C) pain in the arm radiating to the fourth and fifth fingers
D) erosion of the first rib
E) weakness of abduction at the shoulder

Answe: E

41A 43-year-old woman has the following laboratory test results: serum bilirubin, total 4.6 mg/dL and direct 0.3 mg/dL; serum alkaline phosphatase, 108 U/L; serum aminotransaminase, aspartate aminotransferase (AST) 18 U/L and alanine aminotransferase (ALT) 22 U/L. What is the most likely
diagnosis?
A) Chronic hepatitis C virus infection
B) Dubin-Johnson syndrome
C) Hemolytic anemia
D) Primary biliary cirrhosis
E) Primary sclerosing cholangitis

Answer: C

42A 52 year old woman referred because of palpitation, weight loss and increased sweating. 2 years back she was admitted to CCU because of Acute MI complicated by ventricular tachycardia and was discharged on amiodarone. Her TFT revealed TSH 0.01, FT4 > 100. RAIU showed increased uptake and U/S thyroid showed increased vascularity. What is the next immediate step:
A) Stop amiodarone
B) Start carbimazole
C) Start steroids
D) Give appointment with cardiologist to change amiodarone
E) Observe and reassure the patient

Answer: B

43You are asked to see a woman recently admitted to the ICU with hypotension. She is unable to give a history, and no family members or friends are available. Witnesses report that she was sitting at the airport waiting for a connecting flight when she passed out. The person sitting next to her noticed that she became quite agitated when it was announced that the flight would be delayed for 6 hours, just before she passed out. There is no evidence of a pulmonary embolism. During your examination, you note that she is very thin, lacks axillary hair, has sparse pubic hair, and has extra pigmentation on her gums and buccal mucosa. The blood pressure was 40/palp with a heart rate of 130 when she first arrived, but is now 100/64 with a heart rate of 78 with saline running. Her temperature is 98.2° F; the cardiac rhythm is sinus; the routine chemistry tests show a hyperkalemic metabolic acidosis; and the renal status is compatible with prerenal azotemia.
Which of the following should you do now?
A) Begin hydrocortisone 100 mg IV every 8 hours and order an ACTH stimulation test.
B) Measure cortisol and FT4 levels before beginning thyroxine and methylprednisolone.
C) Begin hydrocortisone 100 mg IV every 8 hours.
D) Begin dexamethasone 4 mg IV and order an ACTH stimulation test.
E) Measure ACTH and TSH before beginning prednisone 60 mg and thyroxine 100 μg every day.

Answer: D

44A 45-year-old woman is hospitalized for a cholecystectomy. She appears to be anxious and worried about the surgery. Which type or types of information provided to the patient will be most effective in reducing stress?
A) Sensory information
B) Procedural information
C) Coping information
D) Sensory and procedural information
E) Sensory and coping information

Answer: E

45A 60-year-old woman was hospitalized with a severe respiratory infection for several weeks. Afterward, she displayed symptoms of myalgia and weakness of the lower limbs. In addition, she also showed loss of muscle tone and some flaccidity with loss of tendon reflexes. Examination also revealed a weakness of facial muscles. This constellation of symptoms
progressed for approximately 2 weeks and persisted for more than a year, at which time, recovery took place at a slow rate. There was also some demyelination coupled with lymphatic inflammation at the site of demyelination. The most likely cause of this patient’s condition is
A) Myasthenia gravis
B) Muscular dystrophy (MD)
C) Multiple sclerosis (MS)
D) Guillain-Barré syndrome
E) Lumbar disk prolapse

Answer: D

46A 40-year-old patient with a recent viral infection presents with a significantly tender gland, low radioiodine uptake, and signs and symptoms of thyrotoxicosis. This presentation is most likely
A) Graves’ disease
B) Subacute thyroiditis
C) Toxic multinodular goiter
D) Hashimoto’s thyroiditis
E) Toxic adenoma

Answer: B

47A 30-year-old male presents with multiple soft, raised, beefy-red superficial ulcers in his left groin. Physical examination reveals several enlarged left inguinal lymph nodes. A histologic section from an enlarged lymph node that is stained with a silver stain reveals characteristic Donovan bodies within macrophages. What is the most likely diagnosis?
A) Chancroid
B) Gonorrhea
C) Granuloma inguinale
D) Lymphogranuloma venereum
E) Syphilis

48A 55-year-old woman comes to the emergency department because of abdominal pain. She had just finished eating a steak dinner with her family when she suddenly experienced sharp, crampy pain in the upper right and middle of her abdomen. The pain has lasted for the past 3 hours and she is starting to feel nauseous. On physical examination, she is obese and in obvious discomfort. Her temperature is 38.8 C (101.8 F), blood pressure is 140/87 mm Hg, pulse is 90/min, and respirations are 16/min. Abdominal examination is significant for focal tenderness and guarding in her right upper quadrant. She is particularly tender when you palpate her right upper quadrant as she takes in a deep breath. The most appropriate next step in the evaluation of her abdominal pain is
A) an abdominal x-ray
B) a CT of the abdomen
C) an endoscopic retrograde cholangiopancreatography (ERCP)
D) serum liver function tests including bilirubin
E) an ultrasonography of the abdomen

Answer: E

49A 32-year-old female smoker presents with acute severe asthma. The SaO2 are 80% on 15 L of oxygen and the pO2 is 8.2kPa (10.5-13). There is widespread expiratory wheeze throughout the chest. She is given IV hydrocortisone, 100% oxygen and 5 mg of nebulised salbutamol.
What is the next step in your management
A) IV augmentin 
B) IV magnesium 
C) IV potassium 
D) IV theophylline 
E) Oral prednisolone

Answer: B

50A previously healthy16 year old girl brought by her parents to ER because of repeated vomiting, lethargy, breathlessness and confusion of 1 day duration. She developed 2 attacks of tonic clonic seizures up on arrival to ER. Physical examination was remarkable for confusion, tachypnea and temp. of 38.5 C. CT brain, CXR and CSF analysis were normal. Blood biochemistry revealed Na 132 mmol/L, K 3.0 mmol/l, BUN 12 mmol/l, Creatinine 74 mmol/l and HCO3 12 mmol/L. ABG showed ph 7.5 , PaCO2 16 mmHg, PaO2 120 mmHg, HCO3 13 mmol/L. What is the best management option:
A) start dexamethasone, ceftriaxone and vancomycin
B) Start dexametahsone, ceftriaxone, vancomycin and oseltamivir
C) Start N-acetylcysteine infusion
D) Urinary alkalinization followed by hemodialysis 
E) Treat with septrin and request HIV testing

Answer: D

 

MULTIPLE CHOICE QUESTIONS-8

1. A 24-year-old Asian man presents to his family physician with one episode of painless gross hematuria. He is otherwise healthy with no significant family history. On physical examination, his blood pressure is normal and there are no other abnormalities. Laboratory examination shows a normal metabolic profile, hemogram, liver enzymes, and renal function tests, as well as a negative urine culture. Serologic tests including complement evels, antinuclear antibody, antineutrophil cytoplasmic antibody, and a hepatitis panel are all negative. Urine examinations show 3+ blood, 3+ protein, and many RBCs and RBC casts on microscopic examination. His renal ultrasound is normal. A repeat physical examination and laboratory tests after 1 week show 1+ heme, 5 to 10 RBCs, and trace urinary protein, which quantitates to 150 mg/day. Your recommendation at this point should be:
A) Continue observation.
B) Renal biopsy.
C) Empiric therapy with alternate day steroids.
D) Genetic testing for patient and his family.
E) Urology referral

Answer: A

2. A 69-year-old man presents with confusion. His carers state that over the last month he has become increasingly lethargic, irritable and confused. Despite maintaining a good appetite, he has lost 10 kg in the last month. Blood results are as follows:
Sodium 125 mmol/L
Potassium 4 mmol/L
Urea 3
Glucose (fasting) 6 mmol/L
Urine osmolality 343 mmol/L
The most likely diagnosis is:
A) Hypothyroidism
B) Dilutional hyponatraemia
C) Addison’s disease
D) Acute tubulointerstitial nephritis
E) Syndrome of inappropriate anti-diuretic hormone (SIADH)

Answer: E

3. You see a 19-year-old Caucasian man in your clinic who presents with a history of transient jaundice. On direct questioning, you ascertain that the jaundice is noticeable after periods of increased physical activity and subsides after a few days. The patient has no other symptoms and physical examination is unremarkable. Full blood count is normal (with a normal reticulocyte count) and liver function tests reveal a bilirubin of 37 μmol/L. The most appropriate management is:
A) Reassure and discharge
B) Start on a course of oral steroids
C) Request abdominal ultrasound
D) Request MRCP
E) Refer to Hematology

Answer: A

4. A 60-year-old man presented with epigastric and right hypochondrial pain associated with nausea, vomiting and fever. He consumed alcohol almost daily for the last 30 years. He recalled that a surgeon had previously suggested a gall bladder surgery but he did refused to have it. On exam: temp. 38°C, B.P. 120/70, pulse 100/min. Sclerae were yellow. He had eight spider nevi on his upper body. Chest and heart exam were normal. Abdomen was tender in the right upper quadrant but no mass was palpable. There was no detectable ascites and rectal examination was unremarkable. Hemoglobin 14.2, WBC count 17,000, Platelets 110,000, Serum albumin 30 g/L, total bilirubin 70 (1-22), ALT 70 (< 40), alkaline phosphatase 500 U/L (45-105), amylase 100 U/L (60-180), INR 1.5 What is the likely diagnosis?
A) Cholangitis
B) Acute on chronic pancreatitis
C) Alcoholic hepatitis
D) Acute cholecystitis
E) Empyema of the gall bladder

Answer: A

5A 45-year-old man collapses at home and is brought to accident and emergency. He has a fever at 39.5°C and blood pressure is 90/60 mmHg, although he is in a lucid state. Bruises can be seen on his skin which he remembers being present before he fell. Blood tests show the patient to have a normocytic anaemia with a low platelet count and increased fibrin split products. The most likely diagnosis is:
A) Warm autoimmune haemolytic anaemia
B) Cold autoimmune haemolytic anaemia
C) Paroxysmal nocturnal haemoglobinuria
D) Disseminated intravascular coagulation
E) Thalassaemia minor

Answer: D

6A 40-year-old man with cryptogenic liver cirrhosis presents with acute onset of confusion. His wife reports that over the last few days his abdomen has become increasingly distended, however she is more concerned about him being disoriented to time and place. He takes spironolactone 100 mg daily, furosemide 40 mg bid and lactulose 10 mg bid. Exam: He is confused to time and place and is afebrile. Icteric sclerae , spider nevi and asterixis . There is tense ascites with distended abdominal veins is also noted. WBC count 10,200, Hemoglobin 10.5, Platelets 35,000, INR 1.7, Serum sodium 128, Serum creatinine 90, ALT 60 What should be the next step?
A) Abdominal paracentesis 
B) CT-scan of the brain
C) Start IV cefotaxime 
D) Start oral neomycin 
E) Increase dose of diuretics

Answer: A

7A 62-year-old woman presents to accident and emergency with a 1-day history of sudden onset back pain and difficulty walking. She has not opened her bowels or passed urine for the previous day. She has a past medical history of breast cancer, diagnosed two years earlier and staged as T2N1M0 disease with oestrogen receptor positive status. She has been treated for her cancer with a wide local excision and axillary node clearance, followed by radiotherapy, chemotherapy and tamoxifen. On examination, there is reduced tone in the lower limbs. Power is diminished throughout the lower limbs, but especially on hip flexion. There is reduced sensation below the L1 dermatome. What is the most appropriate diagnostic investigation?
A) A full set of bloods, including bone profile
B) Computed tomography (CT) thorax, abdomen and pelvis
C) Magnetic resonance imaging (MRI) spine
D) Bone scan
E) Positron emission tomography (PET) CT

Answer: C

8A 35-year-old woman comes to your clinic complaining of shortness of breath. It is immediately apparent that she has a bluish tinge of her face, trunk, extremities, and mucus membranes. Which of the following is most likely?
A) Atrial septal defect
B) Myocarditis
C) Raynaud’s phenomenon
D) Vasospasm due to cold temperature
E) Connective tissue disease

Answer: A

9A 50-year-old woman, who has received a recent diagnosis of rheumatoid arthritis, presents to her GP with ongoing pain and stiffness in her hands and feet. Which joints are usually spared at onset of rheumatoid arthritis?
A) Proximal interphalangeal joints
B) Distal interphalangeal joints
C) Metacarpophalangeal joints
D) Wrists
E) Metatarsophalangeal joints

Answer: B

10A 45-year-old woman with unexpected weight loss, loss of appetite and shortness of breath presents to you in clinic. On examination, there is reduced air entry and dullness to percussion in the right lung. A pleural tap is performed and the aspirate samples sent for analysis. You are told that the results reveal a protein content of >30 g/L. From the list below, select the most likely diagnosis:
A) Bronchogenic carcinoma
B) Congestive cardiac failure
C) Liver cirrhosis
D) Nephrotic syndrome
E) Meig’s syndrome

Answer: A

11A 79-year-old woman presents to her GP with pain in the left knee. This is particularly bad in the evenings and is stopping her from sleeping. The GP explains that her discomfort is most likely due to osteoarthritis and arranges for her to have
an x-ray of the knee. Which of the following descriptions are most likely to describe the x-ray?
A) Reduced joint space, subchondral sclerosis, bone cysts and osteophytes
B) Increased joint space, subchondral sclerosis, bone cysts and osteophytes
C) Reduced joint space, soft tissue swelling and peri-articular osteopenia
D) Increased joint space, soft tissue swelling and peri-articular osteopenia
E) Normal x-ray

Answer: A

12A 52-year-old man is referred for newly diagnosed diabetes.  He reports a two month history of polyuria and polydipsia. He has no past history of note and was not taking any regular medications, although he reports recent weak erections. On examination, liver edge is palpable 2 cm below the right costal margin and his testes are found to be small. HbA1c 10.8, Hemoglobin15.1 g/dL, Platelets 341 x109/L, Serum iron 50 µmol/L (12-30), Transferrin saturation 85% (<50%). Which investigation is most likely to assist in making a diagnosis?
A) Genetic analysis for C282Y and H63D mutations 
B) Red cell transketolase
C) Serum ferritin
D) Total iron binding capacity (TIBC)
E) Ultrasound of the abdomen

Answer: A

13A 49-year-old man complains of sudden onset, painless unilateral visual loss lasting about a minute. He describes ‘a black curtain coming down’. His blood pressure is 158/90, heart rate 73 bpm. There is an audible bruit on auscultation of his neck. His past medical history is insignificant other than deep vein thrombosis of his right leg ten years ago. The most likely diagnosis is:
A) Retinal vein thrombosis
B) Retinal artery occlusion
C) Amaurosis fugax
D) Optic neuritis
E) Acute angle glaucoma

Answer: C

14A 69-year-old man present with a 2-week history of abdominal pain which has worsened over the last few days. On examination, the patient is jaundiced and the abdomen is distended with tenderness in the epigastric region. In addition, there is a smooth hepatomegaly and shifting dullness. Which of the following is a cause of hepatomegaly?
A) Iron deficiency anaemia
B) Budd–Chiari syndrome
C) Ulcerative colitis
D) Crohn’s disease
E) Left-sided heart failure

Answer: B

15A 79-year-old woman is admitted to the coronary care unit (CCU) with unstable angina. She is started on appropriate medication to reduce her cardiac risk. She is hypertensive, fasting glucose is normal and cholesterol is 5.2. She is found to be in atrial fibrillation. What is the most appropriate treatment?
A) Aspirin and clopidogrel
B) Digoxin
C) Cardioversion
D) Aspirin alone
E) Warfarin

Answer: E

16A 53-year-old healthy man was found to have a fasting plasma glucose of 7.3 mmol. Further testing showed HbA1c of 6.2. What should be done next?
A) Repeat HbA1c
B) Repeat fasting glucose
C) Advise home glucose monitoring 
D) lifestyle changes, repeat all tests in 3 months
E) Measure 2-hour postprandial glucose

Answer: B

17A 66-year-old woman presents to accident and emergency with a 2-day history of shortness of breath. The patient notes becoming progressively short of breath as well as a sharp pain in the right side of the chest which is most painful when taking a deep breath. The patient also complains of mild pain in the right leg, though there is nothing significant on full cardiovascular and respiratory examination. Heart rate is 96 and respiratory rate is 12. The patient denies any weight loss or long haul flights but mentions undergoing a nasal polypectomy 3 weeks ago. The most likely diagnosis is:
A) Muscular strain
B) Heart failure
C) Pneumothorax
D) Angina
E) Pulmonary embolism

Answer: E

18A 78-year-old woman is admitted with heart failure. The underlying cause is determined to be aortic stenosis. Which sign is most likely to be present?
A) Pleural effusion on chest x-ray
B) Raised jugular venous pressure (JVP)
C) Bilateral pedal oedema
D) Bibasal crepitations
E) Atrial fibrillation

Answer: D

19A 22-year-old woman complains of dizziness and feeling light-headed. She works in an office and most frequently experiences this when standing up to visit the toilet. She has never fainted. The patient has lost 5 kg, but attributes this to eating more healthily. She has noticed a recent scar on the back of her hand which has started to turn very dark. The most appropriate investigation is:
A) Synacthen test
B) Low-dose dexamethasone test
C) Cortisol measurement
D) Urinary free cortisol measurement
E) Abdominal ultrasound (US) scan

Answer: A

20A 71-year-old man with atrial fibrillation is seen in clinic following an episode of syncope. He describes getting a poor night’s sleep and, as he got out of bed in the morning, feeling dizzy for a couple of seconds before the lights dimmed around him. He was woken a couple of seconds later by his wife who had witnessed the event. She says he went pale and fell to the floor and his arms and legs jerked. After waking, he was shaken but was ‘back to normal’ a few minutes after the event. His medication includes aspirin, atenolol and frusemide. What is the most likely diagnosis?
A) Vasovagal syncope
B) Orthostatic hypotension
C) Cardiogenic syncope
D) Transient ischaemic attack (TIA)
E) Seizure

Answer: B

21A 29-year-old man presents with a 4-week history of polyuria and extreme thirst. The patient denies difficulty voiding, hesitancy or haematuria, although the urine is very dilute. The patient does not believe he has lost any weight and maintains a good diet. No findings are found on urine dipstick. The most appropriate investigation is:
A) Serum osmolality
B) Fasting plasma glucose
C) Urinary electrolytes
D) Magnetic resonance imaging (MRI) scan of the head
E) Water deprivation test

Answer: E

22You are discussing a patient with your registrar who has become acutely short of breath on the ward. After performing an arterial blood gas, you have high clinical suspicion that the patient has a pulmonary embolism. Which of the following is the investigation of choice for detecting pulmonary embolism?
A) Magnetic resonance imaging (MRI) of the chest
B) High-resolution CT chest (HRCT)
C) Chest x-ray
D) Ventilation/perfusion scan (V/Q scan)
E) CT pulmonary angiogram (CT-Pa)

Answer: E

23A patient is admitted with pneumonia. A murmur is heard on examination. What finding points to mitral regurgitation?
A) Murmur louder on inspiration
B) Murmur louder with patient in left lateral position
C) Murmur louder over the right 2nd intercostal space midclavicular line
D) Corrigan’s sign
E) Narrow pulse pressure

Answer: B

24A 43-year-old woman suffers from Crohn’s disease. A blood test shows the following results:
Haemoglobin 10.5 g/dL
MCV 120 fL
Platelet count 300× 109/L
The most likely diagnosis is:
A) Vitamin B12 deficiency
B) Iron deficiency
C) Hypothyroidism
D) Folic acid deficiency
E) Anaemia of chronic disease

Answer: A

25A 27-year-old woman develops left leg swelling during week 20 of her pregnancy. Left lower extremity ultrasonogram reveals a left iliac vein deep vein thrombosis. What is the best next step?
A) bed rest
B) catheter-directed thrombolysis
C) enoxaparin
D) inferior vena cava filter placement
E) Repeat doppler U/S after 1 week

Answer: E

26. A 67-year-old woman is admitted to accident and emergency with pyrexia (38.1°C) and a cough productive of green sputum. The observations show a pulse rate of 101, BP 80/60 and respiratory rate of 32. She is alert and orientated in space and time. Blood results reveal a WCC of 21, urea of 8.5 and chest x-ray shows a patch of consolidation in the lower zone of the right lung. She is treated for severe community-acquired pneumonia. Which of the following is the correct calculated CURB-65 score?
A) 6
B) 8
C) 4
D) 0
E) 1

Answer: C

27. A 69-year-old man presents to clinic with a six-month history of progressive lower back pain which radiates down to his buttock. He found the pain was exacerbated while taking his daily morning walk and noticed that it eased going uphill but worsened downhill. He stopped his daily walks as a result and he now walks only slowly to the shops when he needs to, taking breaks to sit down and ease the pain. He has a history of hypertension, diabetes and prostatic hyperplasia. What is the diagnosis?
A) Peripheral vascular disease
B) Osteoporotic fracture
C) Spinal stenosis
D) Sciatica
E) Metastatic bone disease

Answer: C

28A 47-year-old woman is evaluated for difficult-to-control hypertension. She was previously treated for hypokalemia. On physical examination, temperature is 36.0 °C (96.8°F), blood pressure is 178/100 mm Hg, pulse rate is 58/min, respiration rate is 16/min, and BMI is 29. No abdominal bruit is detected. Funduscopic examination shows mild arteriolar narrowing. Laboratory studies:
Electrolytes
Sodium 143 meq/L (143 mmol/L)
Potassium 3.5 meq/L (3.5 mmol/L) (after replacement
therapy)
Chloride 101 meq/L (101 mmol/L)
Bicarbonate 33 meq/L (33 mmol/L)
Aldosterone 
Baseline 23 ng/dL (635 pmol/L)
3 Days after high salt intake 15 ng/dL (414 pmol/L)
Renin activity
Baseline <0.1 ng/mL/h (0.1 μg/L/h)
3 Days after high salt intake <0.1 ng/mL/h (0.1 μg/L/h)
Aldosterone to renin activity ratio >50
Which of the following is the most appropriate next step in management?
A) Adrenalectomy
B) Bilateral adrenal vein catheterization
C) CT of the adrenal glands
D) Duplex ultrasonography of the renal arteries

Answer: C

29Decreased or absent haptoglobin levels are seen in the following conditions EXCEPT
A) Hemolytic anemia 
B) Genetic disorders
C) Acute hepatitis
D) Pregnancy
E) Burns

Answer: C

30A 41-year-old man is diagnosed with iron deficiency anemia and is found to have heme-positive stools. Colonoscopy reveals a large ulcerated tumor in his transverse colon. He also has two smaller polyps in his ascending colon. Pathologic examination of the tumor biopsy reveals adenocarcinoma, while biopsies of the polyps confirm that these are adenomas. His sister has been diagnosed with
uterine cancer, and two cousins have died of colon cancer. All of the following are true statements about this case except
A. Referral for genetic counseling is indicated
B. He is at increased risk for other epithelial-derived tumors
C. He likely has familial adenomatous polyposis (FAP), with a germ line mutation in the APC gene
D. His condition is often associated with a defect in DNA mismatch repair

Answer: B

31Concerning adrenal physiology, which of the following hormones is secreted by the zona glomerulosa?
A) Adrenaline
B) Noradrenaline
C) Cortisol
D) Aldosterone
E) Angiotensin II

Answer: D

32A 76-year-old gentleman is referred with an international normalized ratio (INR) of 8.4. He normally takes warfarin for a prosthetic mitral valve. He had an epistaxis lasting 15 minutes this morning. He is otherwise asymptomatic and haemodynamically stable. Having stopped his warfarin for tonight, Which of the following is the most appropriate course of action?
A) 5 mg iv vitamin K
B) 0.5 mg orally (po) vitamin K
C) 4 units of fresh frozen plasma iv
D) 50 units/kg of prothrombin complex concentrate
E) Recheck INR in 24 hours

Answer: E

33A 64-year-old who suffers from haemochromatosis is seen in A&E with a 2-day history of pain and swelling in his right knee. On examination it is swollen and he has decreased range of movement. You aspirate his knee. What will the aspirate be most likely to show?
A) Gram-positive cocci in clusters
B) Positively bifringent crystals
C) Gram-positive cocci in chains
D) Inflammatory cells with haemosiderin deposition
E) Negatively bifringent crystals

Answer: B

34A 50-year-old male with a history of alcohol dependence and chronic hepatitis C presents to your clinic with three months of fatigue, weakness and weight loss. He has also noted some “yellowing of my eyes.” You suspect cirrhosis. All the following are clinical signs of cirrhosis except
A) Arthralgia
B) Asterixis
C) Dupuytren’s contracture
D) Haemorrhoids
E) Testicular atrophy

Answer: A

35A 75-year-old lady presents with persistent diarrhoea and weight loss. Twenty years previously she had radiotherapy for carcinoma of the ovary. Thirty years ago she lived in Hong Kong for 18 months. Her father died of cancer of the colon aged 62 years.
Hb 9.4 g/dL (11.5-16 g/dL)
WCC 9.6 x 109/L (4-11 x 109/L)
Platelets 234 x 10/L (150-400 x 109/L)
B12 153 pmol/L (160-900pmol/L)
Red cell folate 21 nmol/L (130-630 nmol/L)
Ferritin 5 ug/L (20-250 ug/L)
Erythrocyte sedimentation rate (ESR) 36 mm/hour 
Which of the following is the most likely diagnosis?
A) Radiation enteritis
B) Carcinoma of the colon
C) Crohn's disease 
D) Tropical sprue
E) Acquired lactose intolerance

Answer: A

36A phlebotomist sustains a needlestick injury whilst taking blood from a HIV-positive patient. Which one of the following has been shown to most reduce the risk of HIV transmission?
A) Thorough immediate washing of the injury site with water
B) Oral lamivudine therapy for 1 month
C) Oral zidovudine therapy for 1 month
D) Oral ritonavir therapy for 1 month
E) Oral triple therapy for 1 month

Answer: E

37A 33-year-old female is seen in the maternity department.she suffers from SLE. Her child was born with profound bradycardia. Which of the following antibodies is most likely to be present?
A) Anti-Ro
B) Anti-La
C) Anti-Jo
D) Anti-Scl-70
E) Antiphospholipid antibodies

Answer: A

38A 67-year-old man presents with nausea and pain in his right upper quadrant. On examination, he has tender hepatomegaly with an irregular edge. His blood tests are as follows:
Hb 8 g/dL (11.5-16 g/dL)
WCC 17.8 x 109/L (4-11 x 109/L)
Platelets 104 x 109/L (150-400x109/L)
Neutrophils 68%
Normoblasts 8%
Myeloblasts 8%
Myelocytes 5%
Metamyelocytes 5%
Lymphocytes 15%
What is the most likely cause of his anaemia?
A) Gaucher's disease
B) Leukaemoid reaction secondary to acute infection
C) Myelosclerosis
D) Malignant infiltration of bone marrow 
E) Osteopetrosis

Answer: D

39A 32-year-old woman presents with left loin pain and haematuria. She is known to suffer with Crohn's disease.
Which of the following is likely to be the aetiology of this presentation?
A) Hypercalciuria
B) Hyperbilirubinaemia
C) Hyperuricaemia
D) Hyperoxaluria
E) Type 1 renal tubular acidosis

Answer: D

40Regarding glucocorticoid-induced hyperglycemia all the following statements are correct EXCEPT.
A) The risk of new onset diabetes mellitus is related to the total glucocorticoid dose and d)ration of therapy.
B) Raised fasting plasma glucose is the earliest abnormality. 
C) Reduced insulin sensitivity is the predominant mechanism.
D) Insulin is the most effective and safety therapy
E) Long term use of thiazolidinediones in combination with glucocorticoid is contraindicated.

Answer: B

41A 37-year-old asymptomatic woman presents for a routine physical examination. Physical examination reveals a 2-cm right-sided thyroid nodule that is firm, nontender and moves with swallowing. The rest of the physical examination is unremarkable. Lab Work: TSH 1.8 μU/mL. 
She is referred for fine-needle aspiration biopsy of the thyroid nodule. Which of the following is true regarding interpreting the cytologic results of thyroid fine-needle aspiration biopsy? 
A) Follicular carcinoma can be accurately diagnosed. 
B) Most fine-needle aspiration biopsy specimens reveal malignant cells. 
C) Follicular adenoma can be accurately diagnosed. 
D) Papillary carcinoma can be accurately diagnosed. 
E) Cystic nodules are more likely than solid nodules to provide diagnostic material.

Answer: D

42A 40 year old construction laborer admitted to the hospital with sudden onset of dysphagia and inability to open his mouth. During examination he was conscious but unable to speak, in pain and pointing to his back. His abdomen was rigid. His upper and lower limb examination was normal. The most important initial step in the management of this patient is:
A) Perform an urgent CT scan of the neck to exclude pharyngeal abscess
B) Start immediate Ceftriaxone and Vancomycin
C) Start I.V Penicillin,Tetanus Ig and Lorazepam
D) Start I.V Metronidazole,Tetanus Ig and Lorazepam
E) Search for a wound, perform wound cleaning or debridement followed by I.V penicillin and Tetanus Ig

Answer: D

4374-year-old man 70 Kg enters the ICU with fever and hypotension. He has a history of kidney stones with multiple previous bouts of urinary tract infection. 1.5 L of Normal saline bolus started, Urine and blood cultures are obtained and he was started on Tazocin and Vancmycin, his BP remain 70/40 and additional 500 ml of Normal saline were infused, CVC placed and norepinephrine started 10 mcg/min, He was intubated to decrease work of breathing and his hemodynamic data as follow:
BP: 80/45 (55), CVP: 6, Urine output 20 ml/hr, Lactate: 6, ScVO2: 50%.
What is the next step in managing this patient?
A) Start stress dose steroids for refractory septic shock.
B) Add vasopressin to the current regimen for refractory septic shock.
C) Start Dobutamine to increase cardiac output and oxygen delivery.
D) Bolus IV normal saline to keep CVP 8-12.
E) Start furosamide to keep urine output more than 35 ml/hr.

Answer: D

44A 59-year-old lady is admitted with a 30 minute history of heavy central chest pain associated with nausea and sweating. Her ECG shows ST elevation in leads V1, V2, V3 and V4. Which of the following coronary arteries is most likely to be occluded?
A) Circumflex artery 
B) Left anterior descending artery 
C) Obtuse marginal artery 
D) Posterior descending artery 
E) Right coronary artery

Answer: B

45A 68-year-old man presents with a one-month history of dyspnoea and a 3kg weight loss. On examination there were signs of a large left pleural effusion, confirmed on Chest X-ray.
Pleural fluid analysis:
Protein 38 g/L
Cytology a few lymphocytes and red blood cells.
Which one of the following investigations should be considered next?
A) Bronchoscopy 
B) CT scan of thorax Correct 
C) Repeat pleural aspiration with biopsy 
D) Thoracoscopic pleural biopsy 
E) Tuberculin test

Answer: B

46A 65-year-old male is admitted with a two hour history of central chest pain associated with sweating and nausea.
A myocardial infarction is suspected. Which of the following is an indication for thrombolytic therapy?
A) 1mm ST elevation in leads II, III and aVF. 
B) 1mm ST elevation in leads V2 and V6 
C) 2mm ST depression in leads V2-4 
D) Q waves in leads V2-4 
E) T wave inversion in lead V3-5

Answer: A

47A 32-year-old female presents with 3-day history of colicky abdominal pain and fatigue. 
Lab: HB 2.9 g/dl, Haematocrit 22%, MCV 78, platelets 60000, retic count 9%, direct and indirect Coomb’s test (-), ferritin 10, abdominal US shoes portal vein thrombosis. 
Urine analysis: hemosiderin 
The most likely diagnosis is:
A) Factor V Leiden mutation
B) Paroxysmal cold hemoglobinuria
C) Warm autoimmune hemolytic anemia
D) Aplastic anemia
E) Paroxysmal nocturnal hemoglobinuria

Answer: E

48An 18-year-old attending the A+E department is noted to have central cyanosis. She is perfectly well but was told to go to A+E by her friends who said she looked blue. What is the most likely cause? 
A) Carbon Monoxide Poisoning 
B) Lead Poisoning 
C) Drinking water contaminated with nitrates Correct 
D) Anorexia Nervosa 
E) Severe Anaemia

Answer: C

49A 58-year-old woman comes to the emergency department complaining of crampy left upper quadrant pain that is exacerbated by fatty foods. She has a history of diabetes, hyperlipidemia, and gallstones and her medications include glyburide, simvastatin, and aspirin. She denies any alcohol or drug use. She is morbidly obese and her temperature is 37.9 C (100.2 F), blood pressure is 102/87 mm Hg, pulse is 105/min, and respirations are 23/min. On examination, her lungs are clear to auscultation bilaterally. Her cardiac sounds are muffled, although her cardiac rhythm is regular. No murmurs are audible. She has definite left upper quadrant tenderness to palpation, without rebound or guarding. Rectal examination shows guaiac-negative brown stool. Her amylase and lipase levels are elevated. The most appropriate next step is to order a
A) an abdominal ultrasound
B) a chest radiograph
C) an electrocardiogram
D) a HIDA scan
E) an upper endoscopy

Answer: A

50Which of the following statements is NOT true of primary pulmonary tuberculosis:
A) It is characteristically asymptomatic 
B) Miliary spread is commoner in a younger age group 
C) The initial immunological response causes hilar lympadenopathy 
D) Pleural effusion occurs before tuberculin skin testing is positive 
E) A positive tuberculin skin test develops within two weeks of infection

Answer: E

MULTIPLE CHOICE QUESTIONS-9

1A 78-year-old woman is treated for depression with nortriptyline. Which of the following is the most common side effect of nortriptyline?
A) impaired cardiac contractility
B) heart block
C) weight loss
D) anticholinergic side effects
E) diarrhea

Answer: D

2During a neurologic examination, you ask a patient to stand with both arms fully extended and parallel to the ground with his eyes closed for 10 seconds. What is the name of this test?
A) Babinski sign
B) Dysdiadochokinesis
C) Lhermitte symptom
D) Pronator drift
E) Romberg sign

Answer: D

3A person with liver disease caused by Schistosoma mansoni would be most likely to have what condition?
A) Ascites
B) Esophageal varices
C) Gynecomastia
D) Jaundice
E) Spider nevi

Answer: B

4. All of the following hormones is produced by the anterior pituitary EXCEPT:
A) Adrenocorticotropic hormone
B) Growth hormone
C) Oxytocin
D) Prolactin
E) Thyroid-stimulating hormone

Answer: C

5The following statements concerning pulmonary Stenosis are correct
A) It is the commonest cardiac abnormality in Turner's syndrome
B) The chest x-ray plethoric lung fields
C) There is a recognized association with carcinoid syndrome
D) An ejection click indicates that the Stenosis is sub-valvular
E) The pulmonary component of the second sound is accentuated when the Stenosis is severe

Answer: C

6A 36-year-old woman with acquired immunodeficiency syndrome (AIDS) and a CD4 count of 35/μL presents with odynophagia and progressive dysphagia. The patient reports daily fevers and a 20-lb weight loss. The patient has been treated with clotrimazole troches without relief. On physical examination, the patient is cachectic with a body mass index (BMI) of 16 and a weight of 86 lb. The patient has a temperature of 38.2°C (100.8°F). She is noted to be orthostatic by blood pressure and pulse. Examination of the oropharynx reveals no evidence of thrush. The patient undergoes esophagogastroduodenoscopy (EGD), which reveals serpiginous ulcers in the distal esophagus without vesicles. No yellow plaques are noted. Multiple biopsies are taken that show intranuclear and intracytoplasmic inclusions in large endothelial cells and fibroblasts. What is the best treatment for this patient’s esophagitis?
A) Ganciclovir
B) Glucocorticoids
C) Fluconazole
D) Foscarnet
E) Thalidomide

Answer: A

7A 45-year-old man reports to his primary care physician that his wife has noted coarsening of his facial features over several years. In addition, he reports low libido and decreased energy. Physical examination shows frontal bossing and enlarged hands. An MRI confirms that he has a pituitary mass. Which of the following screening tests should be ordered to diagnose the cause of the mass?
A) 24-Hour urinary free cortisol
B) Adrenocorticotropic hormone (ACTH) assay
C) Growth hormone level
D) Serum insulin-like growth factor-1 (IGF-1) level
E) Serum prolactin level

Answer: D

8Bronchopulmonary aspergillosis is characterized by all of the following EXCEPT: 
A) Underlying asthma 
B) Central bronchiectasis 
C) Elevated serum immunoglobulin E level 
D) Positive serum precipitins for Aspergillus 
E) Positive delayed hypersensitivity skin test to Aspergillus antigens

Answer: B

9The following are complications of nephrotic syndrome with the exception of 
A) Acute renal failure 
B) Accelerated hypertension 
C) Hypocalcaemia 
D) Pneumococcal infection 
E) Venous thrombosis

Answer: B

10You have a febrile 37-year-old male with a very high WBC count, most of which are blasts. Most likely, he has an acute leukemia. A bone marrow has not yet been done, and it is unknown if this is an acute myeloid leukemia or an acute lymphoblastic leukemia. Which of the following statements is NOT TRUE?
A) The patient’s fever is likely due to the leukemic cells, and antibiotics should be started only if you identify a specific infection.
B) The patient is at risk for tumor lysis syndrome and should be started on allopurinol.
C) Aggressive inpatient chemotherapy is required for both the treatment of acute myeloid leukemia and acute lymphoblastic leukemia.
D) You should consult a hematologist/oncologist as soon as possible.

Answer: A

11A 48-year old female with rheumatoid arthritis presents to the emergency department with 2-week pain and tightness behind the left knee. Examination reveals cystic swelling over the left popliteal fossa. Which of the following is the most appropriate next action? 
A) Arthrogram of the left knee
B) Synovial biopsy of left knee
C) Ultrasound study of left knee and popliteal fossa
D) Venogram of left lower limb
E) None of the above

Answer: C

12Concerning pseudomyxoma peritonei: All are true EXCEPT
A) Begins with a mucinous cystadenocarcinoma, most commonly involving either the ovary or appendix.
B) Rupture of the lesion results in diffuse metastatic spread with implantation of the mucin secreting lesions on the omentum as well as the peritoneum. 
C) Average age of presentation of this disease process is 45-55, with men making up 80% of cases.
D) Presenting symptoms are non-specific, generally including pain, but also usually with a palpable mass.
E) A 5-year survival of these patients is only approximately 50%.

Answer: C

13A 60-year-old man with end stage renal failure from chronic glomerunephritis presents with acute onset of gross hematuria and mild flank pain. He has been on hemodialysis for 4 years, and his course otherwise been uneventful. He was afebrile and the hematuria resolved without intervention. Which of the following is the most appropriate action at this time?
A) Angiography
B) Computed tomography (to rule out acquired cystic kidney disease)
C) Renal ultrasound
D) Intravenous pyelography
E) None of the above

Answer: B

14A 30-year-old female with systemic lupus erythematosus is recovering from a fracture of the right femur and right radius following a motor vehicle accident. She has been in the hospital for 5 days. She has a temperature spike of 39.0 °C (102.2 °F). Blood cultures are drawn which grow yeast, species to be identifi ed. She does not appear ill or toxic. No obvious source of infection is found. The patient has an indwelling central catheter. In addition to changing the patient’s central line, which of the following do you recommend?
A) Continued observation.
B) Computed tomography of the chest and abdomen.
C) Start fluconazole.
D) Start liposomal amphotericin B.
E) Repeat blood cultures and treat if positive

Answer: C

15A 78-year-old woman is admitted to the medical intensive care unit (ICU) with multilobar pneumonia. On initial presentation to the emergency department, her initial oxygen saturation was 60% on room air and only increased to 82% on a non-rebreather face mask. She was in marked respiratory distress and intubated in the emergency department. Upon admission to the ICU, she was sedated and paralyzed. The ventilator is set in the assist-control mode with a respiratory rate of 24 breaths/min, tidal volume of 6 mL/kg, FiO2 of 1.0, and positive end-expiratory pressure of 12 cmH2O. An arterial blood gas measurement is performed on these settings; the results are pH of 7.20, PCO2 of 32 mmHg, and PO2 of 54 mmHg. What is the cause of the hypoxemia?
A) Hypoventilation alone
B) Hypoventilation and ventilation-perfusion mismatch
C) Shunt
D) Ventilation-perfusion mismatch

Answer: C

16Which of the following may be responsible for a hypokalaemic hypertension? 
A) Non-classical congenital adrenal hyperplasia 
B) Barter's syndrome 
C) Diabetic nephropathy 
D) Liddle's syndrome 
E) Type IV renal tubular acidosis

Answer: D

17A 63-year-old male recently admitted with sepsis is noted to have a urine output of approximetely 20 mls per hour. The oliguria is more likely to be due to prerenal failure than intrinsic renal failure if:
A) A urine free of red blood cells or casts Correct 
B) A urine:plasma urea ratio < 3 
C) Urine osmolality <350 mOsm/l 
D) A blood pressure of 150/90 and good tissue perfusion. 
E) Urinary sodium >10mmol/l

Answer: A

18A 50-year-old female presents with acute chest pain and dyspnea. Examination reveals bilateral ankle oedema with 24 hr urine protein assessment showing 8g/d (<0.2). Which is the most likely explanation for these findings?
A) Factor V Leiden Incorrect answer selected 
B) Reduced antithrombin III activity This is the correct answer 
C) Reduced concentration of Von Willebrand’s factor 
D) Reduced fibrinogen concentration 
E) Reduced factor VIII

Answer: B

19A 19-year-old woman became breathless while travelling on an aeroplane. Which one of the following features most strongly supports a diagnosis of acute hyperventilation related to a panic disorder? 
A) Carpal spasm. 
B) Finger paraesthesiae. 
C) Hypotension. 
D) Light-headedness. 
E) Loss of conciousness

Answer: A

20Which ONE of the following is a recognised feature of achondroplasia? 
A) Autosomal recessive inheritance 
B) May be diagnosed radiologically at birth 
C) Increased liability to pathological fractures 
D) Shortened spine 
E) Subfertility

Answer: B

21You are asked to see a 67-year-old woman who was recently discharged from the hospital following internal fixation of fragility fracture of left femoral neck. She had hypertension and primary hypothyroidism. Her medications include lisinopril 10 mg and L-thyroxine 100ug per day. Family history was unremarkable. On physical examination BMI 23.9 kg/m2 BP 130/84 mmHg. Laboratory studies showed: Serum creatinine 1.2 mg/dl Serum calcium 9.2 mg/dl: Serum PTH 69 pg/ml (N 10-58): Serum 25 hydroxy vitamin D 42 ng/mg (N 30-70): Serum TSH 1.9 mU/l (N 0.4-4.0 u): Bone density measured by DXA revealed: T score : Hip-2.9 Lumbar spine -2.7. In addition to vitamin D and calcium supplement, which of the following you recommend to reduce the incidence of another hip fracture?
A) Zoledronate 5mg IV yearly
B) Combined estrogen and progestin.
C) Raloxifene 60 mg per day.
D) Alphacalcidol 1 µg per day

Answer: A

22A 29-year-old man with episodic abdominal pain and stress-induced edema of the lips, tongue, and occasionally larynx is likely to have low functional or absolute levels of which of the following proteins?
A. C1 esterase inhibitor
B. C5A (complement cascade)
C. Cyclooxygenase
D. Immunoglobulin (Ig) E
E. T-cell receptor, α chain

Answer: A

23Pseudopseudohypoparathyroidism is characterized by all of the following EXCEPT:
A) Hypocalcemia 
B) G5 mutation
C) Albright’ s hereditary osteodystrophy 
D) inheritance of the genetic defect from the father
E) Normal urinary c-AMP response to parathyroid hormone

Answer: A

24Most of the manifestations of acute rheumatic fever present approximately 3 weeks after the precipitating group A streptococcal infection. Which manifestation may present several months after the precipitating infection?
A) Chorea
B) Erythema marginatum
C) Fever
D) Polyarthritis
E) Subcutaneous nodules

Answer: A

25A 57-year-old woman with depression and chronic migraine headaches reports several years of dry mouth and dry eyes. Her primary complaint is that she can no longer eat her favorite crackers, although she does report photosensitivity and eye burning on further questioning. She has no other associated symptoms. Examination shows dry, erythematous, sticky oral mucosa. All of the following tests are likely to be positive in this patient EXCEPT:
A) La/SS-B antibody
B) Ro/SS-A antibody
C) Schirmer I test
D) Scl-70 antibody
E) Sialometry

Answer: D

26Which of the following has a well-established association with gastroesophageal reflux?
A) Chronic sinusitis
B) Dental erosion
C) Pulmonary fibrosis
D) Recurrent aspiration pneumonia
E) Sleep apnea

Answer: B

27A 68-year-old woman presents with a left arm tremor. She has had the tremor for about 3 years and recently was noticed to have developed a side-to-side head tremor as well. Her tremor worsens when she performs movements that involve her bringing her hand to her face, such as smoking or drinking coffee. Of note is that if she drinks beer, the tremor seems to improve.
Which of the following is the likely etiology?
A) Gilles de la Tourette syndrome 
B) Parkinson disease 
C) Myasthenia gravis 
D) Benign essential tremor 
E) Huntington disease

Answer: D

28The following are recognized features of Pancoast's tumour except:
A) ipsilateral Horner's syndrome
B) wasting of the dorsal interossei
C) pain in the arm radiating to the fourth and fifth fingers
D) erosion of the first rib
E) weakness of abduction at the shoulder

Answer: E

29Major risk factor for osteoporosis includes all of the following EXCEPT.
A) Tobacco smoking 
B) Glucocorticoid therapy.
C) Primary hyperparathyroidism.
D) History of previous fragility fracture after the age 40 year.
E) Hypogonadism

Answer: D

30A 20-year-old male student is assessed for shortness of breath that occurs whilst running. He has no other symptoms and does not smoke. Examination, full blood count, and chest X-ray are normal. Which of the following is most likely to be helpful in confirming the suspected diagnosis?
A) Arterial blood gas studies before and after exercise
B) Determination of lung volumes and diffusing capacity
C) Measurement of venous blood lactate before and after exercise
D) Spirometry before and after administration of bronchodilators
E) Spirometry before and after exercise

Answer: E

31A 65-year-old man has the following blood results on admission to hospital:
Na 132 mmol/L (135-145 mmol/L)
K 8.6 mmol/L (3.5-5 mmol/L)
Urea 42.4 mmol/L (2.5-6.7 mmol/L)
Creatinine 1178 um0l/L (70-150 umol/L)
The patient appears unwell and the ECG shows tented T waves, absent P waves and widened QRS. Which of the following therapeutic interventions should be performed first?
A) Immediate haemodialysis
B) Intravenous insulin and dextrose
C) Intravenous calcium gluconate
D) Nebulized salbutamol
E) Oral calcium resonium

Answer: C

32A 55-year-old man with chronic hepatitis C is being considered for liver transplantation. The patient has cirrhosis that was documented by liver biopsy 10 years ago. For the past 3 months, he has had ascites and edema, which are poorly controlled with diuretics. Lactulose was recently begun because of confusion. Which of the following combinations of laboratory studies will be most helpful in estimating his survival over the next 6 months? 
A) Serum total bilirubin and INR 
B) Serum aspartate aminotransferase and gamma globulin 
C) Serum alanine aminotransferase and hepatitis C RNA (HCV RNA) 
D) Serum alkaline phosphatase and ammonia 
E) Serum albumin and ?-glutamyltransferase

Answer: A

33Signs and symptoms of opioid withdrawal include all of the following EXCEPT
A) Increased blood pressure (BP) and heart rate
B) Seizures
C) Abdominal cramps
D) Jerking of the legs
E) Hyperthermia

Answer: B

34Which of the following β-adrenergic antagonists is a nonselective β1 and β2 blocker?
A) Atenolol
B) Betaxolol
C) Esmolol
D) Metoprolol
E) Nadolol

Answer: E

35In primary hypothyroidism increased requirements for L thyroxine occur in the following conditions EXCEPT:
A) Nephrotic syndrome with massive proteinuria.
B) Pregnancy.
C) Achalasia
D) Helicobacter pylori associated chronic gastritis.
E) Use of estrogen replacement therapy.

Answer: C

36A 70-year-old man is admitted with new-onset atrial fibrillation with an apical rate of 120/minute and a BP of 100/70 mm Hg. Shortly after admission he develops moderate lower abdominal pain and diarrhea with dark red blood.
Which is the most appropriate course of action?
A) Cardioversion 
B) Unfractionated heparin
C) Mesenteric angiography
D) Laparotomy
E) Digoxin

Answer: A

37A 73-year-old male presents to the clinic with 3 months of increasing back pain. He localizes the pain to the lumbar spine and states that the pain is worst at night while he is lying in bed. It is improved during the day with mobilization. Past history is notable only for hypertension and remote cigarette smoking. Physical examination is normal. Laboratory
studies are notable for an elevated alkaline phosphatase. A lumbar radiogram shows a lytic lesion in the L3 vertebra. Which of the following malignancies is most likely?
A) Gastric carcinoma
B) Non-small cell lung cancer
C) Osteosarcoma
D) Pancreatic carcinoma
E) Thyroid carcinoma

Answer: B

38A 36-year-old African-American woman with systemic lupus erythematosus presents with the acute onset of lethargy and jaundice. On initial evaluation, she is tachycardic, hypotensive, appears pale, is dyspneic, and is somewhat difficult to arouse. Physical examination reveals splenomegaly. Her initial hemoglobin is 6 g/dL, white blood cell count is 6300/μL, and platelets are 294,000/ μL. Her total bilirubin is 4 g/dL, reticulocyte count is 18%, and haptoglobin is not detectable. Renal function is normal, as is urinalysis. What would you expect on her peripheral blood smear?
A) Macrocytosis and PMN’s with hypersegmented nuclei
B) Microspherocytes
C) Schistocytes
D) Sickle cells
E) Target cells

Answer: B

39A 22 year-old lady presents to her GP with facial swelling, most marked in the morning, and shortness of breath. Urinalysis shows protein +++. Results were as follows: 
24-hour urinary protein 4.2 g; 
Haemoglobin(Hb) 10.8 g/l (11.5-16.0 g/L); 
Urea 22.8 mmol/L (2.5-6.5 mmol/L); 
Creatinine 374 µmol/L (70-150 umol/L)
C3 0.24 g/L (0.55-1.2g/L)
C4 0.29g/L (0.2-0.5 g/L)
Which of the following is the most likely diagnosis?
A) Minimal change nephropathy
B) Systemic lupus erythematosus
C) Rheumatoid arthritis
D) Cryoglobulinaemia
E) Diabetes mellitus

Answer: B

40A 31-year-old male with hemophilia A is admitted with persistent gross hematuria. He denies recent trauma or any history of genitourinary pathology. The examination is unremarkable. Hematocrit is 28%. All the following are treatments for hemophilia A except
A) desmopressin (DDAVP)
B) fresh-frozen plasma (FFP)
C) cryoprecipitate
D) recombinant factor VIII
E) plasmapheresis

Answer: E

41A 50-year-old male is taken to the General Practitioner by his long suffering wife. His snoring (which has been steadily increasing in loudness over the past 18 months) is troublesome at home. She says that he makes noises and moves around whilst asleep. He reports no problems with sleeping. He does admit to gaining 20 kg in weight over the past one year, and to falling asleep during the day. A sleep study is performed. Which of the following findings would be most compatible with this man's clinical presentation?
A) Fragmented sleep, cessations of airflow measured at the nose accompanied by an increase in oesophageal pressure swings and episodic oxygen desaturation. 
B) Normal sleep quality, bradycardic episodes, oxygen desaturation but normal airflow. 
C) Normal sleep quality but cessations of airflow measured at the nose with decreased abdominal wall motion during these flow cessations. 
D) Progressive oxygen desaturation during the night and alternating periods of hyperventilation and hypoventilation. 
E) Tachycardia, sleep fragmentation, episodes of hypoventilation with minimal oxygen desaturation.

Answer: A

42A 50-year-old female presents with acute chest pain and dyspnoea. Examination reveals bilateral ankle oedema with 24 hr urine protein assessment showing 8g/d (<0.2). Which is the most likely explanation for these findings? 
A) Factor V Leiden 
B) Reduced antithrombin III activity 
C) Reduced concentration of Von Willebrand’s factor 
D) Reduced fibrinogen concentration 
E) Reduced factor VIII

Answer: B 

43A 60-year-old woman is establishing care in your clinic. She has occasional numbness and tingling in her fingers and toes and has noted some numbness around her mouth, especially when she is stressed or anxious. She had thyroid surgery for Graves disease about 2 years ago and takes 100 µg of levothyroxine and 1 tablet of calcium daily. On exam, her blood pressure is 130/80, pulse 80, and she has cramping in her right forearm and fingers when the blood pressure cuff is attached. Based on this history and exam, which of the following is most likely?
A) Calcium 8.5, PTH 65, PO4 4.5
B) Calcium 9.5, PTH 35, PO4 4.0
C) Calcium 6.0 (8.5-10.5), PTH 2 (10-65), PO4 6.0 (2.7-4.5)
D) Calcium 10.8, PTH 108, PO4 2.3
E) Calcium 8.0, PTH 98, PO4 2.1

Answer: C

44A 70-year-old man was referred by his GP with difficult to treat hypertension. He had long-standing hypertension which had been well controlled over many years but recently he was found to have a blood pressure of 190/110 mmHg which proved resistant to additional treatment. He was generally asymptomatic and complied with medication. Investigations showed normal U+Es. Which one of the following is the most likely cause? 
A) Chronic pyelonephritis 
B) Conn’s syndrome (primary hyperaldosteronism) 
C) Phaeochromocytoma 
D) Polycystic kidney disease 
E) Renovascular disease

Answer: E

45Regarding differentiated thyroid carcinoma the following statements are correct EXCEPT.
A) Radiation is a recognized risk factor.
B) Palpillary carcinoma is the most common type.
C) Fine needle aspiration biopsy is very sensitive for the diagnosis of minimally invasive follicular carcinoma.
D) Serum thyroglobulin is a sensitive tumor marker for patients who underwent total thyroidectomy and radioiodine remnant ablation

Answer: C

46Risk factors for foot ulcers in patient with diabetes includes all the following EXCEPT:
A) Foot deformity.
B) Peripheral neouropathy.
C) Tobacco smoking.
D) Female sex.
E) Poor glycemic control.

Answer: D

47A 51-year-old man is post-procedure day number 2 from an upper endoscopy and banding for bleeding esophageal varices. The patient has a 7-year history of chronic active hepatitis and over the past few years has developed stigmata associated with cirrhosis and worsening portal hypertension. Three days ago, he presented to the emergency department with bright red blood per mouth and rectum and a nasogastric tube evacuated bright red blood and coffee grounds from the patient's stomach. He was admitted to the hospital, transfused with 2 units of red blood cells and underwent an endoscopy. On preparation for the patient's discharge, you have a long discussion with your patient about the course of events. In counseling this patient on his future risks and course of therapy, you should advise him that:
A) The risk of rebleeding is between 50% and 80% and medical therapy is indicated
B) The risk of rebleeding is between 50% and 80% and surgical therapy is indicated
C) The risk of rebleeding is between 50% and 80%, but no therapy is indicated given the nature of the problem
D) There is no concern for rebleeding
E) There is no concern for rebleeding, he is an imminent candidate for transplant

Answer: A

48A 36-year-old woman complains of reflux symptoms and intermittent diarrhea. The diagnosis of gastrinoma is suspected so a fasting serum gastrin is obtained and
found to be 280 pg/mL (normal <115 pg/mL). An abdominal CT is negative. What would you do now?
A) Refer her for an exploratory laparotomy
B) Measure serum gastrin in response to secretin infusion
C) Treat her for H. pylori
D) Obtain a dedicated small bowel series
E) Measure gastric acid secretion

Answer: E

49A phlebotomist sustains a needlestick injury whilst taking blood from a HIV-positive patient. Which one of the following has been shown to most reduce the risk of HIV transmission?
A) Thorough immediate washing of the injury site with water
B) Oral lamivudine therapy for 1 month
C) Oral zidovudine therapy for 1 month
D) Oral ritonavir therapy for 1 month
E) Oral triple therapy for 1 month

Answer: E

50A 60-year-old female has been on long-term treatment for rheumatoid arthritis. She presents with dyspnoea on minimal exertion and a non-productive cough. Oxygen saturation on air is 84% and chest radiograph shows diffuse bilateral interstitial infiltrates. Extensive investigation for an infective cause is negative. Which of the following drugs is most likely to be responsible?
A) Methotrexate
B) Penicillamine
C) Pednisolone
D) Cyclosporin
E) Hydroxychloroquine

Answer: A

MULTIPLE CHOICE QUESTIONS-10

1. Which one of the following is NOT a contraindication to intravenous (IV) tPA in patient with ischemic stroke?
A) Age >75
B) INR >1.7
C) Platelets <100,000
D) Stroke within last 3 months
E) Glucose <50

Answer: A

2A 28-year-old female presents to the emergency department with a severe headache and lethargy. The headache came on fairly quickly whilst she was watching television. She is otherwise fit and well, has no medical problems and takes no regular medications. She lives at home with her husband and two-year-old daughter. Whilst in the emergency department she is given two co-codamol for her persisting headache but then vomits. On examination, there is no focal neurology but she is slightly drowsy and her GCS is 14/15. Hb 126 g/l; Platelets 274 * 109/l; WBC 10.9 * 109/l; Na+ 124 mmol/l; K+ 5.0 mmol/l; Urea 4.1 mmol/l; Creatinine 124 µmol/l. What is the most likely diagnosis?
A) Tension headache 
B) Addison's disease 
C) Pituitary apoplexy 
D) Sub-arachnoid haemorrhage 
E) Cranial diabetes insipidus

Answer: C

3An 18-year-old woman with a microcytic anemia is diagnosed with alpha-thalassemia. Formation of which of the following hemoglobins is increased in alpha-thalassemia?
A) H
B) A
C) F
D) A2
E) C

Answer: A

4Which of the following is most characteristic of diabetic neuropathy?
A) it is usually bilateral
B) pain is not a feature
C) it most commonly affects the brain
D) it spares the autonomic system
E) it responds to meticulous control of blood glucose

Answer: A

5A 19 year-old female is referred following a visit to the dentist where marked erosion of her teeth was noted. She was entirely asymptomatic and her only medication was the oral contraceptive pill. On examination her blood pressure was 110/70 mmHg and her body mass index was 21.5 kg/m2 (18 - 25). Investigations sodium 135 mmol/l, potassium 2.1 mmol/l, bicarbonate 42 mmol/l, urea 2.6 mmol/L corrected calcium 2.08 mmol/ alkaline phosphatase 201 iu/l (50-110) What is the most likely diagnosis? 
A) Bulimia nervosa 
B) Conn's syndrome 
C) Laxative abuse 
D) Pregnancy 
E) Primary hypoparathyroidism

Answer: E

6. An 88-year-old man is taken to A&E by his daughter as he had been feeling unwell that day with chest pains. An ECG reveals that he is suffering an inferior myocardial infarction. He has a heart rate of 33 bpm and blood pressure of 90/60 mmHg. He is given 600mcg of atropine to no effect, this is repeated up to 3mg. You commence an infusion of TPA but his rate is not immediately improving. Which of the following treatments would be most suitable to use for his arrhythmia?
A) Adrenaline
B) Glucagon
C) Temporary pacing
D) Dopamine
E) Dobutamine

Answer: C

7A 55-year-old politician with no previous medical history is admitted to A&E. He has been feeling unwell since Saturday morning and an ECG taken on admission to the Emergency Department at 1am on the Monday morning shows atrial fibrillation at rate of 130 bpm with BP 150/80 mmHg. He is Echoed (trans-thoracic) and there is no evidence of the left atrial thrombus, no abnormal ventricular wall movement and his ejection fraction is normal. What would be the most appropriate initial treatment for this patient?
A) Anticoagulate with IV heparin and carry out DC cardioversion
B) Arrange a TOE next morning and if negative give 300 mg flecainide IV per day
C) Anticoagulate with warfarin and give 300 mg amiodarone IV
D) Start 300 mg amiodarone IV with no anticoagulation
E) Anticoagulate with LMWH and give 300 mg oral flecainide per day

Answer: E

8A74-year-old woman has metastatic bone disease on x-ray. Which of the following mediators is least likely to be involved?
A) interleukin-6 (IL-6)
B) ectopic parathyroid hormone (PTH)
C) tumor necrosis factor (TNF)
D) interleukin-1 (IL-1)
E) prostaglandins

Answer: B

9The medication of choice to treat a patient in torsades de pointes is:
A) Epinephrine.
B) Flecainide.
C) Calcium gluconate
D) Magnesium sulfate.
E) Procainamide

Answer: D

10. A 43-year-old white male presents to the emergency room for intractable nausea and vomiting. During inpatient admission paperwork, you complete his VTE risk assessment and realize he is a high VTE risk. What would you choose for DVT prophylaxis (BMI = 45, CrCl = 75 ml/min)?
A) Early ambulation
B) Lovenox 40 mg SQ daily
C) Lovenox 40 mg SQ BID
D) Sequential compression device
E) Lovenox 100 mg SQ daily

Answer: C

11A 61-year-old man has a non-ST-elevation MI and is admitted to the coronary care unit. The following day, he develops bradycardia but no symptoms. His blood pressure is 126/84 mm Hg, pulse 50/min, and on examination, the heart sounds are normal, with no extra sounds or rubs. His ECG has changed. Which of the following ECG findings is the best indication for this patient to receive a pacemaker?
A) persistent bradycardia
B) second-degree AV block Mobitz type I
C) first-degree AV block
D) new right bundle branch block
E) left bundle branch block (LBBB) and second-degree AV block Mobitz type II

Answer: E

12A79-year-old man on quinidine for paroxysmal atrial fibrillation develops thrombocytopenia. Which of the following is the most likely mechanism for this syndrome?
A) it is due to bone marrow suppression
B) it is due to sequestration in the spleen
C) it is due to intravascular destruction of platelets
D) it is immunologically mediated
E) there is cross-reactivity with penicillin

Answer: D

13A 28-year-old man develops viridans group streptococci septicemia. Which of the following cardiac lesions has the highest risk of developing endocarditis?
A) ventricular septal defect
B) atrial septal defect, secundum type
C) mitral valve prolapse with regurgitation
D) pure mitral stenosis
E) asymmetric septal hypertrophy

Answer: A

14A 61-year-old man is taking over-the-counter pseudoephedrine for cold and flulike symptoms. Over the course of the next few days, he experiences improvement in his rhinitis but should be concerned about the possibility of which of the following problems?
A) Contraction of the iris causing visual changes
B) Constriction of the bronchioles causing increased pulmonary secretions
C) Erectile function improvement
D) Thinning of his salivary glandular secretions
E) Urinary retention

Answer: E

15A 45-year-old woman develops symptoms of shortness of breath on exertion, easy fatigue, and jaundice. On examination she is pale, and there is a palpable spleen, but no lymphadenopathy. Her hemoglobin is 9.0 g/dL, the reticulocyte count 9%, and the direct antibody test (Coombs’) is positive. Which of the following bone marrow findings is most likely
to be seen in this patient?
A) megaloblastic changes
B) giant metamyelocytes
C) increased erythroid-to-myeloid ratio
D) increased lymphocytes
E) shift to left of the myeloid series

Answer: C

16A 68-year-old man presents with symptoms and signs of CHF. Which of the following is a contraindication to use furosemide?
A) has hypoalbuminemia
B) is oliguric
C) has acidosis
D) had a rash with trimethoprimsulfamethoxazole
E) is on anticoagulants

Answer: D

17. Which of the following features of barbiturate induced coma is most likely correct?
A) requires at least 20–30 times the full sedative dose
B) is increasing in frequency
C) is characterized by an initial period of hyperventilation
D) causes death by depression of the cardiovascular system
E) causes death by pulmonary complications

Answer: E

18A 62-year-old man presents with dark tarry stools and light-headedness. Upper endoscopy finds an ulcerating lesion in his stomach and biopsies confirm gastric cancer. Which of the following is a risk factor for carcinoma of the stomach?
A) Helicobacter pylori infection
B) high socioeconomic status
C) high protein diet
D) high alcohol consumption
E) high fat diet

Answer: A

19An agitated and nervous 24-year-old woman has had severe wheezing and shortness of breath for 2 days. After receiving oxygen, steroids, and salbutamol (Ventolin) in the emergency room, her breathing improves. She is still wheezing and now feels tremulous and anxious with a pulse of 110/min and respirations 30/min. Arterial blood gases on oxygen reveal a pH of 7.40, PO2 340 mm Hg, PCO2 40 mm Hg, and bicarbonate of 24 mEq/L. She is hospitalized for further treatment. Which of the following treatments or medications should be avoided in her?
A) theophylline
B) sedatives
C) corticosteroids
D) sympathomimetic amines
E) intravenous (IV) fluids

Answer: B

20A 63-year-old man developed a transient episode of vertigo, slurred speech, diplopia, and paresthesias. He is symptom-free now, and clinical examination is entirely normal. His past
medical history is significant for hypertension and dyslipidemia. Which of the following is the most likely cause for symptoms?
A) posterior circulation transient ischemic attack (TIA)
B) anterior communicating artery aneurysm
C) hypertensive encephalopathy
D) pseudobulbar palsy
E) occlusion of the middle cerebral artery

Answer: A

21A 19-year-old man is recently diagnosed with type 1 diabetes and attends your clinic to ask about possible complications in the future. He mentions an uncle who has end-stage renal disease due to poorly controlled diabetes and specifically inquires about testing for early signs of renal impairment. The most appropriate investigation is:
A) Blood pressure
B) Microalbuminuria
C) Serum creatinine
D) Serum electrolytes
E) Urine dipstick for glucose

Answer: B

22Which of the following features is least likely to be found in a patient with tuberous sclerosis? 
A) Adenoma sebaceum 
B) Cafe-au-lait spots 
C) Retinal hamartomas 
D) 'Ash-leaf' spots 
E) Lisch nodules

Answer: E

23Which of the following statements is true about gastric lavage?
A) Except in extraordinary circumstances it should be done only in the first 1.5 hours after an overdose.
B) Patients who have had gastric lavage have higher incidence of pulmonary aspiration than patients who have not.
C) The maximum volume that should be used is 5 liters.
D) It can push pill fragments beyond the pylorus.
E) All of the above are true

Answer: E

24Which ONE of the following is true regarding acute pulmonary embolism?
A) a normal ECG excludes the diagnosis
B) embolectomy is more effective than thrombolysis in improving survival
C) Heparin is as effective as thrombolytic therapy
D) the presence of hypoxaemia is an indication for thrombolysis
E) thrombolysis administered through a peripheral vein is as effective as through a pulmonary artery catheter

Answer: E

25A 32-year-old woman is admitted to the intensive care unit (ICU) for sepsis due to pyelonephritis. Her serum creatinine is 0.5 mg/dL at time of admission. 24-h later her urine output started to decline, but her creatinine remains stable. The intensivist feels that the patient is in the process of developing acute kidney injury (AKI). Which one of the following has been clinically validated as a biomarker for the early diagnosis of AKI?
A) Neutrophil gelatinase-associated lipocalin (NGAL)
B) N-acetyl-β-D-galactosaminidase (NAG)
C) Kidney injury molecule-1 (KIM-1)
D) Interleukin-18 (IL-18)
E) None of the above

Answer: E

26A 51-year-old man with severe, recurrent, and extensive peptic ulceration is given a diagnosis of Zollinger–Ellison syndrome. Which of the following is true with respect to this syndrome? 
A) Common cause of peptic ulcer disease 
B) Never malignant 
C) Should be treated by gastrectomy 
D) Diarrhoea can be the presenting feature
E) Octreotide is contraindicated

Answer: D

27For which of these overdoses is charcoal NOT indicated?
A) Acetaminophen
B) Aspirin
C) Iron
D) Digoxin
E) Opiates

Answer: C

28. Whichof the following antimicrobialsis associated with prolongation of the QT interval?
A) Coamoxiclav
B) Gentamicin
C) Cefuroxime
D) Erythromycin
E) Isoniazid

Answer: D

29Whichof the following is associated with Hyperuricaemia?
A) is usually due to an excess purine consumption
B) occurs in association with acute lymphoblastic leukaemia
C) in primary gout is inherited in an autosomal dominant manner
D) can be reduced with low dose aspirin therapy
E) can be treated with uricosuric drugs even in renal failure

Answer: B

30In an investigation for lung malignancy, all of the following may produce a false positive result on a PET-CT except:
A) Pulmonary hamartoma
B) Intralobar sequestration
C) Tuberculosis
D) Pneumonia
E) Scarring

Answer: B

31A 38-year-old woman presents with bilateral pitting ankle oedema. The jugular venous pressure is not raised and the hepatojugular reflux was negative. The woman's ankle oedema would not be explained by which of the following conditions? 
A) Pelvic tumour 
B) Amlodipine therapy 
C) Cyclic oedema 
D) Hypoalbuminaemia 
E) Right heart failure

Answer: E

32A 35-year-old healthy man was seen for an annual medical review as part of his company employment policy. Examination reveals oral thrush in an otherwise fit and healthy man. Further tests confirm oral candidiasis. He denies any illness and he is receiving no drugs. Which one of the following is the most likely underlying disease? 
A) Lymphoma 
B) Alcoholism 
C) HIV infection 
D) Chronic renal failure 
E) Heroin addiction

Answer: C

33. A 40-year-old Egyptian tourist is admitted with haematemesis. He is known to have chronic liver disease caused by Schistosoma mansoni infection. Physical examination would reveal which one of the following clinical signs? 
A) Splenomegaly 
B) Jaundice 
C) Ascites 
D) Spider nevi 
E) Gynaecomastia

Answer: A

34A 68-year-old man presents with recent onset headache and vomiting. Ophthalmoscopy reveals optic atrophy in the right eye and papilloedema in the left one. The most likely diagnosis is? 
A) Pinealoma 
B) Medulloblastoma 
C) Acoustic neuroma 
D) Sphenoid wing meningioma 
E) Pituitary adenoma

Answer: D

35. A 62-year-old man presented with several non-healing ulcerated lesions on the lower extremities and hand, present for approximately three years. He relates that each lesion began as a tender erythematous papule that subsequently ulcerated. The lesions have failed to respond to topical steroids and topical antifungal agents. He received skin grafts over the lower extremity lesions one year ago but the grafts ulcerated. There are multiple ulcerate lesions bilaterally on the lower extremities and one on the right hand. The lesions have dusky purple margins and a halo of erythema. They range in size from 2.5 to 6 cm in diameter. The bases of the ulcers are verrucous with regions of necrosis and granulation tissue. The most likely diagnosis is 
A) Venous insufficiency 
B) Pyoderma gangrenosum 
C) Tropical ulcer 
D) Bacterial skin infection 
E) Ischaemic ulcers

Answer: B

36A 50-year-old accountant is referred for investigation of fever and general ill health. His initial renal function tests show: urea 45 mmol/I, creatinine 480 umol/l. Which one of the following medication dose interval should be most prolonged when used in this patient? 
A) Gentamicin 
B) Amoxicillin 
C) Vancomycin 
D) Penicillin 
E) Cefuroxime

Answer: C

37A 58-year-old chronic alcoholic was admitted with increasing shortness of breath and a distended abdomen. Physical examination revealed evidence for ascites and pitting oedema in both legs. The pulse was 94 bpm, blood pressure 110/60 mmHg and the jugular venous pressure was not raised. The condition least likely to be responsible for this clinical presentation is 
A) Alcoholic cardiomyopathy 
B) Liver cirrhosis 
C) Bowel cancer with liver metastasis 
D) Tuberculous peritoniti
E) Portal vein thrombosis

Answer: A

38A 24-year-old male inpatient appears confused and is behaving in an unusual manner on the ward. Nursing staff report a disrupted sleep pattern. The patient has no psychiatric history. You wish to exclude an organic cause for his behaviour and order an electroencephalogram (EEG). In which of the following disorders are EEG abnormalities least expected? 
A) Herpes simplex encephalitis 
B) Interictal psychosis 
C) Metabolic encephalopathy 
D) Hebephrenic schizophrenia 
E) Subacute sclerosing panencephalitis

Answer: D

39A 72-year-old woman is admitted to the Coronary Care Unit with an acute inferior myocardial infarction. During ECG recording it is noticed that her heart rate is 40 bpm with sinus rhythm and blood pressure is 87/55 mmHg. The most appropriate immediate action is 
A) Keep monitoring the pulse and the blood pressure for a further 24 hours 
B) Insert temporary pacemaker 
C) Organise emergency percutaneous coronary angioplasty 
D) Give 24-hour isoprenaline infusion 
E) Give intravenous atropine sulphate (0.6 mg)

Answer: E

40An 80-year-old man is referred from a nursing home with fever, rigors, nausea and vomiting. He is bedridden and has sacral decubitus ulcers. Blood cultures grow Streptococcus milleri. The most likely site of infection is? 
A) Sacral bedsores 
B) Urinary tract infection 
C) Liver abscess 
D) Endocarditis 
E) Osteomyelitis

Answer: C

41A 75-year-old lady presents with persistent diarrhoea and weight loss. Twenty years previously she had radiotherapy for carcinoma of the ovary. Thirty years ago she lived in Hong Kong for 18 months. Her father died of cancer of the colon aged 62 years.
Hb 9.4 g/dL (11.5-16 g/dL)
WCC 9.6 x 109/L (4-11 x 109/L)
Platelets 234 x 10/L (150-400 x 109/L)
B12 153 pmol/L (160-900pmol/L)
Red cell folate 21 nmol/L (130-630 nmol/L)
Ferritin 5 ug/L (20-250 ug/L)
Erythrocyte sedimentation rate (ESR) 36 mm/hour 
Which of the following is the most likely diagnosis?
A) Radiation enteritis
B) Carcinoma of the colon
C) Crohn's disease 
D) Tropical sprue
E) Acquired lactose intolerance

Answer: A

42A 50-year-old male with a history of alcohol dependence and chronic hepatitis C presents to your clinic with three months of fatigue, weakness and weight loss. He has also noted some “yellowing of my eyes.” You suspect cirrhosis. All the following are clinical signs of cirrhosis except
A) Arthralgia
B) Asterixis
C) Dupuytren’s contracture
D) Haemorrhoids
E) Testicular atrophy

Answer: A

43A 64-year-old who suffers from haemochromatosis is seen in A&E with a 2-day history of pain and swelling in his right knee. On examination it is swollen and he has decreased range of movement. You aspirate his knee. What will the aspirate be most likely to show?
A) Gram-positive cocci in clusters
B) Positively bifringent crystals
C) Gram-positive cocci in chains
D) Inflammatory cells with haemosiderin deposition
E) Negatively bifringent crystals

Answer: B

44A 76-year-old gentleman is referred with an international normalized ratio (INR) of 8.4. He normally takes warfarin for a prosthetic mitral valve. He had an epistaxis lasting 15 minutes this morning. He is otherwise asymptomatic and haemodynamically stable. Having stopped his warfarin for tonight, Which of the following is the most appropriate course of action?
A) 5 mg iv vitamin K
B) 0.5 mg orally (po) vitamin K
C) 4 units of fresh frozen plasma iv
D) 50 units/kg of prothrombin complex concentrate
E) Recheck INR in 24 hours

Answer: E

45. Concerning adrenal physiology, which of the following hormones is secreted by the zona glomerulosa?
A) Adrenaline
B) Noradrenaline
C) Cortisol
D) Aldosterone
E) Angiotensin II

Answer: D

46. A 33-year-old female is seen in the maternity department.she suffers from SLE. Her child was born with profound bradycardia. Which of the following antibodies is most likely to be present?
A) Anti-Ro
B) Anti-La
C) Anti-Jo
D) Anti-Scl-70
E) Antiphospholipid antibodies

Answer: A

47A 67-year-old man presents with nausea and pain in his right upper quadrant. On examination, he has tender hepatomegaly with an irregular edge. His blood tests are as follows:
Hb 8 g/dL (11.5-16 g/dL)
WCC 17.8 x 109/L (4-11 x 109/L)
Platelets 104 x 109/L (150-400x109/L)
Neutrophils 68%
Normoblasts 8%
Myeloblasts 8%
Myelocytes 5%
Metamyelocytes 5%
Lymphocytes 15%
What is the most likely cause of his anaemia?
A) Gaucher's disease
B) Leukaemoid reaction secondary to acute infection
C) Myelosclerosis
D) Malignant infiltration of bone marrow 
E) Osteopetrosis

Answer: D

48A 32-year-old woman presents with left loin pain and haematuria. She is known to suffer with Crohn's disease.
Which of the following is likely to be the aetiology of this presentation?
A) Hypercalciuria
B) Hyperbilirubinaemia
C) Hyperuricaemia
D) Hyperoxaluria
E) Type 1 renal tubular acidosis

Answer: D

49Regarding glucocorticoid-induced hyperglycemia all the following statements are correct EXCEPT.
A) The risk of new onset diabetes mellitus is related to the total glucocorticoid dose and duration of therapy.
B) Raised fasting plasma glucose is the earliest abnormality. 
C) Reduced insulin sensitivity is the predominant mechanism.
D) Insulin is the most effective and safety therapy
E) Long term use of thiazolidinediones in combination with glucocorticoid is contraindicated.

Answer: B

50A 65-year-old man has the following blood results on admission to hospital:
Na 132 mmol/L (135-145 mmol/L)
K 8.6 mmol/L (3.5-5 mmol/L)
Urea 42.4 mmol/L (2.5-6.7 mmol/L)
Creatinine 1178 um0l/L (70-150 umol/L)
The patient appears unwell and the ECG shows tented T waves, absent P waves and widened QRS. Which of the following therapeutic interventions should be performed first?
A) Immediate haemodialysis
B) Intravenous insulin and dextrose
C) Intravenous calcium gluconate
D) Nebulized salbutamol
E) Oral calcium resonium

Answer: C

MULTIPLE CHOICE QUESTIONS-11

1. A 72-year-old male had the onset, within the last 60 minutes, of right hemiparesis and expressive aphasia. Which of the following is the most important variable to monitor on this patient while a decision is being made regarding whether to employ thrombolytic therapy? 
A) WBC 
B) Hemoglobin 
C) Blood sugar 
D) Serum electrolytes 
E) Oxygenation status

Answer: E

2. A 29-year-old woman with a history of asthma presents with progressive worsening of respiratory function. She reports that her symptoms have developed gradually over the past few months, with occasional fevers and episodes of mild hemoptysis. She denies any weight loss, skin changes, or diarrhea. Her medications include inhaled albuterol as needed and OCPs. She has no family history of cancer, no recent travel, and no recent sick contacts. She works as an accountant and does not smoke or use any illicit drugs. On examination, she has a low-grade fever with scattered wheezes and rhonchi over bilateral lung fields. After coughing vigorously, a brown mucus plug is expectorated. Her laboratory values show eosinophilia with an elevated total serum IgE. A chest x-ray shows interstitial infiltrates in the upper lobes with some areas of atelectasis bilaterally, and a CT scan shows enlarged airways primarily in the upper lobes with bronchial wall thickening. Which of the following is the most likely diagnosis?
A) Tuberculosis
B) Worsening asthma
C) Strongyloides stercoralis
D) Allergic bronchopulmonary aspergillosis
E) Bronchial carcinoid tumor

Answer: D

3. All of the following are associated with increased levels of total T4 in the plasma with a normal free T4 EXCEPT:
A) Cirrhosis
B) Pregnancy
C) Sick-euthyroid syndrome
D) Familial dysalbuminemic hyperthyroxinemia
E) Familial excess thyroid-binding globulin

Answer: C

4. A 49-year-old female with a 5-year history of diabetes mellitus type 2 presents for an initial visit. She has no known complications of diabetes. She takes metformin, glyburide, andaspirin. On examination, you find a pleasant, obese female in no distress. Her blood pressure is 136/86 mm Hg, pulse 86, respirations 14, and temperature 37°C. As you discuss monitoring her diabetes, you recommend screening for early kidney disease. Which of the following approaches is the recommended way to screen for diabetic kidney disease?
A) Obtain a 24-hour urine collection for albumin now and again in 3 years.
B) Obtain a spot urine microalbumin every year.
C) Obtain a spot urine microalbumin/creatinine ratio every year.
D) Obtain a urinalysis every year.
E) Obtain a serum creatinine every year.

Answer: C

5. Risk factors for foot ulcers in patient with diabetes include all the following EXCEPT:
A) Foot deformity.
B) Peripheral neouropathy.
C) Tobacco smoking.
D) Female sex.
E) Poor glycemic control.

Answer: D

6. A patient with ascites is suspected to have secondary hyperaldosteronism. Which one of the following would be typical levels of electrolytes in an aliquot specimen of urine?
A) Sodium 2 mEq/L, potassium 40 mEq/L
B) Sodium 5 mEq/L, potassium 0 mEq/L
C) Sodium 40 mEq/L, potassium 40 mEq/L
D) Sodium 80 mEq/L, potassium 2 mEq/L
E) Sodium 100 mEq/L, potassium 20 mEq/L

Answer: A

7. Which of the following conditions should be considered if both the PT and PTT are prolonged in a patient noted to be oozing from a surgical incision?
A) Severe liver disease, DIC, factor X deficiency.
B) Heparin effect, von Willebrand disease, factor XII deficiency.
C) Warfarin effect, factor VII deficiency, vitamin K deficiency.
D) All of the above.

Answer: A

8. A 27-year-old African-American woman with diabetes presents to the outpatient Neurology clinic with subacute onset of bilateral facial weakness. She explains that the symptoms have developed over the course of the last few days. She also reports having a slightly raised and tender rash over the anterior aspects of both shins. On examination you find bilateral lower motor neuron facial palsy as well as tender erythematous nodules over both shins. Which of the following is the most likely diagnosis?
A) Guillain-Barré syndrome
B) Lyme disease
C) Neurosarcoidosis
D) Diabetes
E) Tuberculosis

Answer: C

9. Which of the following are the cardinal features of idiopathic Parkinson disease?
A) Tremor, bradykinesia, rigidity, and postural instability
B) Bradykinesia, dementia, tremor, and rigidity
C) Rigidity, hallucinations, tremor, and postural instability
D) Tremor, rigidity, bradykinesia, and gaze palsy
E) Tremor, autonomic dysfunction, bradykinesia, and rigidity

Answer: A

10. According to the Eighth Joint National Committee (JNC-8) guideline on managing hypertension in adults, Initial Drugs of Choice for Hypertension include all of the following EXCEPT: 
A) ACE inhibitor 
B) Angiotensin receptor blocker 
C) Thiazide diuretic 
D) Calcium channel blocker 
E) β-blockers

Answer: E

11. A 42-year-old male who works in a hog confinement area presents to your office complaining of cough, fever, wheeze, and dyspnea. He and some other workers were cleaning the confinement area with high-pressure hoses (which aerosolized hog waste), and they all with the same symptoms, which started between 4 and 8 hours after work. On examination, he is febrile with a respiratory rate of 28. He is able to talk in complete sentences. There are slight crackles when you auscultate the lungs. His chest x-ray is normal. The most likely diagnosis is:
A) “Farmer's lung” (hypersensitivity pneumonitis).
B) Organic dust toxicity syndrome.
C) Reactive airway disease.
D) Hydrogen sulfide poisoning.
E) Bronchiolitis obliterans.

Answer: B

12. Which of the following statements best describes Graves’ ophthalmopathy?
A) Although a cosmetic problem, Graves’ ophthalmopathy is rarely associated with major ocular complications.
B) Diplopia may occur from periorbital muscle swelling.
C) It is never found without concomitant hyperthyroidism.
D) The most serious complication is corneal abrasion.
E) Unilateral disease is not found.

Answer: B

13. A 20-year-old woman with no significant past medical history presents with a 2-month history of episodic shortness of breath. These symptoms began with an upper respiratory tract infection. She has fits of coughing and trouble catching her breath with exertion. She states that her breath “sounds like whistles” at times. She tried a friend's albuterol inhaler with some improvement and wonders if she has asthma. On exam, she is breathing comfortably at 16 times per minute and her oxygen saturation is 96% on room air. Her lungs are clear to auscultation, and the remainder of her exam is unremarkable. You want to better categorize this patient's disease. Which of the following tests is most appropriate to order now?
A) Spirometry.
B) Chest x-ray.
C) Arterial blood gas (ABG).
D) Methacholine challenge.
E) Chest CT.

Answer: A

14. Ataxia may be a manifestation of which vitamin deficiency?
A) Vitamin A
B) Vitamin B
C) Vitamin C
D) Vitamin D
E) Vitamin E

Answer: E

15. A 34-year-old woman with diabetes mellitus, hypertension, dyslipidemia, a family history of premature coronary artery disease, and ischemic cardiomyopathy with an ejection fraction of 38% by recent radionuclide angiography is contemplating pregnancy and seeks your advice. She is completely asymptomatic and physical examination demonstrates no evidence of hypervolemia. She is taking low-dose aspirin, a beta-blocker, an ACE inhibitor, and a hydroxymethylglutaryl-coenzyme A reductase inhibitor (statin). Which of the following is the most appropriate statement?
A) Advise the patient to discontinue the ACE inhibitor and pursue pregnancy
B) Advise the patient to discontinue the ACE inhibitor and statin, and pursue pregnancy
C) Advise the patient to discontinue the aspirin and ACE inhibitor and pursue pregnancy
D) Advise the patient to discontinue the beta-blocker and statin and pursue pregnancy
E) Advise the patient not to become pregnant.

Answer: E

16. An asymptomatic 55-year-old male visits a health fair, where he has a panel of blood tests done. He brings the results to you because he is concerned about the TSH level of 12.0 µU/mL (N 0.45-4.5). His free T4 level is normal. Which one of the following is most likely to be associated with this finding?
A) Atrial fibrillation 
B) Reduced bone density 
C) Systolic heart failure 
D) Elevated LDL cholesterol 
E) Type 2 diabetes mellitus.

Answer: D

17. An 18-year-old man with a 12-year history of type 1 diabetes mellitus comes to the physician for a follow-up examination. Medications include 25 U of NPH insulin and 10 U of regular insulin in the morning and 10 U of NPH insulin and 10 U of regular insulin before dinner. His hemoglobin A1c was 14.5% 12 weeks ago. His current pulse is 80/min, respirations are 20/min, and blood pressure is 145/95 mm Hg. Examination shows scattered retinal microaneurysms bilaterally. The remainder of the examination shows no other abnormalities. Laboratory studies show:
Hemoglobin A1c 13%
Serum Na+ 130 mEq/L
K+ 3.2 mEq/L
Cl− 101 mEq/L
HCO3− 23 mEq/L
Glucose 325 mg/dL
Creatinine 1.5 mg/dL
Cholesterol 350 mg/dL
Urine Blood negative
Glucose 4+
Protein 1+
Ketones Negative
Which of the following is the most likely renal diagnosis?
A) Cholesterol renal emboli
B) Diabetic nephropathy
C) Hypertensive glomerulosclerosis
D) Hypokalemic nephropathy
E) Sodium-losing nephropathy.

Answer: B

18. Which of the following is increased in patients with emphysema- dominant COPD?
A) DLCO
B) FVC
C) FEV1
D) RV
E) Alpha-1 antitrypsin

Answer: D

19. An 18-year-old man with a childhood diagnosis of small bowel Crohn’s disease attended the eye hospital Emergency Department with a painless, erythematous right eye. His vision was reported to be unchanged. What is the most likely diagnosis?
A) Anterior uveitis
B) Episcleritis
C) Intermediate uveitis
D) Posterior uveitis
E) Scleritis.

Answer: B

20. A 44-year-old woman undergoes preoperative evaluation prior to surgery to repair a congenital defect of her pelvis. Her expected blood loss is 2.0 l. She has a prior history of severe anaphylactic reaction to a prior erythrocyte transfusion that she received for postpartum hemorrhage at age of 27 years. In addition she has a history of rheumatoid arthritis. On physical examination, the temperature is 36.8 °C (98.5 °F), blood pressure is 140/70 mm Hg, and heart rate is 76 bpm. Laboratory studies indicate a hemoglobin level of 12.0 g/dL, a leukocyte count of 6500 μL, and a platelet count of 150,000 μL. Previous laboratory studies indicate an IgG level of 800 mg/dL and an IgM level of 65 mg/dL. Which of the following is the most appropriate erythrocyte transfusion product for this patient?
A) Leuko-reduced blood
B) Cytomegalovirus-negative blood
C) Irradiated blood
D) Phenotypically matched blood
E) Washed blood

Answer: E

21. A 75-year-old man underwent surgery to correct a large abdominal aortic aneurysm. The procedure appeared to go well, but you are called a few hours later to evaluate the patient who states that he cannot move or feel his legs. On the way to the ICU, you consider the possible causes of his symptoms and plan your physical examination. What is the most important test to help localize the lesion?
A) MRI of the spine
B) Sensory level
C) Reflexes in lower extremities
D) Plantar flexion reflex
E) Toe position sense.

Answer: E

22. A 35-year-old man is suspected of having a small right-sided pleural effusion. What imaging modality is most sensitive to detect a small amount of pleural fluid?
A) chest CT
B) lateral chest film
C) left lateral decubitus chest film
D) standard upright chest film

Answer: A

23. A 70-year-old man with history of HTN and DM presents with a 2-month history of increasing paroxysmal nocturnal dyspnea and SOB with minimal exertion. An echocardiogram shows an ejection fraction of 25%. Which one of the patients current medications should be discontinued?
A) Lisinopril 
B) Pioglitazone 
C) Glipizide 
D) Metoprolol

Answer: B

24. A 40-year-old male is diagnosed with active TB. In educating the pt, he is told to avoid tyramine containing foods and the possibility of the development of hepatitis and peripheral neuropathy. Which of the following medication is the pt being educated about?
A) rifampin
B) Isoniazid
C) Pyrazinamide
D) ethambutol
E) streptomycin

Answer: B

25. A 32-year-old man with family history of HTN measures his own BP daily and keeps in his diary. He visits his primary care physician, and his BP is found to be 124/72 mm Hg, which is much lower than his home BP. He insists that his BP be measured 2 h later, which was found to be 125/73 mm Hg. He asks you to clarify about this discrepancy between office and home BP measurements, and you would say that he has:
A) Essential HTN
B) Prehypertension
C) White coat HTN
D) Masked HTN
E) Secondary HTN

Answer: D

26. An 88-year-old man is taken to A&E by his daughter as he had been feeling unwell that day with chest pains. An ECG reveals that he is suffering an inferior myocardial infarction. He has a heart rate of 33 bpm and blood pressure of 90/60 mmHg. He is given 600mcg of atropine to no effect, this is repeated up to 3mg. You commence an infusion of TPA but his rate is not immediately improving. Which of the following treatments would be most suitable to use for his arrhythmia?
A) Adrenaline
B) Glucagon
C) Temporary pacing
D) Dopamine
C) Dobutamine

Answer: C

27. All of the following are common manifestations of cytomegalovirus (CMV) infection following lung transplantation EXCEPT:
A) bronchiolitis obliterans
B) CMV esophagitis
C) CMV pneumonia
D) CMV retinitis
E) CMV syndrome (fever, malaise, cytopenias, transaminitis, and CMV viremia)

Answer: D

28. A 25-year-old man presents to the ED with a broad complex tachycardia that is irregularly irregular. The patient is haemodynamically uncompromised. An anaesthetist is not available to assist with immediate DC cardioversion. What is the best initial treatment option?
A) IV adenosine
B) IV verapamil
C) Oral beta-blocker
D) IV beta-blocker
E) IV flecainide

Answer: E

29. A young patient is diagnosed with multiple endocrine neoplasia (MEN) type 3 (also known as type 2b) after an episode of bowel obstruction. Which one of the following features would he be unlikely to have or develop in the future with this diagnosis?
A) Medullary carcinoma of the thyroid
B) Marfanoid appearance
C) Mucosal neuromas of the small bowel
D) Facial angiofibromas
E) Prognathism

Answer: D

30. All the following types of cancer commonly metastasize to the central nervous system (CNS) EXCEPT:
A) ovarian
B) breast
C) hypernephroma
D) melanoma
E) acute lymphoblastic leukemia (ALL)

Answer: A

31A 32-year-old woman presents with diarrhea, with mucus and blood in the stool, and recurrent abdominal pain. Colonoscopy is performed and a mucosal biopsy is obtained. Which one of the following pathological findings would make ulcerative colitis a more likely diagnosis than Crohn's disease? 
A) Ileal involvement 
B) Crypt abscesses 
C) Transmural involvement 
D) Granulomas 
E) Skip lesions

Answer: B

32A 61-year-old man is admitted because of increasing confusion and inadequately controlled lower backache. He is found to have multiple myeloma. Which of the following statements regarding this patient's disease is correct? 
A) Osteoporotic vertebral collapse fracture may be responsible for his backache 
B) Radiotherapy is ineffective in treating bone pain 
C) Isotope bone scanning is more sensitive for identifying early disease 
D) Plain radiographs of the spine would show typical widespread osteoblastic bone lesions 
E) The hypercalcaemia is usually steroid-resistant

Answer: A

33. A 64-year-old builder who had a surgical operation for inguinal hernia a develops a wound infection a week later and methicillin-resistant Staphylococcus aureus (MRSA) is recovered from the wound. Which one of the following isolation techniques do you think is appropriate in this condition? 
A) Enteric precautions 
B) Blood and body fluid precautions 
C) Respiratory isolation only 
D) Strict isolation 
E) Contact isolation

Answer: E

34A teacher has brought his son to you because he is very worried that his son might be suffering with haemophilia. Which one of the following best excludes a diagnosis of haemophilia? 
A) Normal activated partial thromboplastin time 
B) Normal prothrombin time 
C) Absence of excessive bleeding after tooth extraction 
D) Absence of spontaneous bleeding episodes 
E) Absence of family history

Answer: C

35A 20-year-old car mechanic who has been abusing inhalants for many years was admitted with sudden collapse. The ambulance crew said he was inhaling solvent by rebreathing from a plastic bag. He is transferred to the Intensive Care Unit. A few minutes later he stops breathing and the cardiac monitor records ventricular fibrillation rhythm. In resuscitating this patient which one of the following should be avoided? 
A) Direct current shock 
B) Intubation 
C) Oxygen therapy 
D) Intravenous ß-blocker 
E) Intravenous adrenaline (epinephrine)

Answer: E

36An 88-year-old woman is referred because of a tense bullous eruption on the inner sides of the upper and lower limbs as well as the abdomen. The most likely diagnosis is 
A) Pemphigus 
B) Bullous pemphigoid 
C) Herpes zoster 
D) Dermatitis herpetiformis 
E) Stevens-Johnson syndrome

Answer: B

37A 40-year-old male dancer presents with thrombophlebitis of the lower limb. Examination reveals five painless, punched-out ulcers on the scrotum with concomitant oral ulcers. Which of the following is the most likely diagnosis? 
A) Granuloma inguinale 
B) Syphilis 
C) Herpes simplex genitalis 
D) Belicet's disease 
E) Systemic lupus erythematosus

Answer: D

38A 50-year-old woman is admitted with high fever and a generalised headache. Examination reveals chemosis and cyanosis of the upper face. There was proptosis and ophthalmoplegia of the left eye. Pin prick sensation was lost from the left forehead. The most likely diagnosis is 
A) Periorbital cellulitis 
B) Osteomyelitis of the left maxillary sinus 
C) Left retro-orbital tumour 
D) Cavernous sinus thrombosis 
E) Superior sagittal sinus thrombosis

Answer: D

39A 59-year-old lady is being investigated for haematuria. A diagnosis of glomerulonephritis is suspected. Which one of the following tests is not indicated?
A) X-ray kidney, ureter, bladder (KUB).
B) Urine microscopy.
C) Serum and urine immunoelectrophoresis.
D) Complement levels.
E) Antineutrophil cytoplasmic (ANCA) and antiglomerular basement membrane (anti-GM) antibodies.

Answer: A

40Which of the following is incorrect regarding specific treatment of overdose?
A) Naloxone for opiate overdose is often given as an infusion due to its short half-life.
B) Flumazenil can be given for benzodiazepine overdose.
C) N-acetylcysteine for paracetamol overdose can only be given intravenously.
D) Oral methionine is effective in aspirin overdose.
E) ’Digibind’ is a fragment of digoxin-specific antibody given in severe overdose.

Answer: D

41A 55-year-old man with chronic hepatitis C is being considered for liver transplantation. The patient has cirrhosis that was documented by liver biopsy 10 years ago. For the past 3 months, he has had ascites and edema, which are poorly controlled with diuretics. Lactulose was recently begun because of confusion. Which of the following combinations of laboratory studies will be most helpful in estimating his survival over the next 6 months? 
A) Serum total bilirubin and INR 
B) Serum aspartate aminotransferase and gamma globulin 
C) Serum alanine aminotransferase and hepatitis C RNA (HCV RNA) 
D) Serum alkaline phosphatase and ammonia 
E) Serum albumin and ?-glutamyltransferase

Answer: A

42Signs and symptoms of opioid withdrawal include all of the following EXCEPT
A) Increased blood pressure (BP) and heart rate
B) Seizures
C) Abdominal cramps
D) Jerking of the legs
E) Hyperthermia

Answer: B

43Which of the following β-adrenergic antagonists is a nonselective β1 and β2 blocker?
A) Atenolol
B) Betaxolol
C) Esmolol
D) Metoprolol
E) Nadolol

Answer: E

44In primary hypothyroidism increased requirements for L thyroxine occur in the following conditions EXCEPT:
A) Nephrotic syndrome with massive proteinuria.
B) Pregnancy.
C) Achalasia
D) Helicobacter pylori associated chronic gastritis.
E) Use of estrogen replacement therapy.

Answer: C

45A 70-year-old man is admitted with new-onset atrial fibrillation with an apical rate of 120/minute and a BP of 100/70 mm Hg. Shortly after admission he develops moderate lower abdominal pain and diarrhea with dark red blood.
Which is the most appropriate course of action?
A) Cardioversion 
B) Unfractionated heparin
C) Mesenteric angiography
D) Laparotomy
E) Digoxin

Answer: A

46An 18-year-old insulin-dependent diabetic is seen in A&E. Over the last 4 days she has suffered from diarrhoea and vomitting, and has cut down on her insulin injections. Her mother has brought her in because she has become drowsy and confused. Her blood results are as follows:
WCC 16.4 X109/L (4-11X109/L)
Hb 17.8 * g/dL (12-16 g/dL)
Platelets 597 X10/L (150-400X109/L)
Na 146 mmpl/L (135-145 mmol/L)
K 6.6 mmol/L (3.5-5 mmol/L)
Urea 8.4 mmol/L (2.5-6.7 mmol/L)
Creatinine 145 umol/L (60-120 umol/L)
Random glucose 26 mmol/L (4.5-5.6 mmol/L)
You are with one nurse in the resuscitation room. What is the first thing you are going to ask her to do? 
A) Start an insulin sliding scale
B) Give 1 L of normal saline over 30 minutes 
C) Run to the lab with blood gases
D) Draw up 10 mL of 10% calcium gluconate
E) Give 100 mmol bicarbonate intravenously (iv)

Answer: B

47A 73-year-old male presents to the clinic with 3 months
of increasing back pain. He localizes the pain to the lumbar
spine and states that the pain is worst at night while he is lying
in bed. It is improved during the day with mobilization.
Past history is notable only for hypertension and remote cigarette smoking. Physical examination is normal. Laboratory
studies are notable for an elevated alkaline phosphatase. A
lumbar radiogram shows a lytic lesion in the L3 vertebra.
Which of the following malignancies is most likely?
A) Gastric carcinoma
B) Non-small cell lung cancer
C) Osteosarcoma
D) Pancreatic carcinoma
E) Thyroid carcinoma

Answer: B

48A 36-year-old African-American woman with systemic lupus erythematosus presents with the acute onset of lethargy and jaundice. On initial evaluation, she is tachycardic, hypotensive, appears pale, is dyspneic, and is somewhat difficult to arouse. Physical examination reveals splenomegaly.
Her initial hemoglobin is 6 g/dL, white blood cell count is 6300/μL, and platelets are 294,000/ μL. Her total bilirubin is 4 g/dL, reticulocyte count is 18%, and haptoglobin is not detectable. Renal function is normal, as is urinalysis. What would you expect on her peripheral blood smear?
A) Macrocytosis and PMN’s with hypersegmented nuclei
B) Microspherocytes
C) Schistocytes
D) Sickle cells
E) Target cells

Answer: B

49A 22 year-old lady presents to her GP with facial swelling, most marked in the morning, and shortness of breath. Urinalysis shows protein +++. Results were as follows: 
24-hour urinary protein 4.2 g; 
Haemoglobin(Hb) 10.8 g/l (11.5-16.0 g/L); 
Urea 22.8 mmol/L (2.5-6.5 mmol/L); 
Creatinine 374 µmol/L (70-150 umol/L)
C3 0.24 g/L (0.55-1.2g/L)
C4 0.29g/L (0.2-0.5 g/L)
Which of the following is the most likely diagnosis?
A) Minimal change nephropathy
B) Systemic lupus erythematosus
C) Rheumatoid arthritis
D) Cryoglobulinaemia
E) Diabetes mellitus

Answer: B

50A 31-year-old male with hemophilia A is admitted with persistent gross hematuria. He denies recent trauma or any history of genitourinary pathology. The examination is unremarkable. Hematocrit is 28%. All the following are treatments for hemophilia A except
A) desmopressin (DDAVP)
B) fresh-frozen plasma (FFP)
C) cryoprecipitate
D) recombinant factor VIII
E) plasmapheresis

Answer: E

MULTIPLE CHOICE QUESTIONS-12

1. When progressive enlargement of a multinodular goiter causes symptomatic
tracheal compression, the preferred management in otherwise good-risk patients
is:
A. Iodine treatment.
B. Thyroid hormone treatment.
C. Surgical resection of the abnormal thyroid.
D. Radioactive iodine treatment.
Answer: C

 

DISCUSSION: When a multinodular goiter enlarges enough to cause symptoms of
tracheal compression, surgical treatment is usually required if the patient is
considered a reasonable operative risk. Medical treatment may be effective in
preventing the initial growth of the goiter but is unlikely to cause enough
regression to relieve symptoms. Radioactive iodine can occasionally be used to
cause some regression in patients who are poor anesthesia risks, but this is a
temporizing treatment rather than a definitive one.


2. The most precise diagnostic screening procedure for differentiating benign
thyroid nodules from malignant ones is:
A. Thyroid ultrasonography.
B. Thyroid scintiscan.
C. Fine-needle-aspiration biopsy (FNAB).
D. Thyroid hormone suppression.
Answer: C

DISCUSSION: Analysis of multiple series in which patients with thyroid nodules
have undergone FNAB has demonstrated a false-negative rate of 2.4% and a
false-positive rate of 3.3%. Sensitivity for this method is 92%; specificity
74%. This surpasses the other methods for accurate selection of patients who
require surgical resection.


3. The preferred operation for initial management of a thyroid nodule that is
considered suspicious for malignancy by FNAB is:
A. Excision.
B. Partial lobectomy.
C. Total lobectomy and isthmusectomy.
D. Total thyroidectomy.
Answer: C

DISCUSSION: There is a consensus that the initial minimum operation for a nodule
suspected to be malignant is total lobectomy and isthmusectomy. Partial
lobectomy or excision of the nodule is associated with a higher risk of local
recurrence if the nodule proves to be malignant. Reoperation on the side of a
partial lobectomy can be technically difficult and associated with a higher risk
of recurrent nerve injury. Ordinarily, total thyroidectomy is not performed
until a conclusive diagnosis of malignancy is established.


4. Advantages of total thyroidectomy for management of papillary carcinomas of
the thyroid larger than 1.5 cm. include:
A. Possibility of using radioactive iodine postoperatively to identify and treat
metastases.
B. The ability to use thyroglobulin levels as a marker for recurrence.
C. Lower overall recurrence rate.
D. Lower risk of hypoparathyroidism.
Answer: ABC

DISCUSSION: Following total thyroidectomy iodine 131 can be used more
efficiently because of the absence of normal thyroid tissue, which has greater
affinity for iodine than papillary carcinoma tissue. When all normal thyroid
tissue is removed, serum thyroglobulin, which is produced by normal and
malignant thyroid tissue, becomes a more effective marker for recurrence. The
overall recurrence rate is lower for patients undergoing total thyroidectomy,
but the risk of hypoparathyroidism is higher for patients who have total
thyroidectomy instead of unilateral lobectomy.


5. Which of the following statements about follicular carcinoma is/are true?
A. It presents at a later age than papillary carcinoma.
B. It disseminates via hematogenous routes.
C. It is the most common type of well-differentiated thyroid carcinoma.
D. Extensive angioinvasion portends a poor prognosis.
E. Follicular carcinomas are frequently multicentric.
Answer: ABD

DISCUSSION: Follicular carcinoma is more common in older patients (peak
incidence in the fifth decade). The tumor has a marked propensity for vascular
invasion and spreads hematogenously to bone, lung, liver, and central nervous
system sites. Local nodal metastases are less common than in papillary
carcinoma. Extensive angioinvasion indicates a less favorable prognosis.
Papillary carcinoma is the most common type of well-differentiated thyroid
carcinomas. Follicular carcinomas are rarely multicentric.


6. A familial form of medullary thyroid carcinoma (MTC) should be suspected
whenever:
A. The tumor is multifocal.
B. The tumor is bilateral (foci of tumor are present in both thyroid lobes).
C. Pathologic examination of the resected thyroid gland reveals the presence of
C-cell hyperplasia in areas of the gland adjacent to foci of MTC.
D. All of the above.
Answer: D

DISCUSSION: Sporadic MTC is unilateral in at least 80% of cases. However, in
patients with MTC occurring as a component of the multiple endocrine neoplasia
(MEN) type 2A or type 2B syndromes, the tumor is virtually always multifocal and
bilateral. Typically, in this setting the MTC appears as multiple whitish-tan
tumor nodules in the middle and upper thirds of each thyroid lobe.
A diffuse premalignant proliferation of the C cells of the thyroid is thought to
precede the development of MTC in patients with familial MTC. This
proliferation, known as C-cell hyperplasia (CCH), consists of parafollicular
clusters of increased numbers of C cells. The finding of CCH in areas of the
thyroid adjacent to gross foci of MTC is strong evidence for familial MTC.


7. All of the following are components of the MEN type 2B syndrome except:
A. Multiple neuromas on the lips, tongue, and oral mucosa.
B. Hyperparathyroidism.
C. MTC.
D. Pheochromocytoma.
Answer: B

DISCUSSION: MTC and pheochromocytoma occur in both MEN 2A and MEN 2B syndromes.
Patients with MEN 2A may also develop hyperplasia of the parathyroid glands.
Although some investigators have reported equivocal histologic abnormalities in
the parathyroid glands of patients with MEN 2B, hyperparathyroidism is not a
component of this syndrome.
In contrast to patients with MEN 2A, those with MEN 2B have a characteristic
phenotype, including a tall, thin “marfanoid” habitus. Patients with MEN 2B also
develop multiple neuromas on the lips, tongue, and oral mucosa, creating the
appearance of thick lips.


8. MEN 2A and MEN 2B syndromes are associated with germline mutations in:
A. The p53 tumor suppressor gene.
B. The H-ras gene.
C. The N-myc gene.
D. The RET proto-oncogene.
Answer: D

DISCUSSION: Germline mutations in the RET protooncogene, a receptor tyrosine
kinase that maps to chromosome 10, are associated with MEN 2A and MEN 2B
syndromes. Homozygous loss of the tumor suppressor gene p53 is associated with
the Li-Fraumeni syndrome, and mutations of p53 are present in a variety of human
neoplasms. Point mutations in the H-ras gene are associated with carcinoma of
the colon, lung, and pancreas. Amplification of the N-myc gene, when present in
neuroblastoma, suggests a poorer prognosis.


9. Which of the following are true concerning islet cell neoplasms of the
pancreas in patients with MEN type 1?
A. Islet cell neoplasms in patients with MEN 1 are characteristically
multicentric.
B. The most common islet cell neoplasm in patients with MEN 1 is gastrinoma.
C. Islet cell neoplasms in patients with MEN 1 may be malignant.
D. All of the above.
Answer: D

DISCUSSION: The pathologic change in the pancreas of patients with MEN 1 is
typically multicentric. Diffuse hyperplasia of islet cells and microadenoma
formation are often identified in areas of the gland distant from grossly
evident tumor. Tumors are commonly multifocal. Islet cell neoplasms of the
pancreas occur in 30% to 80% of patients with MEN 1. The most common islet cell
neoplasm in these patients is gastrinoma. Gastrinomas associated with MEN 1
probably account for 20% to 50% of all cases of the Zollinger-Ellison syndrome.
The second most common islet cell tumor is insulinoma. Other pancreatic islet
cell neoplasms, such as glucagonoma, somatostatinoma, or vasoactive intestinal
polypeptide neoplasm (VIPoma), are rarely associated with MEN 1. Approximately
10% of insulinomas and approximately 15% or more of gastrinomas in patients with
MEN 1 are malignant.


10. Which of the following statements about the differential diagnosis of
hypercalcemia is/are correct.
A. Malignant tumors typically cause hypercalcemia by ectopic production of
parathyroid hormone (PTH).
B. The diagnosis of primary hyperparathyroidism is supported by these serum
levels: calcium, 10.8 mg. per dl.; chloride, 104 mmol. per liter; bicarbonate 21
mmol. per liter; phosphorus, 2.4 mg. per dl.; elevated parathyroid hormone.
C. Familial hypocalciuric hypercalcemia is distinguished from primary
hyperparathyroidism by parathyroid imaging.
D. Although serum albumin binds calcium, the measured total calcium value is
usually unaffected in patients with severe hypoproteinemia.
E. Thiazide diuretics are a good treatment for hypercalcemia and can be given to
patients with apparent hypercalcemia of malignancy.
Answer: B

DISCUSSION: Malignant tumors rarely secrete PTH itself; they can secrete PTHrP
or cytokine activators of osteoclast activity. The diagnosis of primary
hyperparathyroidism is supported by hypercalcemia with mild hyperchloremic
metabolic acidosis and a chloride-phosphate ratio greater than 33 or a modified
chloride (mmol. per liter/mg. per dl.) phosphate ratio greater than 500.
Familial hypocalciuric hypercalcemia is distinguished from primary
hyperparathyroidism by a low urine calcium. Serum calcium changes approximately
0.8 mg. per dl. for every 1 gm. per dl. change in serum albumin. Thiazide
diuretics can cause hypercalcemia and should not be given to patients who are
hypercalcemic.


11. Indications for operation in a patient with previously asymptomatic
hyperparathyroidism include:
A. Age older than 60.
B. Nephrolithiasis.
C. A substantial decline in renal function.
D. A substantial decline in bone mass.
E. Depression and fatigue.
Answer: BCDE

DISCUSSION: Age younger than 50 is considered an indication for operation.


12. The parathyroid glands:
A. Develop from the second and third pharyngeal pouches, along with the palatine
tonsil and the thymus.
B. Migrate caudally in the neck in normal development but can be found anywhere
from the pharyngeal mucosa to the deep mediastinum.
C. Secrete PTH and calcitonin to manage calcium homeostasis.
D. Usually number four, but frequently number only two or three.
E. Contain enzymes that catalyze the conversion of 25(OH) vitamin D 3 to
1,25(OH) 2 vitamin D 3.
Answer: B

DISCUSSION: The parathyroid glands develop from the third and fourth pharyngeal
pouches, along with the thymus and the thyroid ultimobronchial body; there are
four glands in the vast majority of persons. Calcitonin is secreted by the C
cells of the thyroid. Vitamin D 3 hydroxylation occurs in the kidney.


13. Hyperparathyroidism can affect which of the following organs and body
systems?
A. Gastrointestinal tract.
B. Kidneys.
C. Skeleton.
D. Neuromuscular system.
E. Cardiovascular system.
Answer: ABCDE

DISCUSSION: Gastrointestinal involvement includes pancreatitis and peptic ulcer
disease. The kidneys can be affected by nephrocalcinosis or nephrolithiasis. The
bones can be severely affected by bone resorption. Neuromuscular complaints can
include weakness and fatigue. Hypertension is present in as many as 70% of
people with hyperparathyroidism and may be related to the renal disease.


14. Secondary hyperparathyroidism:
A. Is a metabolic disease in which the primary abnormality is decreased
glomerular filtration rate.
B. Is best treated initially by subtotal parathyroidectomy.
C. Is caused by increased production of 1,25(OH) 2 vitamin D 3, causing
increasing intestinal calcium absorption and hypercalcemia.
D. Can have severe effects on bones exacerbated by aluminum contained in
phosphate binders and dialysate water.
E. Is best treated initially by total parathyroidectomy with
autotransplantation.
Answer: AD

DISCUSSION: Secondary hyperparathyroidism is caused by renal disease, including
decreased glomerular filtration rate and decreased 1,25(OH) 2 vitamin D 3
production. It is best treated by medical management restricting dietary
phosphate, administering vitamin D and calcium with phosphate-binding gels, and
limiting aluminum exposure, which can exacerbate bone disease.


15. Hypoparathyroidism:
A. Is most commonly encountered as a postviral syndrome.
B. Can be associated with marked hypocalcemia after parathyroidectomy in
patients with bone disease.
C. Can cause anxiety, depression, or confusion.
D. Can cause physical signs such as Chvostek's and Trousseau's signs.
E. Is treatable acutely with intravenous calcium salts and chronically with oral
calcium and vitamin D.
Answer: BCDE

DISCUSSION: Hypoparathyroidism is most commonly encountered after thyroid
surgery. It can be temporarily severe after parathyroidectomy in patients with
bone disease (“postoperative bone hunger”). The signs and symptoms can include
anxiety, depression, confusion, Chvostek's sign, and Trousseau's sign, as well
as circumoral or extremity tingling, tetany with carpopedal spasms, or seizures.
Treatment is as noted.


16. True statements about pituitary anatomy and physiology include:
A. The pituitary has dual embryonic origin: the anterior pituitary arises from
embryonic ectoderm; the posterior pituitary, from the diencephalon.
B. The hypophyseal portal system integrates function of the anterior and
posterior pituitary.
C. Adrenocorticotropin (ACTH), formed by posttranslational processing of the
precursor POMC, is normally controlled by hypothalamic CRF but may be released
by immune-related mechanisms.
D. Growth hormone (GH) directly stimulates longitudinal growth of the skeleton
and growth of muscles.
E. Cell types of the anterior pituitary are classified by their position in the
anterior pituitary and by their staining characteristics with histologic dyes.
Answer: AC

DISCUSSION: The anterior pituitary arises from Rathke's pouch (embryonic
ectoderm), and it includes the pars distalis, pars intermedia, and pars
tuberalis. The posterior pituitary arises from the diencephalon and includes the
neural stalk, infundibulum, and posterior lobe. The hypophyseal portal system
drains from the hypothalamus and integrates function of the hypothalamus and
anterior pituitary by carrying hypothalamus-derived releasing factors to target
cells in the anterior pituitary. ACTH may be released during stress in response
to interleukin-1, -2, and -6. Somatotrophic actions of GH are indirect and are
mediated by the insulinlike growth factors formerly known as somatomedins.
Current classification of anterior pituitary cells is based on immunochemical
identification of their secretory products (i.e., corticotropes produce ACTH,
lactotropes produce prolactin, and thyrotropes produce thyroid-stimulating
hormone [TSH]).


17. Antidiuretic hormone (ADH):
A. Is related to oxytocin, and both are released from the posterior pituitary in
conjunction with neurophysins.
B. Is released into the circulation by the posterior pituitary in response to a
rise in plasma osmolality above 285 mOsm. or a decrease in circulating blood
volume.
C. May be stimulated by catecholamines and inhibited by phenytoin, alcohol, and
lithium.
D. In excess, may produce a syndrome of euvolemic hyponatremia with
inappropriately concentrated urine that is responsive to free water restriction.
E. Deficiency causes prolonged polyuria and polydipsia and may be diagnosed by a
combination of high plasma osmolality and low urine osmolality following water
deprivation.
Answer: ABCDE

DISCUSSION: ADH and oxytocin are nine–amino acid peptides derived from a common
ancestral peptide, vasotocin. ADH is released from the posterior pituitary with
neurophysin II in response to a rise in plasma osmolality greater than 285
mOsm/kg. H 2O. by a 5% or greater decrease in blood volume, and by
catecholamines. The syndrome of inappropriate ADH release (SIADH) produces
euvolemic hyponatremia and is responsive to free water restriction. Diabetes
insipidus (DI) reflects a deficiency of ADH and causes prolonged polyuria and
polydipsia. Diabetes insipidus is diagnosed by a combination of high plasma
osmolality and low urine osmolality following water deprivation. Exogenously
administered ADH differentiates central (ADH-responsive) DI from nephrogenic
(ADH-unresponsive) DI.


18. Signs and symptoms of acute pituitary apoplexy include:
A. Severe headache.
B. Meningismus.
C. Vision loss.
D. Shock.
E. May be relieved by emergent transsphenoidal decompression of the sella
turcica.
Answer: ABCD

DISCUSSION: Acute pituitary apoplexy follows sudden hemorrhage into a
pre-existing pituitary tumor or following closed head trauma. Symptoms,
including headache, meningismus, and vision loss, are attributable to the
intracerebral blood. Pituitary insufficiency, as well as the accompanying
secondary adrenal insufficiency, may cause hypotension and shock. Other
manifestations may include DI and myxedema. Acute pituitary apoplexy is a
neurosurgical emergency that requires transsphenoidal decompression of the sella
turcica.


19. Prolactinomas of the pituitary:
A. Most often produce dysfunctional uterine bleeding in women.
B. Most commonly produce infertility in men.
C. When asymptomatic, are best treated surgically early in the microadenoma
stage.
D. May enlarge during pregnancy, requiring treatment with bromocriptine or
surgery.
E. Commonly occur in patients with MEN 2.
Answer: D

DISCUSSION: Pituitary prolactinomas are the most common pituitary tumor and
cause amenorrhea, galactorrhea, or both in women. Men usually have
macroprolactinomas and experience symptoms of a space-occupying lesion of the
sella (i.e., headache). A minority of men experience impotence and infertility.
Asymptomatic prolactinomas remain stable over time and require observation only.
Symptomatic prolactinomas require treatment, usually with bromocriptine.
Prolactinomas may enlarge during pregnancy and can produce symptoms requiring
treatment with either bromocriptine or surgery. Prolactiomas are seen in as many
as 40% of patients with MEN 1.


20. Hypercortisolism:
A. Is most often ACTH-dependent, owing to an ACTH-producing pituitary adenoma.
B. Is best diagnosed by measurement of cortisol from a serum sample collected at
8 A.M.
C. Is attributable to an adrenal source if the basal serum ACTH level is above
10 pg. per ml., if the hypercortisolism is suppressed by high-dose
dexamethasone, and if an adrenal tumor is visualized radiographically.
D. May be caused by small cell carcinoma of the lung, carcinoid tumors, tumors
of the endocrine pancreas, pheochromocytoma, or medullary thyroid carcinoma
(MTC).
E. In children is most often caused by adrenocortical neoplasia.
Answer: ADE

DISCUSSION: Cushing's syndrome is most often (80% to 90%) ACTH-dependent and is
most often due to an ACTH-secreting pituitary adenoma (Cushing's disease). Some
10% to 20% of ACTH-dependent Cushing's syndrome is caused by ectopic production
of ACTH from small cell carcinoma of the lung, carcinoid tumors, tumors of the
endocrine pancreas, pheochromocytoma, and MTC. ACTH-independent Cushing's
syndrome secondary to primary adrenal pathology occurs in 10% to 20% of cases in
adults and is the most common form in children. Measurement of cortisol in two
to three consecutive 24-hour collections of urine is the best screening test for
Cushing's syndrome: plasma levels of cortisol show marked variability, and a
single random level is not helpful in establishing the diagnosis.
ACTH-independent Cushing's syndrome suppresses the pituitary and is diagnosed if
the basal serum ACTH level is suppressed below 5 pg. per ml., if the
hypercortisolism is not suppressed by high-dose dexamethasone, or if the
metyrapone stimulation test is negative. The dexamethasone suppression and
metyrapone tests do not, however, distinguish an adrenal from an ectopic ACTH
cause of Cushing's since both suppress the pituitary. Radiographic evidence of
an adrenal tumor must be supported by biochemical testing to confirm that the
adrenal is the primary cause of Cushing's syndrome.


21. Primary aldosteronism:
A. Produces a syndrome of diastolic hypertension, hypokalemia, and edema.
B. Is suggested by findings of serum potassium less than 3.5 mEq. per liter,
urinary potassium excretion greater than 30 mEq. per day, upright plasma renin
below 3 ng. per ml., and a plasma aldosterone concentration–plasma renin
activity ratio greater than 20:1.
C. Is most often due to an aldosterone-producing adrenal adenoma, which may be
distinguished from idiopathic adrenal hyperplasia by its sensitivity to diurnal
changes in ACTH and insensitivity to changes in posture.
D. May be diagnosed in hypertensive patients by demonstration of an adrenal mass
larger than 1 cm. on computed tomography alone.
E. Is best treated surgically if it is due to either aldosteronoma or idiopathic
adrenal hyperplasia.
Answer: BC

DISCUSSION: Primary aldosteronism is a syndrome of diastolic hypertension and
hypokalemia; edema typically is absent. The diagnosis relies on demonstration of
hypokalemia with inappropriate kaliuresis and inappropriately elevated plasma
aldosterone with suppression of plasma renin following sodium loading. Primary
aldosteronism most often is due to an adrenal adenoma, which must be
distinguished from idiopathic adrenal hyperplasia since resection of the adenoma
is nearly always curative, whereas adrenal resection for idiopathic adrenal
hyperplasia is curative less than 20% of the time. CT evidence of adrenal tumor
alone is inadequate to diagnose aldosteronism, even in a hypertensive patient.


22. Adrenocortical carcinoma:
A. May be suspected in a patient with rapidly progressive Cushing's syndrome and
virilizing features or in asymptomatic patients with adrenal tumors larger than
6 cm on CT.
B. Most often is diagnosed early in its course when disease is confined to the
adrenal gland.
C. Is differentiated from benign adrenocortical adenoma by tumor necrosis,
hemorrhage, and cellular features of large hyperchromatic nuclei and more than
20 mitoses per high-power field.
D. Should be resected only if disease is localized to the adrenal gland;
otherwise treatment with mitotane is indicated.
E. Carries a poor prognosis: overall 5-year survival less than 25%.
Answer: AE

DISCUSSION: Patients with adrenocortical carcinoma often present with rapidly
progressive syndromes of combined adrenocortical hormone excess. However,
approximately half do not have syndromes of hormone excess but present with
abdominal pain, increased abdominal girth, weight loss, and anorexia.
Furthermore, incidentally discovered adrenal masses larger than 6 cm. harbor
carcinoma in as many as 92% of cases. Adrenocortical carcinoma is differentiated
from adenoma by pathologic demonstration of either local invasion or distant
metastases. Most patients with adrenocortical carcinoma present with locally
advanced (stage III) or metastatic (stage IV) disease. Prognosis for these
patients is poor: 5-year survival is less than 25% in most series. Patients with
adrenocortical carcinoma should undergo primary surgical resection or palliative
surgical debulking of locally advanced, metastatic, or even recurrent disease
since response of these tumors to medical therapy, including mitotane, is poor.


23. Addisonian crisis, or acute adrenocortical insufficiency:
A. Occurs only in patients with known adrenal insufficiency or in those
receiving long-term supraphysiologic doses of exogenous steroids.
B. Can mimic an acute abdomen with fever, nausea and vomiting, abdominal pain,
and hypotension.
C. May cause electrolyte abnormalities, including hypernatremia, hypokalemia,
hypoglycemia, and hypercalcemia, as well as eosinophilia on peripheral blood
smear.
D. Should be diagnosed with the rapid ACTH stimulation test before steroid
replacement is instituted.
E. May be effectively treated with intravenous “stress-dose” glucocorticoid and
mineralocorticoid replacement.
Answer: B

DISCUSSION: Addisonian crisis, or acute adrenal insufficiency, may be seen
following even mild illness in patients with suppression of the
hypothalamic-pituitary-adrenal axis. This suppression can be produced by as
little as 1 week of supraphysiologic stress dose steroids in the year before the
stressful event. Addisonian crisis is a medical emergency that requires prompt
treatment based on clinical suspicion. Clinical findings include fever, nausea
and vomiting, abdominal pain, and hypotension. Laboratory analysis may reveal
electrolyte abnormalities, including hyponatremia, hyperkalemia, hypoglycemia,
and hypercalcemia, as well as eosinophilia on peripheral blood smear. The rapid
ACTH test is diagnostic, but it should not delay treatment with intravenous
fluid resuscitation, glucose replacement, and high-dose dexamethasone.
Dexamethasone, not hydrocortisone, should be given initially, since it does not
interfere with subsequent determination of plasma cortisol. Stress dose steroids
are inadequate once adrenal crisis has occurred, and exogenous
mineralocorticoids are given when the patient resumes oral intake.


24. Preparation for surgical removal of a pheochromocytoma includes:
A. Beta-adrenergic blockade followed by alpha-adrenergic blockade.
B. Hydration.
C. Alpha-adrenergic blockade, with or without beta-adrenergic blockade.
D. Preoperative Swan-Ganz monitoring in all patients.
E. Planning removal through an anterior, posterior, or laparoscopic approach
based upon tumor localization with CT, magnetic resonance imaging (MRI), and/or
131I-MIBG.
Answer: BCE

DISCUSSION: Principles of preoperative management for pheochromocytoma include
preoperative alpha-adrenergic blockade using phenoxybenzamine or phentolamine.
Beta-adrenergic blockade with propranolol is then used selectively in patients
who develop tachycardia, have a history of cardiac arrhythmia, or have primarily
epinephrine-secreting tumors. Beta-adrenergic blockade should be undertaken only
after successful alpha blockade is established. Patients with pheochromocytoma
frequently exhibit intravascular volume depletion, and careful hydration is
mandatory. Central venous pressure monitoring alone is helpful to guide
hydration; more intensive monitoring with a Swan-Ganz catheter is indicated for
patients with pre-existing heart disease. Formerly, the anterior approach was
preferred for adrenalectomy as it facilitated complete abdominal exploration and
search for extra-adrenal pheochromocytoma. Accurate preoperative localization
with CT, MRI, and 131I-MIBG has allowed selective use of the posterior, or even
the laparoscopic, approach for adrenalectomy.


25. Indications for surgical adrenalectomy include:
A. An adrenal mass larger than 6 cm.
B. Hypertensive patients with aldosteronism that is ACTH insensitive and posture
sensitive and who also have multiple adrenal nodules on CT.
C. Cushing's syndrome secondary to adrenal neoplasms or to persistent ectopic
ACTH syndrome when the primary tumor is inoperable.
D. Pheochromocytoma in adults and children.
E. Congenital adrenal hyperplasia secondary to 21-hydroxylase deficiency.
Answer: ACD

DISCUSSION: Adrenalectomy is indicated for the management of adrenal masses
larger than 6 cm. on CT since tumors of this size harbor carcinoma in more than
90% of cases. Surgical adrenalectomy is also the treatment of choice for primary
adrenal causes of Cushing's syndrome. Surgical adrenalectomy may be considered
as an alternative to medical adrenalectomy with metyrapone, aminoglutethimide,
or mitotane in patients with ectopic ACTH syndrome when treatment of the primary
tumor is unsuccessful. Patients with aldosteronism that is ACTH sensitive and
posture insensitive likely have an adrenal adenoma that is curable by
adrenalectomy. Aldosteronism that is ACTH insensitive and posture sensitive is
likely caused by idiopathic adrenal hyperplasia, which is best managed medically
with spironolactone, triamterene, amiloride, or nifedipine. Adrenalectomy
following preoperative alpha-adrenergic blockade, with or without
beta-adrenergic blockade, and hydration is the treatment of choice of all
pheochromocytomas. Adrenalectomy is not indicated in the management of any of
the congenital adrenal hyperplasias.


26. Incidental adrenal masses:
A. May be seen in as many as 10% of abdominal CT studies.
B. Most commonly represent pheochromocytoma; adrenocortical adenoma,
adrenocortical carcinoma, and metastases from other primary cancers occur less
frequently.
C. May represent adrenocortical carcinoma if greater than 6 cm. in diameter.
D. Should be routinely evaluated by measurement of 24-hour urine levels of
catecholamines and their metabolites, cortisol, and aldosterone plus fine-needle
aspiration.
E. Should be resected if biochemically active, if greater than 6 cm., or if they
grow over six months' follow-up.
Answer: CE

DISCUSSION: The incidental adrenal mass is seen in as many as 1.3% of abdominal
CT scans performed for other reasons. Adrenocortical adenomas are most common,
followed by adrenocortical carcinoma, metastases from other primary cancers, and
pheochromocytoma. Biochemical evaluation must weigh the prevalence of adrenal
neoplasms against the consequences of a missed life-threatening diagnosis, as in
pheochromocytoma. All adrenal masses should be evaluated for pheochromocytoma
with measurement of 24-hour urine catecholamines and their metabolites.
Aldosterone and cortisol measurement are indicated if clinical features suggest
aldosteronism or Cushing's syndrome. Fine-needle aspiration of adrenal masses is
indicated for clearly cystic lesions or if metastasis is suspected based on the
presence of another known primary. Fine-needle aspiration is not routinely
indicated in the evaluation of adrenal lesions and is contraindicated until
pheochromocytoma is definitively excluded. Adrenal lesions should be resected if
they are functional, are larger than 6 cm., or have enlarged during follow-up.


27. Which of the following statements is true about the synthesis of thyroid
hormone and its physiology?
A. The iodine utilized in hormone synthesis is derived principally from dietary
sources.
B. The role of thyroid-stimulating hormone (TSH) in thyroid physiology is
limited to regulation of the release of thyroid hormone in plasma.
C. Enough thyroxine (T 4) is stored in the normal thyroid to provide a euthyroid
state for 3 weeks despite absence of iodine intake.
D. The regulation of thyroid function involves pituitary, but not hypothalamic,
input.
Answer: AC

DISCUSSION: Iodine is necessary for the synthesis of thyroid hormone, and
approximately 200 to 500 mg is ingested daily. Most of it is absorbed from the
small intestine and is cleared from the plasma by secretion of thyroid hormone.
TSH is required for the normal production and secretion of thyroid hormone. It
is generally accepted that TSH also has a major role in thyroid growth. The
thyroid gland has a storage reserve of approximately 3 weeks. The principal
regulatory mechanisms of the thyroid gland are the
hypothalamic-pituitary-thyroid control system and the intrathyroidal regulatory
systems. TRH is produced by the superoptic and paraventricular nuclei within the
hypothalamus and passes down their axones. Following secretion into the
hypophyseal portal blood systems TRH passes to the pituitary and induces
stimulation of TSH secretion.


28. Correct statements about thyroid function tests include which of the
following?
A. Contraceptive pills and pregnancy increase the amount of thyroxin-binding
globulin (TBG), and, consequently, the total T 4 level.
B. Anticonvulsive medications and chronic debilitating illnesses decrease the
amount of TBG and, consequently, the total T 4 level.
C. Intravenous pyleography can lower the rate of active iodine uptake by the
thyroid.
D. A triiodothyronine (T 3) suppression test that demonstrates
nonsuppressibility of thyroid function is compatible with the diagnosis of
Graves' disease, toxic adenoma, or functioning carcinoma.
E. An increased serum cholesterol level in a hypothyroid patient indicates a
thyroid cause.
Answer: ABCD

DISCUSSION: Hormone binding proteins are the principal intravascular factors
influencing total hormone concentration. Various factors may cause changes in
the concentration of TBG. Contraceptive pills and pregnancy increase the amount
of TBG, while anticonvulsive medications and chronic debilitating disease may
decrease the amount of TBG. Administration of excess amounts of iodine may lower
the rate of active iodine uptake in the thyroid gland. The thyroid suppression
test is based on the principle that administration of thyroid hormone does not
suppress the patient's thyroid function when normal homeostatic mechanisms are
disrupted. Thyroid function is nonsuppressible in hyperthyroidism or in the
presence of thyroid hormone–secreting tumors. Serum cholesterol, deep tendon
reflex time, and cardiovascular functions can be measured to reflect
hypothyroidism and hyperthyroidism but do not give an indication of the etiology
of hypothyroidism.


29. Hyperthyroidism can be caused by all of the following except:
A. Graves' disease.
B. Plummer's disease.
C. Struma ovarii.
D. Hashimoto's disease.
E. Medullary carcinoma of the thyroid.
Answer: E

DISCUSSION: Common types of hyperthyroidism include diffuse toxic goiter
(Graves' disease, named after the Dublin physician Robert Graves who described
it in 1835 but known since its original description by Parry in 1786 and
described by von Basedow in 1840) and toxic adenoma or toxic multinodular goiter
(Plummer's disease). Uncommon causes include thyrotoxicosis factitia,
functioning metastatic thyroid carcinoma, trophoblastic tumors that secrete
human chorionic gonadotropin (having thyroid-stimulating properties),
inappropriate secretion of thyrotropin by pituitary tumors, struma ovarii,
iodide-induced hyperfunction, and thyroiditis.


30. Which of the following is true about the use of radioiodine to treat
hyperthyroidism?
A. If hyperthyroidism is secondary to radioiodine use, it will occur within 2
years of treatment.
B. There is a markedly increased risk of future thyroid cancer following
radioiodine therapy.
C. The risk of leukemia following radioiodine therapy is approximately 10%.
D. Mutation abnormalities occur in 15% of fetuses in utero following internal
treatment of the mother with radioiodine during pregnancy.
E. Radioiodine may pass through the placenta and lactating breast to produce
hypothyroidism in a fetus or infant.
Answer: E

DISCUSSION: Hypothyroidism secondary to radioiodine therapy increases in
frequency with time to 85% after 5 years. There has been no demonstration of an
increase in malignancies following radioiodine therapy. Radioactive iodine can
cross the placenta and lactating breast to produce hypothyroidism in a fetus or
a nursing infant and is, therefore, contraindicated.


31. Arrange the following complications of thyroid surgery (bilateral subtotal
thyroidectomy) in decreasing order of incidence in patients with Graves'
disease.
A. Laryngeal nerve paralysis.
B. Hypoparathyroidism.
C. Hypothyroidism.
D. Recurrent hyperthyroidism.
Answer: CDAD

DISCUSSION: The incidence of recurrent disease is inversely related to the
incidence of hypothyroidism and is 1% to 5%. Within 1 to 2 years, hypothyroidism
may develop in 5% to 50% of patients (with a slight additional increase in
subsequent years). The associated morbidity—related primarily to damage to the
recurrent laryngeal nerves and parathyroid glands—is estimated to be 0.5% to
3.0%.


32. The most common cause of goitrous hypothyroidism in adults is:
A. Graves' disease.
B. Riedel's thyroiditis.
C. Hashimoto's disease.
D. de Quervain's thyroiditis.
Answer: C

DISCUSSION: Hashimoto's disease was first described in Japan by Hakaru Hashimoto
in 1912 and is the best-known of the immunologic thyroid diseases. It is the
most common cause of goitrous hypothyroidism in adults and of sporadic goiter in
children. The incidence is 0.3 to 1.5 cases per 1000 population per year and it
is 10 to 15 times more common in women than in men, with the highest incidence
in the group aged 30 to 50 years.


33. Therapy for Hashimoto's disease includes:
A. Radioiodine.
B. Antithyroid medications.
C. Subtotal thyroidectomy.
D. None of the above.
Answer: D

DISCUSSION: There is no specific treatment for Hashimoto's disease. Patients are
usually followed medically, and replacement therapy with T 4 is begun in
patients with hypothyroidism that is symptomatic or associated with a goiter
that is causing pressure symptoms. Early initiation of thyroid hormone therapy
has been recommended by many to prevent further thyroid enlargement and reduce
the risk of myxedema, especially in postpartum patients. Surgical reduction of
goiter should be performed if severe pressure symptoms that have not responded
to corticosteroid therapy are present. This usually consists of subtotal
thyroidectomy. Biopsy to rule out malignancy in nodules suspicious for thyroid
carcinoma (usually papillary) or lymphoma is indicated. If carcinoma is
suspected, lobectomy should be performed, and if frozen section demonstrates
carcinoma, subtotal or total thyroidectomy should be performed.


34. Indications for surgical thyroidectomy for Graves’ disease include which of
the following?

a. Ocular involvement
b. Symptomatic large goiter
c. Women of childbearing age
d. Concomitant thyroid nodule
e. All of the above
Answer: b, c, d

DISCUSSION: Antithyroid drugs are the initial therapy in most patients with Graves’ disease,
either as a definitive therapy or in preparation for 131I therapy or surgical
ablation. Because of the high failure rate of long-term treatment with
thionamides, the use of these drugs as definitive treatment has decreased.
Drawbacks include the important 0.5% incidence of agranulocytosis and a
recurrence rate as high as 43% during the first year when the drug is stopped.
Five years after treatment only 25% of patients remain in remission. Unlike
other definitive treatments, hypothyroidism does not occur as a result of
thionamide treatment if an appropriate dosage is used. 131I has been used as
definitive treatment for patients with Graves’ disease for many years with
predictable and long-lasting good results in most patients. It has few, if any,
serious side effects. It is ablative to the thyroid gland and hypothyroidism is
a nearly inevitable result of effective therapy, although it may take years to
become clinically apparent. About 70% of patients treated with 131I are
hypothyroid within 10 years of treatment. The risk of recurrence of
hyperthyroidism after an initial response is less than 5%. Most adult patients
in the United States are treated with 131I as definitive treatment for Graves’
disease. Exceptions are women in the childbearing years where a subsequent fetus
would be affected, patients with concomitant thyroid nodules where carcinoma is
a concern, those with extremely large glands and, increasingly, those who are
opposed to 131I therapy. Thyroidectomy is an important alternative in selected
patients with Graves’ disease. Although controversial in the past, it now
appears that ocular involvement does not respond more favorably to thyroidectomy
than to 131I ablation.


35. Which of the following statements regarding anatomic relationships of the
thyroid gland are true?

a. The middle thyroid artery is intimately related to the superior laryngeal
nerve
b. The superior thyroid artery is usually the first branch of the external
carotid artery
c. Thyroidea ima arteries are found in approximately 20% of individuals
d. The parathyroid glands may lie within the pretracheal fascia
Answer: b, d

DISCUSSION: The thyroid gland is a vascular organ supplied by four main arteries: two
superior and two inferior. The superior thyroid artery usually arises as the
first branch of the external carotid artery just above the bifurcation of the
common carotid artery. The superior thyroid artery descends medially on the
surface of the inferior pharyngeal constrictor muscle to divide into an anterior
and posterior branch at the apex of the thyroid lobe on its anteromedial
surface. Its relation to the external branch of the superior laryngeal nerve is
important during thyroid lobectomy. The inferior thyroid arteries usually arise
from the thyrocervical trunks and ascend behind the carotid sheath before
passing downward and medial to enter the thyroid gland at its middle portion.
There are no arteries directly entering the lower poles from below with the
exception of a thryoidea ima artery that may replace an absent inferior artery.
Thyroidea ima arteries arise from either the innominate artery or aorta in 1% to
4% of individuals, entering the lower surface of the isthmus after coursing on
the trachea.
The pretracheal fascia is referred to as the thyroid sheath and it varies in
consistency and completeness among individuals. Posteromedially, it is usually
condensed and firmly attaches the thyroid gland to the upper two or three
tracheal rings and the cricoid cartilage. The superior parathyroid gland may lie
between the sheath and the thyroid capsule, within the sheath, posterior to it
in a potentially open plane, or occasionally anterior to the sheath. The
inferior parathyroid gland likewise may be within the sheath, particularly when
the gland is adherent to the lower pole of the thyroid.


36. Radioactive iodine is effective treatment for metastatic lung disease for
which of the following thyroid neoplasms?

a. Hürthle cell carcinoma
b. Papillary carcinoma
c. Follicular carcinoma
d. Medullary carcinoma
e. Anaplastic carcinoma
Answer: b, c

DISCUSSION: Radioactive iodine is used only in patients who have differentiated thyroid
carcinomas. It is of no value in the treatment and follow-up of patients with
Hürthle cell, medullary or anaplastic carcinomas. Most papillary carcinomas are
capable of taking up radioactive iodine. Most papillary carcinomas in patients
under 50-years of age do so, providing that the patient has had a total
thyroidectomy and there is no normal thyroid tissue to compete for the 131I.
About 20% of all papillary carcinomas do not trap sufficient iodine for imaging
or therapy. These are usually patients with papillary carcinoma variants: a tall
cell variant of papillary carcinoma, insular carcinoma, or clear cell carcinoma.
Nearly all metastatic follicular carcinomas retain the ability to trap 131I
sufficiently for imaging and for therapy. Even well-differentiated papillary and
follicular carcinoma cannot compete successfully for 131I with normal thyroid
tissue and unless this has been removed or subsequently ablated with an initial
dose of 131I, many metastases cannot be detected or treated.


37. The types of thyroiditis that can cause abnormalities of surgical
significance are which of the following?

a. Chronic lymphocytic thyroiditis (Hashimoto disease)
b. Riedel struma
c. Acute (viral) thyroiditis
d. Granulomatous (subacute) thyroiditis
Answer: a, b, d

DISCUSSION: There are three types of thyroiditis that can cause thyroid abnormalities of
surgical significance. The most common is chronic lymphocytic (Hashimoto)
thyroiditis, an autoimmune disease that can occur in any age group.
Occasionally, Hashimoto disease causes unilateral thyroid enlargement that
simulates malignancy. The rarest form of thyroiditis is Riedel struma which can
mimic a diffuse thyroid carcinoma because of the fibrotic infiltrative process
that results. Hashimoto thyroiditis is associated with reduced functional
capacity of the thyroid which increases TSH secretion, and a goiter develops.
Because of the associated fibrosis, a nodular goiter or neoplasm is suggested.
Thyroidectomy may be indicated for treatment of a solitary nodule, particularly
if it is cold, suspicious, definitely malignant or solid, and fine needle
aspirate is indeterminate.
Granulomatous, DeQuervain, and subacute thyroiditis are terms that refer to a
disease that usually occurs in young women within weeks of an upper respiratory
or other viral infection. The disease is usually self-limited, but may persist
for several months longer. In unusual patients, the disease may be confined to
one lobe and result in a firm, slightly tender mass suggesting carcinoma.
Lobectomy may be indicated to rule out the presence of malignancy. Total
thyroidectomy may be considered for persistent, painful thyroiditis after months
of steroid therapy have failed.
Goiter with a woody or fibrous component involving the adjacent strap muscles
and carotid sheaths is referred to as Riedel struma. It is rare and the cause is
not known. It is associated with other types of fibrotic processes including
retroperitoneal fibrosis, sclerosing cholangitis, and fibrosing mediastinitis.
Although considered self-limited, the process may be associated with
considerable morbidity as a result of localized pain and compression of adjacent
structures such as the airway. Occasionally, tracheostomy is required. Airway
compression may also require open biopsy resection of the isthmus with as much
as the fibrosis as possible without endangering the recurrent laryngeal nerves.


38. The principal blood supply to the parathyroid glands is which of the
following?

a. Superior thyroid arteries
b. Inferior thyroid arteries
c. Thyroidea ima arteries
d. Parathyroid arterial branches directly from the external carotid artery
e. Highly variable
Answer: b

DISCUSSION: The principal blood supply to both parathyroid glands is the inferior thyroid
artery. Parathyroid glands invariably have a single end artery supplying them,
and if the main trunk of the inferior thyroid artery is ligated during
thyroidectomy, there is no collateral blood supply to maintain their viability.
It is preferable to divide only the branch of the inferior thyroid artery medial
to those that supply either of the parathyroid glands. This requires individual
clamping of smaller vessels under the thyroid sheath as these vessels penetrate
into the thyroid capsule. Ligation of the main trunk of the inferior thyroid
artery was commonly used for bilateral subtotal thyroidectomy in the past. It
did not routinely cause hypoparathyroidism only because enough collateral blood
supply was maintained to each end artery to one or more parathyroid glands. This
is to be avoided.


39. Which of the following statements regarding papillary thyroid carcinoma are
true?

a. Seventy to 80% of new cases of thyroid carcinoma in the United States are of
the papillary type
b. Total ipsilateral lobectomy and isthmus resection are adequate therapy for
minimal thyroid carcinoma
c. Microscopic evidence of multicentric disease is present in 70% to 80% of
cases
d. Nearly all patients less than 15 years of age have metastatic disease in
local lymph nodes
Answer: a, b, c, d

DISCUSSION: Seventy to 80% of the 11,000 new patients with thyroid carcinoma diagnosed
annually in the United States have papillary carcinoma. Papillary carcinomas of
the thyroid include minimal thyroid carcinoma, intrathyroidal, and
extrathyroidal (invasion through the true thyroid capsule) disease. Minimal
thyroid carcinoma refers to those papillary carcinomas that are less than 1 cm
in diameter and not associated with any clinically apparent lymph node
metastases. In contrast to clinically significant papillary carcinomas, these
are common and are found in 2% to 13% of adult thyroid glands serially sectioned
after autopsy studies of individuals who have died from other causes. For tumors
between 0.5 and 1 cm, a total lobectomy and isthmus resection are satisfactory
treatment.
Most clinically significant papillary carcinomas are 1 to 4 cm in diameter and
are contained within the thyroid capsule. Multicentricity is relatively common
and can be found on gross sectioning of the thyroid gland in 20% to 30% of
cases. Furthermore, after serial sectioning of the entire thyroid gland in
patients with papillary carcinoma, microscopic foci are found in 70% to 80%.
Local cervical lymph node metastases are found in about 30% of all patients with
papillary carcinoma. The presence of lymph node metastases does not correlate as
closely to the size of the tumor as it does to the age of the patient. The
younger the patient, the greater the likelihood of metastatic lymph node
involvement. Nearly all patients under 15 years of age have involved metastatic
lymph nodes. The presence or absence of lymph node metastases in patients with
intrathyroidal primary papillary carcinomas does not appear to have an
appreciable effect on long-term survival if distant metastases are not present
at the time of initial treatment.


40. A 30-year-old female presents for evaluation of a palpable thyroid nodule.
Technetium-99m (99mTc) scan demonstrates a single cold nodule. The differential
diagnosis includes which of the following?

a. Carcinoma
b. A nonfunctioning adenoma
c. A thyroid cyst
d. A colloid nodule
e. An autonomous nodule
Answer: a, b, c, d

DISCUSSION: Radioisotope scanning measures the functional activity of the thyroid gland and
maps its correlation with physical findings. For routine scanning, 131I scanning
has been replaced by either 123I or 99mTc. Because of the lower radiation
exposure, cost and easy use, 99mTc is preferentially used. The differential
diagnosis of a solitary nonfunctioning nodule includes carcinoma, colloid
nodule, nonfunctioning adenoma, and cyst. The prevalence of carcinoma ranges
from 5% to 20% in cold nodules and such lesions require further evaluation. If
the scan detects other nonfunctioning areas in addition to the palpable nodule,
the gland is at low risk for carcinoma because most patients with this finding
have a multinodular goiter. A solitary, discrete area of increased activity is
more often found in a young patient with an otherwise normal thyroid gland.
Multiple hot spots are typically found in an older patient with a multinodular
goiter.
A functioning solitary nodule that is independent of TSH is considered an
autonomous nodule and can be the cause of hyperthyroidism. It is hot on
scintigraphic scan. The presence of a cold nodule is insufficient information to
determine a treatment plan. Although nearly all carcinomas are cold, most cold
nodules are benign. Fine needle aspiration cytology is considered the most
reliable means of evaluation for the diagnosis of thyroid nodules that are
nonfunctional or hypofunctional by nuclide scan.


41. Which of the following pharmacologic agents can be used in the treatment of
thyrotoxicosis to block the production of thyroid hormone?

a. Propylthiouracil
b. Propranolol
c. Methimazole
d. Carbimazole
e. Iodine
Answer: a, c, d, e

DISCUSSION: A number of substances interfere with normal production of thyroid hormone by
blocking one of the relevant enzymatic steps. Iodine was the first effective
drug to be used in the treatment of thyrotoxicosis. It can block the
organification and coupling steps in thyroid hormone synthesis as well as
prevent the release of thyroid hormone. Furthermore, iodine in large doses
probably inhibits the ability of TSH to stimulate cyclic AMP release at the
follicular cell membrane.
Commonly used antithyroid drugs are propylthiouracil (PTU), methimazole
(Tapazole) and carbimazole. Carbimazole is widely used in Great Britain, only
PTU and methimazole are commonly used in the United States. PTU interferes with
the incorporation of iodine into the tyrosine residues of thyroglobulin, thus
preventing oxidation of iodide to iodine. It also inhibits the peripheral
conversion of T4 to T3. Although both PTU and methimazole are thionamides,
methimazole does not have this peripheral effect, making PTU the preferred drug
for patients with thyroid storm. About 3% of patients taking PTU demonstrate at
least one side effect during the first 3 months of therapy whereas the
prevalence with methimazole is about 7%. These range from minor skin rashes to
agranulocytosis that can be irreversible.
b-adrenergic antagonists have been used in the treatment of hyperthyrodism.
Propranolol is the most widely used of these drugs. However, b-blockade does not
alter thyroid function per se. Rather, its effect is to provide symptomatic
relief of hyperthyroidism because of interference with the action of thyroid
hormones at the cellular level.


42. A 45-year-old woman has a solitary, nonfunctioning thyroid nodule and fine
needle cytology is nondiagnostic. Which of the following is the initial surgical
procedure of choice?

a. Total extracapsular thyroidectomy
b. Subtotal thyroid lobectomy and resection of the isthmus
c. Total extracapsular thyroid lobectomy, resection of the isthmus, and modified
unilateral neck dissection
d. Total extracapsular thyroid lobectomy and resection of the isthmus
Answer: d

DISCUSSION: Total extracapsular thyroid lobectomy and isthmus resection is the procedure of
choice when a decision has been made to surgically remove a thyroid nodule. The
entire lobe with the isthmus is submitted for frozen-section pathologic
examination if fine needle aspirate has not already resulted in a definitive
diagnosis of carcinoma. In performing total lobectomy, both parathyroid glands
are carefully preserved with their blood supply. This is done in the event that
total thyroidectomy is necessary if either the frozen or permanent histologic
sections confirm the presence of thyroid carcinoma. Total lobectomy offers the
best opportunity for accurate histologic diagnosis and is associated with the
lowest incidence of complications when the need for reoperation is considered.
In one experience, 800 consecutive cases of total unilateral lobectomy were
performed for benign or malignant nodules suspected of cancer, and no permanent
recurrent laryngeal nerve palsies occurred. Primary total lobectomy is safer
than a partial lobectomy followed by resection of the residual lobe after a
delayed diagnosis of malignancy. Reoperation to complete a lobectomy is
associated with a greater risk to both recurrent laryngeal nerve and the
parathyroids on the ipsilateral side.
Although there is controversy as to whether a total lobectomy and isthmus
resection or a total thyroidectomy is the best definitive operation for
unilateral papillary carcinoma, a subtotal lobectomy is universally considered
an inadequate operation. A definitive cancer operation can be accomplished with
one procedure in 80% of the cases when a skilled thyroid pathologist is
available for frozen-section interpretation.


43. Which of the following statements regarding fine needle aspiration cytology
of a thyroid nodule are true?

a. It differentiates neoplastic and nonneoplastic nodules in most cases
b. It does not allow differentiation of papillary, medullary and anaplastic
carcinoma
c. It cannot differentiate malignant and benign follicular or Hürthle cell
neoplasms
d. It is not recommended when a patient has a history of head and neck radiation
Answer: a, c, d

DISCUSSION: When interpreted by a skilled cytologist, fine needle aspiration is highly
accurate and is considered the preferred method of selecting patients with
thyroid nodules for surgery. Nearly 80% of patients with thyroid nodules were
spared surgical exploration as a result of such studies in one report. Because
of the risk of false-negative diagnoses (10%), advocates of this technique
emphasize the importance of clinical judgment in addition to the cytologic study
in selecting operative candidates.
In most cases, fine needle aspiration cytology enables the pathologist to
distinguish nonneoplastic from neoplastic nodules and to identify the type of
malignant tumor. Papillary, medullary, and anaplastic carcinoma all have a
typical cytologic appearance. Cytologic studies cannot differentiate malignant
from benign follicular or Hürthle cell neoplasms. In this case, a definitive
diagnosis depends on histologic examination of the entire excised tumor.
Fine needle aspiration has dramatically reduced the number of diagnostic
surgical operations for benign lesions in centers where it is used extensively.
In patients with thyroid nodules and a history of previous head and neck
radiation, operation is generally recommended regardless of cytologic findings.
In these patients, both benign and malignant lesions may develop and the chances
of sampling error are considerable.


44. Hyperthyroidism results from all of the conditions noted below. Of the
following which commonly require surgical management?

a. Graves’ disease
b. Struma ovarii
c. Functioning metastatic thyroid carcinoma
d. Toxic diffuse goiter
e. Single toxic thyroid nodule
Answer: a, d, e

DISCUSSION: Hyperthyroidism is associated with clinical manifestations related to an excess
of thyroid hormone. There are three causes of primary concern to the surgeon.
Graves’ disease, or toxic diffuse goiter, is most common, accounting for more
than 80% of all patients with hyperthyroidism. The other two relevant causes of
hyperthyroidism are toxic nodular goiter and a single toxic nodule. Common
causes of hyperthyroidism that rarely require surgery are postpartum
thyroiditis, iodine-induced hyperthyroidism, self-administered or iatrogenic
hyperthyroidism, struma ovarii, functioning metastatic carcinoma and several
rare forms of thyroiditis.


45. Which of the following statements regarding medullary carcinoma of the
thyroid are true?

a. Approximately 75% of all cases are hereditary
b. The overall 10-year survival rate is less than 10%
c. Medullary carcinoma of the thyroid is associated with both multiple endocrine
neoplasia IIa (MEN IIa) and multiple endocrine neoplasia IIb (MEN IIb) syndromes
d. Prophylactic total thyroidectomy is recommended for MEN IIa and MEN IIb
patients after the age of 10 years
Answer: c

DISCUSSION: Medullary carcinoma of the thyroid (MCT) accounts for about 7% of all malignant
tumors of the thyroid. It is familial in 20% to 30% of all cases and its
secretion of a biologic marker (calcitonin) allows detection of its presence
with the tumor is too small to palpate. MCT appears in three clinical settings.
The first is a sporadic tumor, usually in patients 30 years or older. Second,
MCT occurs as a component of the MEN IIa syndrome with or without adrenal
medullary disease (pheochromocytoma) or hyperparathyroidism. A family history of
thyroid carcinoma with or without pheochromocytoma is invariably present. MCT in
the MEN IIa syndrome is always bilateral and multicentric and arises from C-cell
hyperplasia. MCT usually does not develop before age 12 and is almost always
clinically apparent before age 30.
Lastly, MCT is found as a component of the MEN IIb syndrome without or without
bilateral adrenal medullary disease and always with the facies and autonomic
nervous system dysplasia expressed as ganglioneuromatosis from the lips to the
anus. These patients often have a marfanoid habitus and skeletal deformities as
well. The MEN IIb syndrome occurs as a sporadic mutation, but its familial
occurrence is becoming more common because patients are surviving now long
enough to reproduce. MCT or its precursor, C-cell hyperplasia, develops by age 2
in MEN IIb and is always bilateral and multicentric. As a result of its early
appearance and late detection, the disease has usually been more advanced when
treatment has been instituted. Minimal treatment of MCT is total thyroidectomy.
The overall prognosis for 10-year survival is approximately 50%, although the
tumor growth rate in individual patients has shown great variability. Patients
with the MEN IIb disease require total thyroidectomy as soon as the syndrome is
recognized, preferably by the age of 2 years. In familial cases, the
characteristic findings are sufficient even without calcitonin testing to
justify operation. If the diagnosis is not made until adolescence or later, both
central compartment and lateral node involvement require neck dissection for
definitive treatment.


46. Which of the following statements regarding thyroid physiology are true?

a. Normally about 20% of T3 is secreted directly from the thyroid gland
b. The thyroid gland is the only endogenous source of T4
c. Excess thyroid hormone results in an increase in the number of ATP-dependent
sodium pumps on the cell membrane
d. The majority of thyroid hormone in circulating plasma is bound to albumin,
thus limiting the availability of the metabolically active form
Answer: a, b, c

DISCUSSION: The thyroid is the only endogenous source of T4, whereas most T3 is produced by
the peripheral conversion of T4. This takes place in the liver, muscle, kidney
and anterior pituitary. Under normal circumstances, only about 20% of T3 is
secreted directly from the thyroid gland. In some thyroid diseases (e.g.,
Graves’ disease and toxic nodular goiter), the proportion of T3 secreted
directly by the thyroid may be markedly increased.
Once thyroid hormones are released into the circulation, they are bound to
thyroid-binding globulin (85%), albumin (10%) and transthyretin (prealbumin).
These binding proteins allow the thyroid hormones to remain soluble in plasma,
contributing to systemic distribution to various target-cell populations. A
limited amount of thyroid hormones circulates freely in the plasma in
metabolically active form (free T4, free T3).
At the cell membrane, an excess of thyroid hormone results in an increased
number of ATP-dependent sodium pumps, thus increasing resting energy expenditure
and oxygen consumption. Thyroid hormone also facilitates the transport of
glucose and amino acids across the cell membrane. In addition, T3-induced
mitochondrial oxidation of substrate results in increased ATP production.


47. The definitive treatment of choice for toxic multinodular goiter is?

a. Total thyroidectomy
b. Bilateral subtotal thyroidectomy
c. Unilateral total lobectomy on the side of dominant disease
d. 131I treatment
Answer: b

DISCUSSION: Hyperthyroidism in the patient with a multinodular goiter usually develops in
women after age 50 but is seen occasionally in younger patients. Most patients
have had a nontoxic nodular goiter for many years. Preferred treatment for most
patients with toxic multinodular goiters is thyroidectomy after adequate
preparation renders the patient euthyroid. 131I may be an alternative in
selected poor-risk patients with goiters that are not causing airway
compression. Although 131I can be used to treat the hyperthyroidism, larger and
often repeated doses of 131I may be required. 131I does not significantly reduce
the goiter size and may, because of radiation-induced thyroiditis, cause acute
enlargement. This may be hazardous in the patient with some degree of
preexisting airway compression. Any airway symptoms, particularly in patients
with substernal goiters, should be considered strong contraindications to the
use of 131I.
Standard surgical treatment of toxic nodular goiter has consisted of bilateral
subtotal thyroidectomy. Remnant size is not as important as the excision of all
autonomous nodules. Because thyroid replacement or suppression is used routinely
to prevent recurrence of goiter when a subtotal resection is done, the risk of
hypothyroidism is not a consideration in determining remnant size. Alternative
procedures are total lobectomy with isthmus resection and contralateral subtotal
lobectomy, or total thyroidectomy. The latter is not demonstrably superior and
may have more technical complications.


48. A 50-year-old male has undergone an ipsilateral thyroid lobectomy and
isthmus resection for what appeared on frozen section to be a benign nodular
lesion 2.0 cm in diameter. Seventy-two hours later the final pathology returns
and the diagnosis is a high-grade angioinvasive follicular carcinoma. What do
you recommend?

a. 99mTc bone scan to rule out occult bone metastases
b. Ipsilateral radical neck dissection
c. Total thyroidectomy
d. Observation with sequential 131I scans every 3 months
Answer: c

DISCUSSION: Follicular carcinoma accounts for about 10% of all new carcinomas of the
thyroid. Most patients are those with minimal invasion of the capsule or vessels
within the neoplasms. Such tumors are seldom diagnosed definitively by either
needle aspiration cytology or by frozen-section diagnosis at the time of
lobectomy. Most frequently, the diagnosis is made after the study of permanent
sections. Microinvasive encapsulated follicular carcinomas are rarely associated
with metastatic lymph nodes and distant metastases involving bone are also rare
at the time of diagnosis.
Angioinvasive follicular carcinomas are usually large and frequently show venous
invasion of perithyroidal and lateral neck veins at the time of diagnosis. They
may have already metastasized to different sites, most frequently the bone.
These tumors are often diagnosed at the time of operation because of these
characteristics. Most patients younger than 40 do well, but patients older than
50 years have a guarded prognosis. Follicular carcinomas of the thyroid are
treated by total thyroidectomy. Lymphatic dissections are not usually required
because only about 5% of all patients have lymphatic involvement. The most
effective therapy for bone or pulmonary metastases is radioactive iodine. The
controversy in the surgical management of follicular carcinoma is whether
patients with low-grade encapsulated neoplasms diagnosed after total lobectomy
by permanent section evaluation should be treated with total thyroidectomy. In
patients with smaller follicular neoplasms found malignant as determined by
microinvasion of the capsule, completion thyroidectomy is usually not done;
instead patients are observed closely after performing a 99mTc bone scan to rule
out occult bone metastases. Patients with large or high-grade angioinvasive
lesions require total thyroidectomy.

49. In 1990 a National Institutes of Health Consensus Development Conference
reviewed the available evidence regarding the management of asymptomatic primary
hyperparathyroidism. The panel concluded that surgical intervention was
indicated for which of the following indications?

a. Age less than 50 years of age
b. Reduced creatinine clearance
c. Presence of kidney stone(s) (as detected by abdominal radiograph)
d. Substantial reduced bone mass as determined by direct measurement
e. Markedly elevated 24-hour urinary calcium excretion
Answer: a, b, c, d, e

DISCUSSION: All of the indications listed above are considered indications for operative
treatment of asymptomatic patients with primary hyperparathyroidism. Their
conclusions regarding operative indications are summarized in Table 57-8. The
NIH Consensus Development Conference mandated close (every 6 months) follow-up
for patients with known primary hyperparathyroidism not treated by operation. In
addition, surgery was agreed upon for those patients in whom medical
surveillance was neither desirable nor suitable, such as when the patient
requests surgery, consistent follow-up is unlikely, co-existent illness
complicates management, or if the patient is young (< 50 year of age). In one
recent study of a group of 142 asymptomatic patients followed without operation,
after 10 years, more than 20% of the patients had required surgery for an
increase in serum calcium to greater than 11 mg/dL or for specific complications
attributable to the disease. Another 20% were lost to or declined follow-up. The
remainder either died of unrelated causes or had persistent asymptomatic
disease. This remains an area of considerable controversy. The best available
recommendations are derived from this October 1990 NIH Consensus Development
Conference.

 

69. The approximate 5-year survival rate for adrenocortical carcinoma is which
of the following?

a. 0%
b. 20%–25%
c. 50%–60%
d. Nearly 100%
Answer: b

The prognosis for adrenocortical carcinoma is not good. The overall 5-year
survival rate is 20% to 25% for these malignancies. When there is localized
disease at the time of surgery, the 5-year survival may be higher, in the 40% to
50% range. The true prognosis in childhood is not clear, but the data suggest a
2-year survival rate of about 20%.


70. Which of the following statements is true with respect to pheochromocytoma?

a. Pheochromocytoma associated with MEN IIa is usually unilateral and rarely
malignant; therefore, unilateral exploration through a posterior flank incision
is usually sufficient
b. Clonidine fails to suppress basal plasma-catecholamine levels in patients
with pheochromocytoma
c. NP-59 (131I-6 b-iodomethyl-19-norcholesterol) is taken up as cholesterol by
the adrenal medulla
d. The ratio of plasma 3,4-dihydroxyphenoglycol (DHPG) to norepinephrine is
generally elevated in patients with pheochromocytoma compared to patients with
essential hypertension
Answer: b, d

Pheochromocytoma associated with the MEN IIa syndrome is more often bilateral
and more often malignant, therefore, abdominal exploration through an anterior
approach is indicated. The ability to measure catecholamines in the plasma has
made possible the clonidine suppression test. In patients without
pheochromocytoma, clonidine suppresses high basal plasma concentrations into the
normal range, whereas concentrations in patients with pheochromocytoma are not
suppressed. Another use of plasma catecholamine measurements is in examining the
ratio of 3,4-dihydroxyphenoglycol (DHPG) to norepinephrine in plasma. DHPG is
released from the chromaffin cells and adrenergic neurons to a much greater
extent than norepinephrine in pheochromocytoma patients compared with patients
who have essential hypertension, i.e. the ratio of DHPG to norepinephrine is
higher in patients with pheochromocytomas.
NP-59 (131I-6 b-iodomethyl-19-norcholesterol) is taken up as cholesterol by the
adrenal cortex and is incorporated in the adrenocortical steroidogenic pathway.
This is a useful agent for imaging adrenocorticol lesions.
131I-methaiodobenzylguanidine (MIBG) is a norepinephrine analogue that is useful
in localizing pheochromocytomas throughout the body, especially when the tumors
are multiple, extraadrenal, recurrent, or metastatic.


71. Which of the following statements regarding aldosterone are true?

a. Its secretion is directly related to the serum potassium concentration
b. Angiotensin II is a more potent regulatory factor than ACTH
c. Primary hyperaldosteronism is characterized by hyperkalemia
d. Secondary hyperaldosteronism occurs with renal artery stenosis
Answer: a, b, d

Aldosterone secretion is controlled by changes in the afferent arteriolar
pressure in the renal cortex as well as by changes in sodium content in the
renal tubule. These changes are sensed by the juxtaglomerular apparatus and by
the macula densa and act through the renin angiotensin system. At least two
other factors influence aldosterone secretion. Aldosterone secretion is directly
related to the serum potassium concentration. An increase in serum potassium
directly stimulates aldosterone production, whereas a decrease in serum
potassium has the opposite effect. Because of its early point of action in the
steroidogenic pathway, ACTH also increases secretion of aldosterone, although it
is much less potent in this regard than in its stimulation of cortisol. The
stimulatory effects of potassium and ACTH on aldosterone secretion can be
overcome by angiotensin II stimulation.
Primary hyperaldosteronism is characterized by mineralocorticoid hypersecretion
which promotes a positive sodium balance and hypokalemia. About 80% of patients
with primary hyperaldosteronism have serum potassium levels of 3.5 mEq/L or
less. Causes of secondary hyperaldosteronism are related to increased renin
secretion. These include renal artery stenosis, congestive heart failure and
renal salt-wasting.


72. A term neonate is noted to have ambiguous female genitalia. This infant is
at risk for which of the following potentially life-threatening problems?

a. Cardiomyopathy with congestive heart failure
b. Sodium wasting nephropathy with hypovolemia
c. Respiratory failure from surfactant deficiency
d. Spontaneous hemorrhage from thrombocytopenia
e. Pulmonary embolus from a hypercoaguable state
Answer: b

Enzymatic defects in the steroidogenic pathway produce a syndrome known as
congenital adrenal hyperplasia. This syndrome presents predominantly in the
neonatal period with sexual ambiguity. These enzymatic defects result in a
lowered cortisol secretion. The specific enzyme defects present determine the
clinical form of the syndromes. These include a 21-hydroxylase deficiency, an
11b-hydroxylase deficiency, and a 17-hydroxylase-deficiency. The 21-hydroxylase
deficiency and the 11b-hydroxylase deficiency result in excess androgen
production in utero and result in masculinization with ambiguous genitalia in
the female newborn. Masculinizing effects in the male may not be detected until
precocious puberty becomes obvious. About 40% of patients with 21-hydroxylase
deficiency, the most common form, have salt-wasting or sodium loss by urine.
Hypovolemic shock can result. Cardiomyopathy, respiratory failure,
thrombocytopenia and pulmonary emboli are not associated with this syndrome.


73. Which of the following adrenal lesions can be treated definitively by
medical means?

a. Benign functional adrenocortical adenoma
b. Adrenocortical carcinoma
c. Congenital adrenal hyperplasia
d. Cushing disease
e. Pheochromocytoma
Answer: c

The treatment of adrenal tumors is primarily surgical removal. Although
pharmaceutical agents are useful in preparing the patient for surgery or in
palliating the patient with recurrent adrenal carcinoma, no agents render
definitive therapy for adrenal tumors. Congenital adrenal hyperplasia stands
alone among the primary, hyperfunctioning adrenal syndromes that are amenable to
medical therapy for definitive treatment. Functioning benign lesions of the
adrenal cortex that are not ACTH dependent, such as adenomas or macronodular
hyperplasia, respond to metyrapone and aminoglutethimide, which are inhibitors
of enzymes in the adrenal steroidogenic pathway. Both agents can effect a
decrease in the production of cortisol when there is no increase in ACTH
secondary to feedback stimulation. These drugs are not satisfactory long-term
agents because of their high incidence of drug reactions, patient noncompliance,
and continued growth of the lesions. They may be useful in patients whose
surgery must be delayed. Although malignant, functioning adrenocortical lesions
should be debulked whenever possible. Several chemotherapy agents offer adjunct
therapy. The most noteworthy is mitotane (o,p,-DDD). This is a cytolytic agent
that has a 30% to 70% response rate in terms of decreasing steroid output.
Unfortunately, patient survival is not affected. As mentioned, nonoperative
treatment with cortisone acetate and possibly fludrocortisone is definitive
therapy for congenital adrenal hyperplasia. Cushing disease is best treated by
transsphenoidal resection of the pituitary adenoma. Pheochromoctyoma requires
definitive surgical resection although the preoperative pharmacologic
preparation with catecholamine blockade is required.

74. Which of the following statements regarding the pituitary gland are true?

a. ADH is a product of the neurohypophysis
b. The preferred surgical approach to the pituitary gland is via the sphenoid
sinus
c. Growth hormone, ACTH, LH, FSH and serotonin are products of the
adenohypophysis
d. The adenohypophysis is regulated by neurotransmitters released by the
supraoptic hypophyseal tract
Answer: a, b

The anterior pituitary gland is the adenohypophysis which constitutes 80% of the
gland. The posterior pituitary, the neurohypophysis, constitutes the remainder
and should be considered virtually an extension of the hypothalamus of the
brain. The pituitary resides within bony confines of the sella turcica (Turkish
saddle) and is bordered laterally by the cavernous sinuses (venous), inferiorly
and anteriorly by the sphenoid sinus (air), posteriorly by the dorsum sella and
superiorly by the membranous diaphragma sella. The cavernous sinuses each
contain the siphon region of the internal carotid artery and portions of the
cranial nerves III, IV, V and VI all within the venous plexus. The optic chiasm
lies immediately above the diaphragma sella. Directly below the anterior and
inferior portions of the sella is the aerated sphenoid sinus. This is
sufficiently large in 97% of the patients to allow a transnasal, transsphenoidal
surgical approach to the pituitary.
The adenohypophysis is regulated by a portal venous system between the median
eminence of the hypothalamus and the adenohypophysis itself. This system
involves a transport of (1) thyrotropin-releasing hormone (TRH), to stimulate
the secretion of the thyroid-stimulating hormone (TSH); (2)
corticotropin-releasing hormone, to stimulate adrenocorticotropic hormone
(ACTH); (3) growth hormone-releasing hormone, to stimulate secretion of growth
hormone (GH); (4) gonadotropin-releasing hormone, to stimulate luteinizing
hormone (LH) and follicle-stimulating hormone (FSH); and (5)
prolactin-inhibitory factor (dopamine), to inhibit prolactin. The
neurohypophysis is regulated by means of direct transport of hormones through
nerve fibers from the supraoptic and paraventricular nuclei in the hypothalamus.
The neurohypophysis is a virtual extension of the hypothalamus. Products of the
neurohypophysis are antidiuretic hormone (ADH; vasopressin) and oxytocin. The
pituitary gland is not known to release serotonin.


75. Which of the following statement(s) is/are true with respect to growth
hormone secreting pituitary adenomas?

a. Fewer than 50% of patients will have growth hormone levels over 10 ng/mL
b. Oral glucose administration suppresses growth hormone levels in patients with
acromegaly
c. Over 80% of growth hormone-secreting microadenomas can be cured with
transphenoidal resection
d. Preoperative treatment of macroadenomas with a somatostatin analogue may
improve postoperative remission rates
Answer: c, d

The endocrine diagnosis of acromegaly rests largely on serum growth hormone (GH)
levels, because 90% of patients will have levels over 10ng/mL. When acromegaly
is apparent but consistently elevated growth hormone levels are not obtained,
the glucose suppression test is the most useful diagnostic procedure. In normal
patients, 1 to 2 hours after the oral administration of 100 g of glucose, the
growth hormone level falls well below 5 ng/mL. This suppression is not seen with
GH-secreting adenomas, and often a paradoxical rise in GH is observed.
The goals of treatment are to lower the circulating growth hormone or
somatomedin C levels to within a normal range and to reduce the size of the mass
lesion causing compression-related symptoms. When a microadenoma is removed
transsphenoidally, endocrine remission may be expected in 80% to 88% of cases.
When a macroadenoma is resected, postoperative remission is reported in 30% to
68% of cases. The rate of remission is inversely related to preoperative GH
levels and tumor size. Preoperative treatment of macroadenomas with a
somatostatin analogue may improve postoperative remission rates.


76. A 30-year-old woman presents with amenorrhea, headache and bitemporal
hemianopsia. Appropriate diagnostic tests include:

a. Cerebral angiography
b. Serum prolactin levels
c. Magnetic resonance imaging of the brain
d. Abdominal and pelvic CT scan
Answer: b, c

Patients with pituitary lesions present symptoms and signs related to a mass
effect on the pituitary and its surrounding structures, to hypersecretion of the
hormones by the lesion itself, or to a combination of both. As mass lesions in
the pituitary enlarge, they encounter the various contents of the cavernous
sinuses, including the third, fourth, sixth and first two divisions of the fifth
cranial nerves, as well as the internal carotid artery. The growth of a tumor in
the relatively unrestricted upward direction is much more common and often
results in compression of the optic chiasm with the resultant loss of vision,
typically a bitemporal hemianopsia. Prolactin-secreting pituitary adenomas often
present with endocrine symptoms including amenorrhea and galactorrhea in women.
In men, the loss of libido, infertility and visual loss are typical. Magnetic
resonance imaging (MRI) has evolved as the first choice for diagnostic imaging
and is often the only tool needed to reach a therapeutic decision with regard to
pituitary adenomas. With intravenous infusion of a paramagnetic substance such
as gadolinium, MRI demonstrates intrasellar tumors as small as 5 mm. In
addition, the extent of suprasellar and sphenoid sinus extension, as well as
lateral extension into the cavernous sinuses, is demonstrable. Cysts and
hemorrhage can be differentiated, as can blood flowing within an aneurysm. CT
scanning has a place in pituitary imaging if MRI scanning is unavailable. Plain
skull X-rays are not needed generally. Cerebral angiography is performed only if
an aneurysm is suspected or if a lesion is so large that occlusion or
compression of the internal carotid artery is in question. For this patient, the
symptoms clearly point to a central nervous system, pituitary etiology rather
than abdominal end organ failure with regard to the amenorrhea.


77. Which of the following condition(s) is/are associated with
hyperprolactinemia?

a. Chronic renal failure
b. Exogenous estrogen administration
c. Diabetes mellitus
d. Cirrhosis
Answer: a, b, d

Elevated serum prolactin levels do not always indicate the presence of a
pituitary tumor. Important alternative causes are chronic renal failure,
hypothyroidism, various drugs including phenothiazines, tricyclic
antidepressants, exogenous estrogen, opiates, reserpine, verapamil and others.
In addition, hepatic disease, pregnancy and a variety of pituitary and
hypothalamic lesions cause hyperprolactinemia. If the prolactin level is over
150 ng/ml, a pituitary tumor is almost invariably the cause, but often
microadenomas produce prolactin levels of less than 100 ng/ml. The size of
pituitary tumors has been shown to relate to the degree of prolactin elevation,
which may reach into the thousands of nanograms per milliliter. There are no
reliable provocative tests to differentiate prolactinomas from other causes of
hyperprolactinemias, so the diagnosis relies on ruling out other causes and
imaging of the adenoma.


78. Pituitary adenomas are best classified according to functional hormone
output. This information may be derived from which of the following?

a. Hematoxylin and eosin staining
b. Immunohistochemical staining of pituitary tissue
c. In situ hybridization studies
d. Selective venous sampling from the inferior petrosal sinuses
Answer: b, c, d

DISCUSSION: Pituitary adenomas have been classified historically as acidophilic, basophilic, and chromophobic. Adenomas may show a variable staining pattern with conventional hematoxylin and eosin dyes, so it is difficult to classify adenomas based on these stains. Immunohistochemistry, ultrastructural studies, and in situ hybridization analyses for specific hormones are the most reliable methods of classifying pituitary adenomas today. Immunohistochemical staining of pituitary adenomas with specific antibodies has reliably classified adenomas using highly purified polyclonal and monoclonal antibodies against prolactin, GH, ACTH, FSH-b LH-b, and TSH-b. Many studies with these antibodies have revealed that some pituitary tumors are composed of several cell types, which produce various hormones. Some adenomas may not store specific hormones, so immunohistochemical staining may be weak or absent. The mRNA is usually present in the cytoplasm of adenomas. Localization of mRNA for specific protein hormones is becoming more widely used in the classification of pituitary adenomas. Null-cell adenomas constitute up to 25% of pituitary neoplasms. Selective venous sampling from the inferior petrosal sinus via transfemoral catherization is an effective method to compare venous effluent from the pituitary to systemic levels for a specific hormone. In addition, this technique may demonstrate laterality. This latter issue is potentially important as certain small adenomas may not be discernible
from the gross appearance at the surgery.


79. Which of the following statements is true with respect to Cushing's Disease?

a. Pituitary microadenomas are often small and deep within the gland itself
b. The treatment of choice for hypercortisolism due to a pituitary adenoma in women of childbearing age is transsphenoidal total hypophysectomy
c. Patients who fail to remit with both surgery and radiation to the pituitary require either medical or surgical adrenalectomy
d. The long-term recurrence rate after resection of an ACTH-producing pituitary microadenoma is approximately 40%
Answer: a, c

DISCUSSION: Pituitary microadenomas secreting ACTH may be very small and are often located
deep within the gland itself. If the tumor is not evident on opening the dura,
incisions must be made into the gland and internal exploration carried out. If
no tumor is identified, then a decision must be made as to whether to resect all
or a portion of the gland. If the endocrine evidence is convincing for pituitary
origin and the patient has no desire to have children, then total hypophysectomy
is warranted. If the petrosal sinus sampling clearly indicates laterality then
appropriate hemiresection of the gland may be done.
About 75% of patients have microadenomas as the source of ACTH secretion. The
postoperative remission rate in these patients is 88% to 96%, and the long-term
recurrence rate appears to be no more than 5%. 10% to 20% of patients who
undergo exploration have macroadenomas, and the postoperative remission rates in
these patients have been reported to be from 33% to 61%. Most of these patients
require postoperative radiation therapy. Patients who fail both surgery and
radiation require either surgical adrenalectomy or medical suppression of
adrenal function.


80. A 45 year-old woman presents for evaluation of hypertension, recent onset
obesity, hirsutism and depression. Cerebral MRI does not show a pituitary
lesion. Evaluation may include determination of which of the following?

a. AM serum cortisol levels after low dose dexamethasone suppression
b. Simultaneous serum ACTH measurement in peripheral and inferior petrosal sinus
sites
c. Chest and abdominal CT scan
d. Urinary free cortisol excretion
Answer: b, c, d

DISCUSSION: The findings of Cushing Syndrome often include central obesity, hypertension,
hirsutism, fatigue, easy bruisability, stria, moon-like facies, dorsal fat pad,
and often depression or other mental changes. Less common abnormalities include
headache, osteoporosis, diabetes mellitus, galactorrhea, peripheral edema and
amenorrhea. Often, a patient presents without the classic cushingoid appearance
and complains only of severe fatigue or depression. The cause of
hypercortisolism is an ACTH-secreting pituitary adenoma (Cushing disease) in up
to 80% of cases, with remainder due either to an adrenocortical tumor or to an
ectopic neoplasm secreting ACTH or corticotropin-releasing factor.
Pituitary-dependent hypercortisolism is much more common in women(80%) and an
ectopic etiology more common in men.
Up to 60% of patients with pituitary etiologies have nondiagnostic imaging
studies, therefore, the diagnosis often relies completely on endocrine testing.
Multiple measurements of cortisol and ACTH to evaluate the diurnal pattern are
important but often misleading. They are mainly of value when clearly elevated.
The determination of urinary free cortisol excretion over 24 hours is an
extremely important measurement. If the overnight dexamethasone screening test
yields an 8 AM serum cortisol level of less than 5 ug/dl, then hypercortisolism
is rarely present. Generally, patients with a pituitary etiology of
hypercortisolism do not show suppression with the low-dose dexamethasone test,
but do with the higher dose test. Patients with adrenal or ectopic etiologies do
not experience suppression with either dose. Chest and abdominal CT scans are
appropriate to look for adrenal or lung tumors. The most specific test when the
MRI is negative and evidence implicates the pituitary, is simultaneous
measurement of ACTH levels in both inferior petrosal sinuses and a concurrent
determination of the peripheral ACTH level. This approach produces specific
information about the existence of an ACTH-secreting pituitary tumor and even
the laterality of the tumor.


81. The most common mass lesion in the sella turcica is which of the following?

a. Craniopharyngioma
b. Aneurysm
c. Benign pituitary cyst
d. Pituitary adenoma
Answer: d

DISCUSSION: Pituitary adenomas are the most common mass lesions in the sella turcica or
parasellar region. They constitute 8% to 10% of all brain tumors. Occasionally,
they are cystic and may be confused with other lesions. Craniopharyngiomas are
the next most common tumor, although these are more often suprasellar in
location. These are more common in children, but up to one third occur in
adults. They are usually cystic and are calcified in 70% of children and 40% of
adults. More rare lesions include meningiomas, germinomas, metastatic
malignancies from lung and breast primaries, gliomas, dermoids and benign
epidermoid. Rathke cysts, aneurysms, and a variety of inflammatory and
granulomatous processes.


82. Pharmacologic treatment of growth hormone (GH) excess secondary to a
pituitary adenoma may include the use of which of the following?

a. Bromocriptine
b. Vasopressin
c. Octeotide
d. Prednisone
Answer: a, c

DISCUSSION: Bromocriptine, a dopamine receptor agonist, has been demonstrated to lower GH
levels in 71% of 126 acromegalic patients. A clinical response was achieved in
up to 95% of acromegalic patients, and reduced somatomedin C levels were found
in some patients with persistently elevated GH levels. Bromocriptine does not
appear to be an effective primary treatment for acromegaly, but may help to
control GH and somatomedin C levels as an adjuvant therapy. A somatostatin
analogue, octeotide, has recently been used to treat acromegaly and has been
demonstrated to significantly reduce GH and somatomedin C levels in most
patients and normalize values in 50%. This treatment provides only minimal tumor
shrinkage, and GH levels rise again immediately following cessation of the drug.
This drug may prove to be useful as a preoperative treatment or in surgical
failures. Vasopressin and prednisone have no role in the treatment of
acromegaly.

Comments

28.07.2021 22:17

johnson

This is real take it seriously, Who will believe that herbal medicine can cure herpes, I never believe that this will work, I have to spend a lot of money getting drugs from the hospital to keep me an

25.10.2020 16:15

marwa

Aslam alikom , I need MCQs and scenario-based questions in occupational medicine, research methdology and biostatistics

23.07.2020 10:50

Fawad Ahmed Khan

nice informative I m appearing in SCE in acute medicine how can I get benefit from this source hoping for your reply

24.10.2019 12:51

Nasr

I need single best answer mcqs for vaccination

26.08.2019 16:00

feras

asalam aliokom
i need pediatric surgery MCQ

01.06.2019 18:18

Rishabh Goel

Pls upload question related to respiratory pathophysiology

17.03.2018 10:00

hanan fayed

thank you for your time and effort

18.01.2018 09:31

Syed Arslan

download free medical books from this website,
https://arslanlibrary.com/

16.12.2017 21:48

Taha Mohammad Hassanin

Thank you brother. God bless you. Can you please send MCQs segregated as per specialty i.e. MCQs for gastroenterology , hepatobiliary diseases

28.09.2017 19:52

sobhy

good

01.08.2017 15:05

dr.noha

thanx for this collection of mcq 👍👍but how can we down load this mcq ??

26.06.2017 05:29

munmun

would b of much use if a short explanatory not is attached with the answer keys...thanks.

19.06.2017 13:27

Dr Nahal

Thanks alot
Please may I have the full copy

09.06.2017 21:28

abdulwahid

How I can do download for these questions. ..thank you.

07.06.2017 16:16

Dr.Saad Zia

That really great. I applreciate the effort you put in. Keep it up

06.06.2017 11:50

sayed

thank you for sharing